Download 10022 ebook - Department of Mathematical Sciences

Document related concepts
no text concepts found
Transcript
MATH 10022
Core Mathematics II
http://www.math.kent.edu/ebooks/10022/CMII.pdf
Department of Mathematical Sciences
Kent State University
May 16, 2010
2
Contents
1 Functions
1.1 Functions . . . . . . . . . . . . . . . . . . . . . . . . . . . . .
5
5
2 Linear Equation and Applications
2.1 Solving Linear Equations . . . .
2.2 Applications of Linear Equations:
2.3 Applications of Linear Equations:
2.4 Applications of Linear Equations:
. . . . . . . .
Geometry . .
Percents . .
Proportions
.
.
.
.
.
.
.
.
.
.
.
.
.
.
.
.
.
.
.
.
.
.
.
.
.
.
.
.
.
.
.
.
17
17
24
37
49
3 Systems of Linear Equations
3.1 Graphing Method . . . . . . . . .
3.2 Substitution Method . . . . . . .
3.3 Elimination Method . . . . . . .
3.4 Applications of Systems of Linear
. . . . . .
. . . . . .
. . . . . .
Equations
.
.
.
.
.
.
.
.
.
.
.
.
.
.
.
.
.
.
.
.
.
.
.
.
.
.
.
.
.
.
.
.
.
.
.
.
.
.
.
.
57
57
66
77
86
4 Exponents and Polynomials
4.1 Positive Integer Exponents
4.2 Polynomials . . . . . . . . .
4.3 Negative Integer Exponents
4.4 Scientific Notation . . . . .
.
.
.
.
.
.
.
.
.
.
.
.
.
.
.
.
.
.
.
.
.
.
.
.
.
.
.
.
.
.
.
.
.
.
.
.
.
.
.
.
97
. 97
. 107
. 118
. 127
5 Factoring
5.1 GCF and Factor by Grouping . . . . . . . . . .
5.2 Factoring Special Products . . . . . . . . . . .
5.3 Factoring Trinomials . . . . . . . . . . . . . . .
5.4 Factoring Review . . . . . . . . . . . . . . . . .
5.5 Solving Equations by Factoring . . . . . . . . .
Exploration: Factors and x-Intercepts .
.
.
.
.
.
.
.
.
.
.
.
.
.
.
.
.
.
.
.
.
.
.
.
.
.
.
.
.
.
.
.
.
.
.
.
.
.
.
.
.
.
.
.
.
.
.
.
.
.
.
.
.
3
.
.
.
.
.
.
.
.
.
.
.
.
.
.
.
.
.
.
.
.
.
.
.
.
.
.
.
.
133
133
142
152
162
167
177
4
6 Radicals
6.1 Introduction to Radicals . . . . . . . .
6.2 Simplifying Radical Expressions . . . .
6.3 Addition and Subtraction of Radicals .
6.4 Multiplication of Radicals . . . . . . .
6.5 Rationalizing the Denominator . . . .
6.6 Rational Exponents . . . . . . . . . .
CONTENTS
.
.
.
.
.
.
.
.
.
.
.
.
.
.
.
.
.
.
.
.
.
.
.
.
.
.
.
.
.
.
.
.
.
.
.
.
.
.
.
.
.
.
.
.
.
.
.
.
.
.
.
.
.
.
.
.
.
.
.
.
.
.
.
.
.
.
.
.
.
.
.
.
.
.
.
.
.
.
181
181
187
194
200
206
215
7 Answers to Exercises
227
A Formulas
259
Chapter 1
Functions
1.1
Functions
A function f is a rule that assigns to every element x contained in a set A,
exactly one element y in a set B. Another way to to think of a function is
the phrase β€œfor every x, there is only one y.” The symbol f (x), read β€œf of
x,” is called the value of the function at x and is usually equated with
the variable y. In other words, we write y = f (x). The set A is called the
domain of the function. The domain of a function is the set of all values of
x for which the function is defined. If x is any element in the domain, then
x is called the independent variable. The domain can also be thought
of as the set of all input values. The range of the function is the set of
all possible values of f (x), as x varies throughout the domain. Hence, the
range of a function is the set of all y values assumed by the function. If y
represents an output of the function f from an input x, then y is called the
dependent variable.
A function may be defined by a set of ordered pairs, a table, an arrow
diagram, a graph, or an equation. Because functions play an important role
in mathematics, it is important to recognize when a particular relationship
represents a function.
Example 1. Determine which of the following are examples of functions.
For each function, determine the domain and range.
(a) {(1, 2), (3, 6), (6, 8), (9, 2), (12, 5)}
(b)
x
y
1
βˆ’1
2
4
βˆ’5
6
2
7
βˆ’4
9
5
6
CHAPTER 1. FUNCTIONS
(c)
1
1
2
4
3
9
βˆ’3
(d)
1
1
2
4
9
(e) x + y 2 = 9 where x is the input.
Solution 1. (a) Remember that ordered pairs are written as
(x, y). Since every element in the first position is paired
with exactly one element in the second position, this relationship defines a function. For this example,
Domain = {1, 3, 6, 9, 12}
Range = {2, 5, 6, 8}
(b) This relationship is not a function, since 2 is paired with
both 4 and 7.
(c) This relationship defines a function, since every element in
the first set is paired with exactly one element in the second
set.
Domain = {βˆ’3, 1, 2, 3}
Range = {1, 4, 9}
1.1. FUNCTIONS
7
(d) This relationship is not a function, since 2 is paired with
both 4 and 9.
(e) This relationship is also not a function. Consider when
x = 5. We have 5 + y 2 = 9 or y 2 = 4. Thus, y = ±2.
Therefore, the number 5 is paired with both 2 and βˆ’2, which
violates the definition for a function.
Note that in part (e) of the previous example, we were able to show
that the relationship was not a function by using an example. However, you
can not prove that a relationship is a function by showing it works for one
example.
Practice 1. Determine which of the following are examples of functions.
For each function, determine the domain and range. (Answer on page 10)
(a) {(3, 2), (5, βˆ’8), (7, 6), (5, 4), (9, 11)}
(b)
x
y
3
βˆ’5
6
8
9
12
12
8
15
9
(c)
3
10
5
12
7
14
9
We mentioned that a function can also be defined by a graph. The graph
of a function is a set of points (x, y) in the xy-plane such that y = f (x).
However, which set of points are graphs of functions? The following test
gives us the answer.
The Vertical Line Test: A set of points in the xy-plane is the
graph of a function if and only if no vertical line intersects the set
of points more than once.
8
CHAPTER 1. FUNCTIONS
If every vertical line intersects a curve only once at (x, y), then exactly one
value is defined by y = f (x). However, if a vertical line intersects a curve
twice at (x, y1 ) and (x, y2 ), then the value of x is paired with two different
output values. Hence, the definition for a function is violated.
Example 2. Determine if each of following curves is the graph of a function.
y
y
x
x
Solution 2. (a) Since every vertical line would intersect the
curve only once, the curve is the graph of a function.
(b) Notice that the following vertical line intersects the curve
twice.
y
x
Therefore, the curve is not the graph of a function.
Practice 2. Determine if each of the following curves is the graph of a
function. (Answer on page 10)
1.1. FUNCTIONS
9
y
y
x
x
In our definition of a function y = f (x), the independent variable x serves
as a placeholder for all input values. Therefore, to evaluate a function at a
number, we substitute the number for the placeholder.
Example 3. Consider the function f (x) = 3x2 βˆ’ 2x βˆ’ 8. Find
(a) f (0)
(c) f (2)
¡ ¢
(d) f 12
(b) f (βˆ’1)
Solution 3. (a) f (0) = 3(0)2 βˆ’ 2(0) βˆ’ 8 = βˆ’8
(b) f (βˆ’1) = 3(βˆ’1)2 βˆ’ 2(βˆ’1) βˆ’ 8 = 3 + 2 βˆ’ 8 = βˆ’3
(c) f (2) = 3(2)2 βˆ’ 2(2) βˆ’ 8 = 12 βˆ’ 4 βˆ’ 8 = 0
(d) f
¡1¢
2
=3
¡ 1 ¢2
2
βˆ’2
¡1¢
2
βˆ’8=
3
4
βˆ’ 1 βˆ’ 8 = βˆ’ 33
4
Practice 3. Consider the function f (x) = x2 βˆ’ 5x + 3. Find each of the
following. (Answers on page 10)
(a) f (βˆ’1)
(b) f (3)
(c) f (βˆ’2)
¡ ¢
(d) f 13
10
CHAPTER 1. FUNCTIONS
ANSWERS TO PRACTICE PROBLEMS
1.
(a) not a function
(b) function
(b) function;
Domain= {3, 6, 9, 12, 15};
Range = {βˆ’5, 8, 9, 12}
2.
3.
(a) 9
(c) function;
Domain= {3, 5, 7, 9};
Range = {10, 12, 14}
(b) βˆ’3
(a) not a function
(d)
(c) 17
13
9
SECTION 1.1 EXERCISES
(Answers are found on page 227.)
Determine which of the following are examples of functions. For each function, determine the domain and range.
1. {(7, βˆ’2), (5, 6), (3, βˆ’8), (4, βˆ’8), (10, βˆ’2)}
2. {(3, 1), (3, 2), (3, 3), (3, 4), (3, 5)}
3. {(1, 7), (βˆ’1, 7), (2, 7), (βˆ’2, 7)}
4. {(4, 64), (βˆ’3, βˆ’27), (βˆ’5, βˆ’125), (1, 1), (βˆ’1, 1)}
5.
x
y
3
βˆ’10
6.
x
y
1
3
7.
x
y
βˆ’2
4
8.
x
y
7
4
2
3
8
3
2
2
βˆ’1
3
7
4
3
3
4
3
5
3
βˆ’1
1
0
0
1
1
3
βˆ’6
8
1
1
8
2
4
1.1. FUNCTIONS
11
9.
βˆ’3
12
6
3
5
βˆ’5
8
10.
βˆ’2
2
4
βˆ’3
9
3
11.
1
2
3
4
5
6
12.
2
4
6
12
12
CHAPTER 1. FUNCTIONS
Determine which of the following are examples of functions.
13. y = 8x βˆ’ 2
14. x2 + y 2 = 4
15. 3x2 + 7y = 11 where x is the input.
16. 3x + 7y 2 = 11 where x is the input.
Determine if each of the following curves is a graph of a function.
17.
19.
y
y
x
18.
x
20.
y
y
x
x
1.1. FUNCTIONS
13
21.
23.
y
y
x
22.
x
24.
y
y
x
x
Evaluate the following functions.
25. For f (x) = 3x βˆ’ 7, find
(a) f (0)
(c) f (3)
(b) f (βˆ’5)
(d) f (βˆ’8)
26. For f (x) = x2 βˆ’ 3x βˆ’ 4, find
(a) f (0)
(b) f (2)
(c) f (3)
¡ ¢
(d) f 12
14
CHAPTER 1. FUNCTIONS
27. For g(x) = x3 βˆ’ x2 , find
(a) g
¡1¢
(c) g (1)
¡2 ¢
(b) g βˆ’ 12
28. For R(x) =
(d) g(βˆ’2)
4x + 2
, find
2x βˆ’ 1
(a) R(0)
(c) R(3)
(b) R(1)
(d) R(βˆ’1)
29. For s(t) = 2t2 βˆ’ 7t, find
(a) s(βˆ’2)
(c) s(1)
(b) s(βˆ’1)
(d) s(2)
30. For h(w) = βˆ’13, find
(a) h(0)
¡ ¢
(c) h βˆ’ 15
(b) h(44)
(d) h(91)
31. For f (x) = 3|x βˆ’ 5| + 7, find
(a) f (0)
(c) f (4)
(b) f (2)
(d) f (βˆ’3)
32. For P (x) = x +
1
, find
x
(a) P (1)
(c) P (2)
(b) P (βˆ’1)
(d) P (βˆ’4)
33. For f (x) = βˆ’4(x + 1)2 βˆ’ 3, find
(a) f (0)
(c) f (βˆ’3)
(b) f (βˆ’1)
(d) f (2)
1.1. FUNCTIONS
15
34. For f (x) = 5(x βˆ’ 1)2 + 3x, find
(a) f (0)
(c) f (βˆ’2)
(b) f (2)
(d) f (1)
√
35. For f (x) = 3 x βˆ’ 1 + 4, find
(a) f (1)
(c) f (10)
(b) f (5)
(d) f (17)
16
CHAPTER 1. FUNCTIONS
Chapter 2
Linear Equation and
Applications
2.1
Solving Linear Equations
In this chapter, we will discuss applications of linear equations. Before we
work any applications, we need to review solving a linear equation from
Core Mathematics I. Recall that a linear equation in one variable is an
equation that can be written in the form
ax + b = c
for real numbers a, b, and c, with a 6= 0. They are also called first-degree
equations, because the variable is raised to the first power. A solution of
an equation in one variable is a number that makes the equation true when
each occurrence of the variable in the equation is replaced by the number.
In order to solve a linear equation, we use the Addition Property of
Equality and the Multiplication Property of Equality that were discussed in
Core Mathematics I. These two properties are stated again for convenience.
They are very useful for solving all kinds of equations (not just linear ones).
Addition Property of Equality:
If a = b then a + c = b + c.
Multiplication Property of Equality: Let c 6= 0
If a = b then ac = bc.
17
18
CHAPTER 2. LINEAR EQUATION AND APPLICATIONS
Remember to perform the same operation on both sides of the
equation.
The Addition Property allows us to subtract the same quantity from both
sides by adding the opposite of the quantity. The Multiplication Property
allows us to divide each side by the same nonzero quantity by multiplying
both sides by its reciprocal. (Click here to review solving linear equations
from Core Math I.)
Example 1. Solve for x: 9(x + 5) βˆ’ 2(2x βˆ’ 1) = 3(x + 8) βˆ’ 6
Solution 1. We first use the distribute property, and then isolate
the variable.
9(x + 5) βˆ’ 2(2x βˆ’ 1) = 3(x + 8) βˆ’ 6
9x + 45 βˆ’ 4x + 2 = 3x + 24 βˆ’ 6
5x + 47 = 3x + 18
2x + 47 = 18
2x = βˆ’29
x=βˆ’
Practice 1. Solve for x:
29
2
3(5x + 1) βˆ’ (7x + 2) = 2(x + 6). (Answer on
page 21)
Next, we consider a linear equation containing fractions. How do we deal
with the fractions? We can choose to leave the fractions in the problem or to
rewrite the equation without fractions. Remember that by the multiplication
property of equality we can multiply both sides of our equation by the same
nonzero constant without changing the solution. If we choose to eliminate
the fractions, which constant do we choose? Consider
µ
¶
1
1
x
xβˆ’
= + 7.
2
3
4
2.1. SOLVING LINEAR EQUATIONS
19
If we multiply both sides by two, the fraction on the right hand side of
the equation would not be eliminated. If we multiply by four, the 31 will
still remain inside the parentheses. Therefore, it might be easier to clear all
parentheses using the distributive property before multiplying every term
by the least common denominator. When we use the distributive property
first, we get
1
1
x
x βˆ’ = + 7.
2
6
4
Now we see that if we multiply by twelve, all the fractions in the equation
will be eliminated. Recall that in this example, twelve is the least common
multiple; that is, the smallest natural number that is a multiple of all of the
denominators involved. (Click here to review least common multiple from
Core Math I) Therefore, we can eliminate fractions by multiplying each term
by the least common multiple of all the denominators. The next example
illustrates this procedure.
It is important to remember when clearing fractions that all terms need
to be multiplied by the least common denominator, not just the fractions.
µ
¶
1
1
x
Example 2. Solve for x:
xβˆ’
= +7
2
3
4
Solution 2. We first distribute, and then eliminate the fractions
by multiplying every term by 12.
µ
¶
1
1
x
xβˆ’
= +7
2
3
4
1
1
x
xβˆ’ = +7
2
6
4
µ ¶
µ ¶
³x´
1
1
12
x βˆ’ 12
= 12
+ 12(7)
2
6
4
6x βˆ’ 2 = 3x + 84
3x βˆ’ 2 = 84
3x = 86
x=
Therefore, the solution is x =
86
.
3
86
3
20
CHAPTER 2. LINEAR EQUATION AND APPLICATIONS
2
Practice 2. Solve
3
µ
¶
1
3x
2x βˆ’
=
+ 2 (Answer on page 21)
4
5
Suppose we apply our properties and the variable is eliminated. What
is the solution? The next example illustrates one such case.
Example 3. Solve for x:
1
5(3x βˆ’ 2) + (4x + 6) = 10x + 7(x + 1)
2
Solution 3. We first distribute, and then isolate the variable.
1
5(3x βˆ’ 2) + (4x + 6) = 10x + 7(x + 1)
2
15x βˆ’ 10 + 2x + 3 = 10x + 7x + 7
17x βˆ’ 7 = 17x + 7
βˆ’7 6= 7
Therefore, there is no solution to this equation.
When an equation has no solution this means that there is no value of
x for which the expression will ever yield a true statement. Does this mean
that whenever the variable is eliminated from the equation, there will be no
solution? The next example illustrates that this is not the case.
Example 4. Solve for x:
1
1
(x βˆ’ 1) +
4
3
µ
¶
9
1
3
3x +
= (4x + 1) βˆ’ x
4
2
4
Solution 4. We first distribute, and then eliminate the fractions
2.1. SOLVING LINEAR EQUATIONS
21
by multiplying every term by 4.
µ
¶
1
9
1
3
1
(x βˆ’ 1) +
3x +
= (4x + 1) βˆ’ x
4
3
4
2
4
1
1
3
1 3
x βˆ’ + x + = 2x + βˆ’ x
4
4
4
2 4
µ ¶
µ ¶
µ ¶
µ ¶
µ ¶
1
3
1
3
1
4
+ 4(x) + 4
= 4(2x) + 4
βˆ’4
x βˆ’4
x
4
4
4
2
4
x βˆ’ 1 + 4x + 3 = 8x + 2 βˆ’ 3x
5x + 2 = 5x + 2
2=2
Therefore, the answer is all real numbers.
When solving a linear equation, if the variable is eliminated and the
resulting statement is true then the answer is all real numbers. This means
that the original equation is an identity. An identity is a statement that is
true regardless of what number is replaced in the variable. A linear equation
that has one solution is called a conditional equation. If the variable is
eliminated and the resulting statement is false, then the equation has no
solution.
µ
¶
1
21
Practice 3. Solve: 6(4x βˆ’ 3) = (6x + 4) + 4
x βˆ’ 5 (Answer below)
2
4
¶
µ
¶
µ
1
1
Practice 4. Solve: 2 βˆ’ 3 4x +
= 4x βˆ’ 8 2x +
(Answer below)
5
3
ANSWERS TO PRACTICE PROBLEMS
1. x =
2. x =
11
6
65
22
3. all real numbers
4. no solution
22
CHAPTER 2. LINEAR EQUATION AND APPLICATIONS
SECTION 2.1 EXERCISES
(Answers are found on page 229.)
Solve each equation.
1.
7(5x + 1) = 3(x + 2)
2.
3(7x βˆ’ 9) = 5(x βˆ’ 1) + 6
3.
7(x βˆ’ 6) = 2(x + 4) + 5x
4.
2x βˆ’ 9 = 2(x + 3) + 5x
5.
8x βˆ’ 7(6x βˆ’ 7) + 2 = 3
6.
3x βˆ’ 2(x + 5) = 7x βˆ’ 9
7.
4(x + 2) βˆ’ 2(2x βˆ’ 1) = 10
8.
4(2x βˆ’ 3) + 8(x + 1) = 2x βˆ’ 1
9.
5(3x βˆ’ 4) + 6x = 4x βˆ’ 9
10.
4x + 3(2x βˆ’ 9) + 6(x βˆ’ 1) = 0
11.
9(x + 1) + 6(x + 3) βˆ’ 4(x + 2) = 0
12.
3(4x βˆ’ 9) βˆ’ 7(5x + 1) = 2(x + 3) βˆ’ 6
13.
3(x + 5) βˆ’ 8(x + 3) = βˆ’5(x + 2)
14.
2
1
17
βˆ’ x + 2x = x +
7
2
2
2.1. SOLVING LINEAR EQUATIONS
15.
1
1
(2x βˆ’ 3) + 5 = (3x + 4)
2
3
16.
3
1
5
xβˆ’ x=xβˆ’
5
10
2
17.
1
1
(x + 18) + (2x + 3) = x + 3
9
3
18.
1
3
19.
1
1
βˆ’ (x βˆ’ 12) + (x + 2) = x + 4
4
2
20.
µ
¶
5
1
1
βˆ’ xβˆ’ xβˆ’
= (x + 1)
6
2
4
21.
µ
¶
1
1
2
xβˆ’ x+
= (x + 4)
3
4
12
22.
1
1
(x + 3) + (x βˆ’ 6) = x + 3
3
6
23.
1
3
(x + 2) + (x + 4) = x + 5
2
4
24.
9
1
1
1
(8x βˆ’ 1) + = (4x + 5) βˆ’
4
4
2
2
(6x βˆ’ 7) = 12 (4x βˆ’ 5) +
1
6
(Click here for additional linear equation exercises from Core Math I)
23
24
CHAPTER 2. LINEAR EQUATION AND APPLICATIONS
2.2
Applications of Linear Equations: Geometry
We now consider how linear equations are used to solve application problems.
The key to solving any application problem begins with the translation. Can
you think of any words that represent addition, subtraction, multiplication
or division? Below are some of the key words that are used to represent the
different operations.
β€’ Addition – sum, added to, increased by, more than
β€’ Subtraction – difference, minus, decreased by, subtracted from
β€’ Multiplication – product, times, double (2x), tripled (3x)
β€’ Division – quotient, ratio, divided by
Although there are no step by step directions that apply to every application problem, stated below are some suggested steps.
1. Read the problem carefully and explain what your variable represents.
2. Create and label any pictures, charts, or diagrams that will ease your
problem solving.
3. Translate the problem into an equation. Be sure that you can read
the problem back.
4. Use the addition and multiplication properties to solve the equation.
5. Check that your answer makes sense in the problem and that you
have answered the question being asked. If the problem asks for two
answers, make sure you give both.
Example 1. The sum of twice a number and three is seven more than the
number. Find the number.
Solution 1. Let x = the number.
We translate the expression to obtain
2x + 3 = 7 + x.
2.2. APPLICATIONS OF LINEAR EQUATIONS: GEOMETRY
25
Next, we have
2x + 3 = 7 + x
x+3=7
x=4
Therefore, our number must be 4.
Practice 1. The sum of three times a number and four is the same as the
number increased by two. Find the number. (Answer on page 31.)
When translating, the order the words appear in the sentence is sometimes important. To illustrate this, consider the difference between the
following expressions:
β€’ The sum of four times a number and 5 is translated 4x + 5.
β€’ Four times the sum of a number and 5 is translated 4(x + 5).
Example 2. Three times the difference between twice the number and four
is five times the number increased by six. Find the number.
Solution 2. Letting x = the number, we translate the expression
as
3 (2x βˆ’ 4) = 5x + 6.
Thus,
3 (2x βˆ’ 4) = 5x + 6
6x βˆ’ 12 = 5x + 6
6x βˆ’ 12 βˆ’ 5x = 5x + 6 βˆ’ 5x
distributive property
subtracting 5x
x βˆ’ 12 = 6
x βˆ’ 12 + 12 = 6 + 12
x = 18
Hence, the number is 18.
adding 12
26
CHAPTER 2. LINEAR EQUATION AND APPLICATIONS
CAUTION: Remember when working with differences, the order
is important. The expressions a minus b or the difference of a and
b are both translated a βˆ’ b. However, a less than b is translated
b βˆ’ a.
Practice 2. Six less than four times a number is the same as twice the sum
of the number and three. Find the number. (Answer on page 31.)
Two integers which differ by 1, such as page numbers in a book, are
called consecutive integers. Examples are 4 and 5, 9 and 10, 15 and 16,
etc. In an application problem, if we let x be the first integer, how would
we represent the second integer? If we consider our example of 4 and 5, we
see that we need to add 1 to the first integer to obtain the second integer.
Hence, if x is the first integer, then x + 1 is the second. For consecutive even
or consecutive odd integers, the numbers now differ by two; for example, 7
and 9, or 4 and 6. How do we set up the variables for this problem? The
next example answers this question.
Example 3. Find two consecutive even integers such that the smaller added
to three times the larger gives a sum of 62.
Solution 3. Since we are dealing with consecutive even integers
we will let
x = first integer
x + 2 = second integer
Translating the expression yields
x + 3 (x + 2) = 62
x + 3x + 6 = 62
distributive property
4x + 6 = 62
4x + 6 βˆ’ 6 = 62 βˆ’ 6
4x = 56
56
4x
=
4
4
x = 14
subtract 6
divide by 4
Since the problem asks for two consecutive integers the answer
is 14 and 16.
2.2. APPLICATIONS OF LINEAR EQUATIONS: GEOMETRY
27
Practice 3. Find two consecutive positive integers such that the sum of
three times the smaller and five times the larger is 85. (Answers on page 31.)
We now turn our attention to the use of formulas to solve an application
problem. For carpeting a floor, painting a wall, and building a deck, it is
sometimes beneficial to know the square units of the items being worked on.
For other jobs, knowing the perimeter is useful. For those not familiar with
these formulas, they can be found in Appendix A on page 259.
Example 4. Mary wants to carpet her living room which is 15 feet wide by
17 feet long. What is the minimum square footage of carpeting Mary must
purchase?
15 ft
17 ft
Solution 4. To solve we use the area formula for a rectangle
with w = 15 and l = 17. Thus,
Area in square feet = l · w
= (17 ft)(15 ft)
= 255 square feet
Practice 4. Jennifer wants to spread lawn fertilizer on her back yard which
is 105 feet wide by 75 feet long. How many square feet does she need to
cover? If each bag of fertilizer covers 1, 000 square feet, how many bags
must Jennifer purchase? (Answers on page 31.)
We now consider an example where the formula is being used without
substituting in a number for the length and width.
Example 5. The length of a rectangle is 2 inches more than five times
the width. If the perimeter is 106 inches, find the length and width of the
rectangle.
28
CHAPTER 2. LINEAR EQUATION AND APPLICATIONS
Solution 5. First, since both dimensions are in terms of the
width we have
w = the width of the rectangle
5w + 2 = the length of the rectangle
w
5w + 2
Next, we know the perimeter of any rectangle is given by P =
2w + 2l. Since the perimeter of this rectangle is 106 inches we
have
P = 2w + 2l
106 = 2w + 2(5w + 2)
106 = 2w + 10w + 4
106 = 12w + 4
102 = 12w
8.5 = w
Since the problem asks for both the width and length, we find that
width = 8.5 feet
length = 5(8.5) + 2 = 44.5 feet
Practice 5. The amount of fencing required to enclose a small rectangular
garden is 34 feet. If the length is 1.5 times the width, find the length of the
garden. (Answer on page 31.)
We now turn our attention to some angle properties from geometry.
Three of these definitions are stated below.
β€’ Vertical Angles are opposite angles formed by intersecting lines.
Vertical angles always have the same measurement. In the following
diagram 1 and 3 are vertical angles. Likewise, 2 and 4 are vertical
angles.
2.2. APPLICATIONS OF LINEAR EQUATIONS: GEOMETRY
29
2
3
1
4
β€’ Supplementary Angles are two angles whose sum is 180β—¦ . If A and
B are supplementary angles, then we say that A is the supplement of
B. Since 180β—¦ is the measure of a straight line, supplementary angles
form a straight line. In the following diagram, 1 and 2 are supplementary angles. Likewise, in the previous diagram (with respect to vertical
angles) 1 and 2 are supplementary; 2 and 3 are supplementary; 3 and
4 are supplementary; and 1 and 4 are supplementary.
1
2
β€’ Complementary Angles are two angles whose sum is 90β—¦ . If A and
B are complementary angles, we say that A is the complement of B,
and vice versa. In the following diagram, 1 and 2 are complementary
angles.
1
2
Let us first consider some examples to illustrate the above definitions.
Additional angle definitions can be found in Appendix A on page 259.
Example 6. Find the measure of each marked angle.
˚
(11x - 34)
˚
(4x +19)
30
CHAPTER 2. LINEAR EQUATION AND APPLICATIONS
Solution 6. First, we need to recognize that these two angles
are supplementary angles. So their sum is 180β—¦ .
11x βˆ’ 34 + 4x + 19 = 180
15x βˆ’ 15 = 180
15x = 195
x = 13
Now that we have the value of x, we substitute this into both
expressions to yield the measure of each angle; namely,
11(13) βˆ’ 34 = 109β—¦
4(13) + 19 = 71β—¦
Practice 6. Find the measure of each marked angle. (Answers on page 31.)
(3x + 25)
(6x - 17)
Example 7. Find the measure of the angle whose complement is five times
its measure.
Solution 7. Recalling the definition stated earlier, we have the
following variable expressions:
x = the angle
90 βˆ’ x = the complement of the angle
Since the complement is five times the angle (5x), we have the
2.2. APPLICATIONS OF LINEAR EQUATIONS: GEOMETRY
31
following:
5x = 90 βˆ’ x
5x + x = 90 βˆ’ x + x
6x = 90
6x
90
=
6
6
x = 15
Therefore, the measure of the angle we are looking for is 15β—¦ .
Practice 7. Find the measure of the angle whose supplement is 8 times its
measure. (Answer below.)
ANSWERS TO PRACTICE PROBLEMS
1. βˆ’1
5. 10.2 feet
2. 6
6. 67β—¦ and 67β—¦
3. 10, 11
4. 7875 square feet; 8 bags
7. 20β—¦
SECTION 2.2 EXERCISES
(Answers are found on page 229.)
1. Twice the sum of a number and three is eighteen. What is the number?
2. The sum of twice a number and three is nineteen. What is the number?
3. Twelve is two more than twice a number. What is the number?
4. If seven is added to twice a number the sum is one hundred one. What
is the number?
5. Seven times a number equals that number added to sixty. What is the
number?
6. The product of a number and two hundred two equals the sum of that
number and four hundred two. What is the number?
32
CHAPTER 2. LINEAR EQUATION AND APPLICATIONS
7. Twelve times a number equals the sum of twice that number and fifty.
What is the number?
8. The product of eight and a number equals twice the sum of that number and thirty-six. What is the number?
9. The sum of seventy-two and a number equals the number subtracted
from ninety. Find the number.
10. If the sum of a number and 8 is tripled the result is the sum of twentyfour and three times the number. Find the number.
11. The product of a number and three is the half the sum of the number
and three hundred. Find the number.
12. If the sum of a number and five is tripled, the result is one less than
twice the number. Find the number.
13. If five times the smaller of two consecutive integers is added to three
times the larger, the result is 59. Find both integers.
14. If four is added to twice a number and this sum is multiplied by three,
the result is the same as if the number is multiplied by two and four
is added to the product. What is the number?
15. The sum of three numbers is 81. The second number is twice the first
number, and the third number is three less than four times the first.
Find the three numbers.
16. A number divided by twelve added to fifteen equals fifteen subtracted
from one-fourth of the number. Find the number.
17. Find two consecutive odd integers whose sum is one hundred eighty.
18. Find two consecutive even integers whose sum is twenty-six.
19. Find two consecutive odd integers whose sum is ten thousand thirtytwo.
20. Find three consecutive even integers whose sum is ten thousand thirtytwo.
21. Find three consecutive integers whose sum is ten thousand thirty-two.
22. If twice the sum of two consecutive even integers is increased by five,
the result is 97. Find both integers.
2.2. APPLICATIONS OF LINEAR EQUATIONS: GEOMETRY
33
23. Find three consecutive integers such that the sum of three times the
middle integer and five times the smallest integer is the same as seven
times the largest integer.
24. A 17-foot wire is to be cut so that one piece is 2 feet longer than twice
the shorter piece. Find the length of both pieces.
25. Adult tickets for a show cost $5.50 while children’s tickets cost $2.50.
If there were twice as many adults as children and the total receipts
for the show were $1, 026, how many adults were at the show?
26. Kayla, Savanna, and Cheyenne are the pitchers on the local girls
softball team. Kayla pitched twelve more games than Savanna and
Cheyenne pitched four more than twice the number of games Savanna
pitched. If the team played a total of 28 games, how many games did
each girl pitch?
27. Roger, Will, and Jacob sold magazine subscriptions to raise money for
playground equipment. Will sold nine more subscriptions than Roger.
Jacob sold three less than twice as many subscriptions as Roger. If
together they sold a total of 66 magazine subscriptions, how many
subscriptions did each boy sell?
28. As an employee at the local market, Charlotte’s duties include stocking
shelves, cashier, and training new employees. Yesterday during an
eight hour workday, Charlotte trained new employees half as long as
she was cashier and stocked shelves three-quarters of an hour less than
she was cashier. How many hours did Charlotte spend on each duty?
29. Angela needs to build a rectangular storage unit. She wants the length
of the rectangle to be three feet more than twice the width. The
perimeter of the rectangle is 36 feet. Find the length and width of the
storage unit.
30. The perimeter of a rectangular garden is 126 feet. If the length of
the garden is seven feet more than three times the width, find the
dimensions of the garden.
31. A small pasture is to be fenced off with 100 yards of new fencing
along an existing fence. The existing fence will serve as one side of
a rectangular enclosure. (See diagram that follows). What integer
dimensions produce the maximum area that can be enclosed by the
new fencing?
34
CHAPTER 2. LINEAR EQUATION AND APPLICATIONS
existing fence
x
x
y
32. In problem #31, suppose that the 100 yards of fencing needs to be
used to enclose the entire area. What integer dimensions produce the
maximum enclosed area?
33. Mark decides to paint his living room walls Winter Snow. According
to the can, each gallon of paint will cover 300 square feet. If his
rectangular living room is 20 feet long and 15 feet wide with 10 foot
ceilings, how many gallons of paint will Mark need to buy to complete
his project if
(a) only one coat is needed to complete the project?
(b) two coats are needed to complete the project?
34. A thirty-nine inch wire is cut into four pieces. The second piece is six
inches less than twice the first piece. The third piece is as long as the
first piece. The fourth piece is as long as the second piece. Find the
length of each piece.
35. A thirty-six foot long board is cut into two pieces. The length of one of
the pieces is four more than three times the length of the other piece.
Find the length of the two pieces.
36. A 200-yard bolt of cloth is cut into two sections of unequal length. One
section is ten yards less than twice the length of the other section. Find
the length of the sections.
37. A 50 inch board is cut into three pieces so that the second piece is four
times as long as the first piece and the third piece is five inches longer
than the second piece. Find the length of the third piece.
38. Michael recorded two songs during his 45-minute recording session.
He took twice as long to record the second song compared to the first
song. How long did it take for him to record the first song?
39. Emma recorded three songs during her 180-minute recording session.
She took thirty minutes more to record the second song than the first,
and three times as long to record the third song as the first. How long
did she take to record each song?
2.2. APPLICATIONS OF LINEAR EQUATIONS: GEOMETRY
35
40. Suppose that the total number of tickets sold at last Thursday’s matinee at Regal Cinema was 1050. If there were half as many children’s
tickets sold as adult tickets, how many adult tickets were sold for the
matinee shows last Thursday?
41. Suppose that the total number of tickets sold at last Friday’s evening
shows at Regal Cinema was 2008. If the number of adult tickets sold
was fifty-eight more than twelve times the number of children’s tickets
sold, how many of each type ticket was sold at last Friday’s evening
shows?
42. The length of a rectangular yard is 2 less than five times the width.
If 44 yards of fencing are required to fence all four sides of the yard,
find the length of the yard.
43. The perimeter of Sally’s garden is 60 feet. What is the length of her
garden if its width is 10 feet?
Find the measure of each marked angle.
44.
46.
(5x + 11)
(3x - 15)
(11x - 37)
(7x+27)
47.
45.
(x+1)
(4x - 56)
(5x + 3)
(4x + 6)
36
CHAPTER 2. LINEAR EQUATION AND APPLICATIONS
48.
49.
(3x-1)
(4x+7)
(11x - 4)
(8x + 17)
50. Find the measure of an angle such that the sum of the measures of its
complement and its supplement is 100β—¦ .
51. The supplement of an angle measures 15β—¦ more than four times its
complement. Find the measure of the angle.
2.3. APPLICATIONS OF LINEAR EQUATIONS: PERCENTS
2.3
37
Applications of Linear Equations: Percents
Percent problems are probably the most commonly used type of problems in
every day life – sales tax, discounts, depreciation, etc. In Core Mathematics I
we discussed transforming a percent into a decimal and a decimal into a
percent. (Click here to review percents and decimals from Core Math I)
When working with percents, there are typically three types of problems:
1. Given the whole and the percent, find the part;
2. Given the whole and the part, find the percent;
3. Given the part and the percent, find the whole.
Given the whole and the percent
If we are given the whole and the percent, the part can be found by
multiplying the percent by the whole. The word β€œof” is a key to where the
multiplication takes place. Consider the following example.
Example 1. Suppose that 75% of 400 fourth graders in a school district
have passed their proficiency exams. How many fourth graders have passed
their proficiency exam?
Solution 1. Recall that 75% = 0.75. Therefore,
75% of 400 = 0.75 × 400 = 300.
Therefore, 300 fourth graders have passed their proficiency exams.
Practice 1. If 42 12 % of the 80 faculty at the local high school have master’s
degrees, how many faculty have master’s degrees? (Answer on page 43).
Given the whole and the part
Once again, we will recall that the word β€œof” is a key to where the
multiplication takes place.
Example 2. If $450 of a loan has been repaid, what percent of the $2000
loan has been repaid?
38
CHAPTER 2. LINEAR EQUATION AND APPLICATIONS
Solution 2. Let x be the percent. Then we have that
450 is x percent of 2000.
Translating and solving we get
450 = x · 2000
450
x · 2000
=
2000
2000
0.225 = x
Therefore, $450 is 22.5% of $2000.
Practice 2. A skirt originally selling for $45 is on sale for $27.90. What
percent of the original price is the sale price? What is the discount rate?
(Answer on page 43).
Given the percent and the part
Example 3. Joe decides to save 15% of his weekly salary. If he saves $75
every week, what is Joe’s weekly salary?
Solution 3. Let x = Joe’s weekly salary. Then
15% of his weekly salary is $75.
Translating we get
.15x = 75
75
.15x
=
.15
.15
x = 500.
Thus, Joe’s weekly salary is $500.
2.3. APPLICATIONS OF LINEAR EQUATIONS: PERCENTS
39
Practice 3. The Miller family decides that they can spend $120 of their
monthly income on entertainment. What is the the Miller family’s monthly
income if they spend 2.5% of their monthly income on entertainment? (Answer on page 43).
When solving application problems, it sometimes helps to write your
own formula as the next example illustrates.
Example 4. Marilyn is paid $315 per week, plus a 5% commission on sales.
What is her total earnings if her sales were $625.
Solution 4. From the problem we know that
total earnings = $315 + (5% of her sales)
= $315 + (5% of $625)
= 315 + (.05 × 625)
= 315 + 31.25
= 346.25
Hence, her total earnings are $346.25.
Practice 4. If income tax is $3, 750 plus 28% of taxable income over $28, 000,
how much is the income tax on a taxable income of $35, 000. (Answer on
page 43).
Sometimes we deal with discounts and markups in the same problem.
Example 5. Kim paid $330 for a dresser to sell at her antique shop. She
wants to price it so that she can offer a 10% discount and still make a 20%
profit off the price she paid for it. What should be the marked price of the
dresser at the antique shop?
Solution 5. Let x = the price the dresser should be marked.
Profit Kim wants to make = .20(330) = $66.
40
CHAPTER 2. LINEAR EQUATION AND APPLICATIONS
Price willing to sell the dresser: $330 + $66 = $396.
NOTE: $396 is the price after Kim offers a 10% discount.
396 = marked price βˆ’ discount
396 = x βˆ’ .10x
396 = .90x
440 = x
Thus, Kim should mark the dresser at $440.
Practice 5. Buy 4 Less marks down all remaining winter coats at a discount
of 40% off the original price. After several weeks, in a last attempt to reduce
inventory, the coats are further discounted 50% off the last marked price.
Your friend believes that this means the coats are now 90% off the original
price. Is your friend correct? If not, what percent off are the coats from the
original price? (Answers on page 43).
Now let us turn our attention to some interest problems. If you have car
loans, savings accounts, or any investments, you should be concerned about
the type of interest you are paying or that you earn on the investment.
There are two common types of interest – simple and compound. Simple
interest pays interest only on the original amount of money you deposited
or invested, called the principal. Compound interest pays interest on
the interest in the account as well. Before we discuss a formula, lets look at
the following example.
Example 6. Suppose you invest $10, 000 in a bank account that earns an
annual interest rate of 3% simple interest.
(a) How much interest would you earn in one year?
(b) How much interest would you earn in three years?
(c) How much interest would you earn in a half a year?
Solution 6.
(a) Because we are dealing with simple interest, the account
only pays interest once per year and only on the original
amount deposited. Recall that 3% = 0.03. Therefore, the
amount of interest is 0.03 × 10, 000 = $300.
2.3. APPLICATIONS OF LINEAR EQUATIONS: PERCENTS
41
(b) Once again, the account only pays interest once per
year and only on the original amount deposited. Hence, we
know that the account pays $300 in interest per year, so for
3 years we would have 3 × 300 = $900.
(c) We know that we earn $300 per year so in half a year
we would earn half as much; namely, 12 × 300 = $150.
Now, the original amount of $10, 000 is called the principal and we will
denote this by p. The interest rate is r and the time is t. Using the above
example as motivation, what would you consider a valid formula for the
interest I earned in terms of p, r, and t? Well, lets summarize our results:
1 year interest = 10, 000 × 0.03 × 1 = 300
3 years interest = 10, 000 × 0.03 × 3 = 900
1
half year interest = 10, 000 × 0.03 × = 150
2
Do you see the pattern? Principal times rate times time. We can state
this formula as follows.
Formula for simple interest is given by
I = prt
where I is the interest, p is the principal, r is the annual interest
rate, and t is the time in years.
Example 7. Mary deposits $2000 into an account earning 4.5% simple interest for 2 years. When the 2 years are over, how much money does Mary
have in the account?
Solution 7. Since we are earning simple interest we will use the
above formula with p = 2000, r = 0.045 and t = 2. Thus,
I = prt
= 2000 · 0.045 · 2
= 180.
Therefore, the amount of money in the account at the end of two
years is $2000 + $180 = $2180.
42
CHAPTER 2. LINEAR EQUATION AND APPLICATIONS
Practice 6. Savanna deposits $15, 000 into an account paying simple interest. At the end of two years, the amount in the account is $16, 875. What
is the annual percentage rate on this account? (Answers on page 43).
How much more can one earn with compound interest rather than simple
interest? Lets return to our previous example to find out.
Example 8. Mary deposits $2000 into an account earning 4.5% interest
compounded annually.
(a) Find the interest and account balance after 1 year.
(b) Find the account balance after 2 years.
(c) How much more money do you have after 2 years if interest is
compounded annually than you did in Example 7 when interest was
simple interest?
Solution 8.
(a) We use the formula to get
I = prt
= 2000 × 0.045 × 1
= 90.
So there is $90 of interest in the account and a balance of
$2090 after 1 year.
(b) For the second year, we earn interest on the interest.
The amount of interest we earn in the second year is
I = 2090 × 0.045 × 1 = 94.05
The account therefore has $2090 + $94.05 = $2184.05 after
2 years.
(c) From example 7 we found that the amount in the account after two years of simple interest is $2, 180. Hence,
the difference of $2184.05 βˆ’ $2180 = $4.05 is the additional
amount received for compound rather than simple interest.
2.3. APPLICATIONS OF LINEAR EQUATIONS: PERCENTS
43
Practice 7. The Baker Family invests $30, 000 for their son’s education into
an account paying 6.75% compounded annually. How much will the account
contain after 3 years? How much more interest will the account have earned
than if their account only paid 6.75% simple interest? (Answer below).
ANSWERS TO PRACTICE PROBLEMS
1. 34
5. The final price is 70% off the original.
2. 62%, 38%
6. 6.25%
3. $4800
4. $5710
7. $36, 494.30; $419.30 more
SECTION 2.3 EXERCISES
(Answers are found on page 231.)
1. Which is better: A 10% discount followed by a 10% markup, or a 10%
markup followed by a 10% discount? Explain your reasoning.
2. Which is better: A discount of 20% followed by the addition of sales
tax at a rate of 6%, or the addition of sales tax at a rate of 6% followed
by a discount of 20%? Explain your reasoning.
3. Mary deposited $100 in an account paying 5% annual simple interest.
At the end of one year how much interest will Mary’s account earn?
4. William deposited $1000 in an account paying 4 12 % annual simple
interest. At the end of one year how much interest will William’s
account earn?
5. Lester deposited $100 in an account paying 4 14 % annual simple interest.
At the end of two years how much will Lester earn in interest on his
account?
6. Linda deposited $1000 in an account paying 3 12 % annual simple interest. At the end of four years how much will Linda earn in interest on
her account?
44
CHAPTER 2. LINEAR EQUATION AND APPLICATIONS
7. Maurita A. Coss, a financial planner, invested money at 4% annual
simple interest one year ago today. If she earned $32 in simple interest
in one year, how much money at 4% annual simple interest did she
invest one year ago today?
8. A chemist needs a 50% solution of sulfuric acid having a volume of
72 liters. How much pure acid is in the 72 liters of 50% solution?
9. How much pure acid is in 48 liters of a 40% solution of sulfuric acid?
10. Mackenzie’s collection of twenty-four CDs is 75% country. How many
country CDs does Mackenzie have in her collection?
11. How much orange juice is in twelve cups of a 75% solution of orange
juice?
12. A math class has the names of 25 students on the roster. What is the
percent of students present on a particular day when only one of the
students is absent?
13. A geography class has the names of 30 students on the roster. If 18
of the students in the class are female, what percentage of the class is
male?
14. The Calculus class is 25% male. If there are 18 women in the class,
how many student are in this class?
15. An introductory psychology class is 40% male. If there are 300 students in the class, how many are women?
16. For General Chemistry, 20% of the students earned an A- or an A
for their final grade. If there are 35 names on the roster, how many
students earned at least an A- in the class?
17. Tanya spent one and a half hours completing her Math 10022 homework today and six hours working on her other assignments. What
percent of her total homework time today did she spend on her mathematics homework?
18. If there are 45 ounces of gold in 180 ounces of a gold alloy, what is the
percent of gold in the alloy?
19. How many ounces of pure copper are there in 300 ounces of an alloy
that is 40% copper?
2.3. APPLICATIONS OF LINEAR EQUATIONS: PERCENTS
45
20. Suppose that this year’s sale at Nordstrom included a $120 wool scarf
from Italy that before the sale sold for $150. What is the percent of
sale discount on the scarf?
21. Aaron invested $14, 000 of his $20, 000 inheritance in an account yielding 3% in annual simple interest. He invested the remainder of his
inheritance in an account earning 2% in annual simple interest. How
much did he earn in simple interest from both accounts?
22. Andrew invested $300 at 3% simple interest. On the same day he also
invested $200 at 4% simple interest. Find his total yearly earnings
from the interest on both accounts at the end of one year.
23. Amy was driving to a city 180 miles from her starting point. After she
had traveled 45 miles, what percent of her proposed journey had she
completed?
24. Jennifer has 80 coins, all nickels and dimes. If she has 30 dimes, what
percentage of her coins are nickels?
25. A plumber charges $80 in time and materials to unstop a drain. If the
parts cost $20, what percentage of his bill was for his labor?
26. An auto mechanic charges $80 in time and materials to repair a rim
leak on a tire. If the parts cost $10, what percentage of his bill was
for his labor?
27. Kim is a real estate agent who makes a 6% commission of every property she sells. How much commission did she make on the $100, 000
piece of land she recently sold to Embassy Suites for their new hotel?
28. Suppose the Beldon family decides to give 8% of their monthly income
to charity. If their income the past month was $150, 000, how much
money would they have given to charity this past month?
29. Suppose that Tom is a recent graduate of Kent State University and
decides to put $20 a week in a savings account. If his weekly takehome pay is $800, what is the percentage of his take-home pay that
he puts into savings?
30. You are offered two different real estate positions. Job 1 pays $250 per
week plus a 6% commission on sales. Job 2 pays 9% commission only.
If sales average about $20, 000 per week, which job would pay more?
46
CHAPTER 2. LINEAR EQUATION AND APPLICATIONS
31. To find the cost of a $20 item discounted 25%, Amy multiplied by 14
and Jack multiplied by 34 . Both found the right answer. Explain how
this could be.
32. Candidates A, B, and C were in an election in which 1000 votes were
cast. Candidate A received 35% of the votes while Candidate B received 52 more votes than Candidate C. Which candidate won the
election and by how many votes?
33. The Miller family spends twenty percent of their monthly salary on
food. Twice the amount spent on food is spent on the house payment,
and one half of the amount spent on food is spent for utilities. If
the total amount spent on these three amounts is $1, 050, what is the
Miller family’s monthly salary?
34. The size of the smallest angle of a triangle is 30% of the size of the
largest angle. The size of the third angles is 20β—¦ more than the smallest
angle. Find the size of each angle. (Note that the sum of the angles
of a triangle is 180β—¦ .)
35. College tuition at Local University in 2005 is up 9% that of tuition in
2004. If the tuition in 2005 is $7, 050, what was the tuition in 2004?
36. Carl decided to plant a rectangular shaped garden. If the width is
80% of the length and the perimeter is 45 feet, what is the area of the
garden?
37. Sally bought textbooks at the bookstore for $244.33, which included
6% sales tax. What did the books cost?
38. John sells a decorative wreath at his floral shop for $34.95. If the
wreath has a markup of 65%, what was John’s cost of the wreath?
(Round to two decimal places)
39. A pair of slippers originally costs $35.50. This week the slippers are
advertised at 35% off. What is the sale price of the slippers (before
tax)? (Round to two decimal places)
40. A new 10 piece cookware set sells for $348. This weekend it is on
sale for 33% off. What is the sale price of the cookware (before tax)?
(Round answer to two decimal places.)
41. Kelly bought a bike and a year later sold it for 30% less than what she
paid for it. If she sold the bike for $126, what did she pay for it?
2.3. APPLICATIONS OF LINEAR EQUATIONS: PERCENTS
47
42. In a basketball game, 17 free throws went in the basket and 3 missed.
What percent were missed?
43. A car was to be sold for a 12% discount which amounted to $1800.
How much would the car sell for after the discount?
44. Sarah paid $36, 648.12, including tax, for her new car. The sales tax
rate is 7.25%. What was the actual price of the car (before tax)?
(Round answer to two decimal places.)
45. If the federal income tax is $3, 750 plus 28% of taxable income over
$65, 000, what is the federal income tax on a taxable income of $87, 000?
46. The university basketball team won 91 games, which was 70% of the
games played. How many games did the team play?
47. At the local market, bananas originally sold for 45 cents a pound.
Today, the cost is 52 cents a pound. What is the percent increase in
the cost of bananas per pound? (Round answer to two decimal places.)
48. A laptop that was originally purchased for $1, 850 now has an estimated value of $1, 258. What is the percent decrease in the value of
the laptop?
49. Brenda purchased clothing totaling $140 before taxes. If the sales tax
rate is 6.75%, what was her total cost for the clothing?
50. Suppose you invest $25, 500 in a bank account that earns an annual
interest rate of 2 12 % simple interest.
(a) How much interest would you earn in one year?
(b) How much interest would you earn in three years?
(c) How much interest would you earn in a half a year?
51. If $12, 000 is deposited into an account earning 4% interest compounded annually,
(a) Find the interest and account balance after 1 year.
(b) Find the account balance after 2 years.
(c) How much additional money do you have after 2 years if interest is compounded annually than you would have if interest was
calculated using simple interest?
48
CHAPTER 2. LINEAR EQUATION AND APPLICATIONS
52. Beverly deposits $18, 250 into an account paying simple interest. At
the end of three years, the amount in the account is $21, 808.75. What
is the annual percentage rate on this account?
2.4. APPLICATIONS OF LINEAR EQUATIONS: PROPORTIONS
2.4
49
Applications of Linear Equations: Proportions
Much of our use of mathematics involves comparison and change. A ratio
is a quotient of two quantities. The ratio of the number a to the number b
is written
a
a to b,
, or a : b.
b
Fractions are ratios of integers but not all ratios are fractions. For example, the ratio of the diagonal of a square to one of its sides is never a ratio
of integers. Indeed, a ratio may compare any kind of quantity or magnitude
to any other kind such as dollars to ounces in Example 1.
One application of ratios is in unit pricing. This helps one to determine
which size of a product offered at different prices represents the best bargain.
Example 1. The local Bargain Basement offers the following prices for a
jar of Fluff Marshmallow Creme:
Size
15 oz
32 oz
48 oz
Price
$1.35
$2.76
$4.56
Which size represents the best buy?
Solution 1. To find the best buy, we compare the price for each
jar to the number of ounces per jar. This ratio gives the price
per ounce. The best deal, of course, will be the item with the
lowest price per ounce. The following chart gives those ratios:
Size
15 oz
32 oz
48 oz
cost per ounce
$1.35
= 0.09
15
$2.76
= 0.08625
32
$4.56
= 0.095
48
Comparing the price per ounce, we determine that the 32 ounce
jar is the best buy.
50
CHAPTER 2. LINEAR EQUATION AND APPLICATIONS
The above example illustrates that buying the largest size does not always
result in the best buy.
Practice 1. The local grocery store had the following prices on coffee:
Size
13 oz
34 oz
60 oz
Price
$2.75
$7.34
$13.52
Which size represents the best buy? (Answer on page 52).
A proportion is a statement that says that two ratios are equal. Ratios and proportions are not limited to the area of mathematics. In map
making, proportions are used to represent the distance between two cities.
Likewise, for scale models we use proportions. Even in cooking, the idea
of a proportion is used whenever we increase or decrease the ingredients to
accommodate the number of servings we want. If three of the numbers in a
proportion are known, then we can always solve for the fourth number by
multiplying both sides of the equation by the least common multiple of the
denominators.
Example 2. A new commercial states that three out of four dentists recommend the new Flash toothpaste. If 411 dentists recommend the toothpaste,
how many dentists were interviewed?
Solution 2. Let x = the number of dentists interviewed. Then
411
3
=
4
xµ
µ ¶
¶
3
411
4x
= 4x
4
x
3x = 4(411)
multiply by LCM of denominators
3x = 1644
x = 548
Hence, 548 dentists were interviewed.
Note that in the above example, there are four correct ways to set up
4
x
3
4
411
x
the proportion. They are 43 = 411
x , 3 = 411 , 411 = x , and 3 = 4 . After
multiplying by the LCM of the denominators in each proportion, we obtain
3x = 1644.
2.4. APPLICATIONS OF LINEAR EQUATIONS: PROPORTIONS
51
Practice 2. Cheyenne scored 75 goals during her soccer practice. If her
success-to-failure rate is 5 : 4, how many times did she attempt to goal?
(Answer on page 52.)
Example 3. You have just invented a new recipe to enter in the local cooking
contest. As invented, the recipe calls for 2 cups of onions for three servings.
Contest rules state that the recipe must make four servings. How many cups
of onions must be used for the recipe to make four servings?
Solution 3. Let x = the number of cups of onions for four
servings.
x
2
=
4
3 µ ¶
³x´
2
12
= 12
4
3
3x = 8
8
x=
3
2
x=2
3
multiply by LCM of denominators
2
Therefore, for four servings the recipe would require 2 cups of
3
onions.
Practice 3. A recipe that makes 3 dozen peanut butter cookies requires 1 14
cups of flour. How much flour would you require for 5 dozen cookies? (Answer on page 52).
Example 4. The driver at pump 2 filled her tank with 12.5 gallons of gasoline for $33. How many gallons of gasoline did the driver at pump 6 get if
his total was $42.90?
52
CHAPTER 2. LINEAR EQUATION AND APPLICATIONS
Solution 4. Let x = the number of gallons of gasoline for driver
at pump 6.
x
12.5
=
42.90
33
33x = 536.25
x = 16.25
Therefore, the driver at pump 6 got 16.25 gallons of gasoline.
Practice 4. Akron, OH is located approximately 60 miles west of the Pennsylvania border. An Ohio map represents this distance as 3 inches. On
11
the same map, Youngstown, OH is represented by approximately
of an
20
inch from the Pennsylvania border. How far is Youngstown, OH from the
Pennsylvania border? (Answer below).
ANSWERS TO PRACTICE PROBLEMS
1. The 13 ounce can is the best buy
1 cups of flour
3. 2 12
2. 135 goals attempted
4. 11 miles
SECTION 2.4 EXERCISES
(Answers are found on page 232.)
1. Find the best buy for each item.
(a) Salad Dressing: 8 ounces for $0.98, 16 ounces for $1.34, or 32
ounces for $2.74
(b) Chocolate Chips: 6 ounces for $0.94, 12 ounces for $1.50, or 24
ounces for $2.83
(c) Strawberry Jam: 11 ounces for $1.19, 24 ounces for $2.64, or 32
ounces for $3.68.
2. If a light flashes every 6 seconds, how many times will it flash in threefourths of an hour?
2.4. APPLICATIONS OF LINEAR EQUATIONS: PROPORTIONS
53
3. If a man 6 feet tall casts a shadow 4 feet long, how long a shadow will
be cast by a cell phone tower that is 75 feet tall?
4. If 500 sheets of paper makes a pile two inches high, how many sheets
of paper are there in a pile a foot high?
5. If the scale of a map is 13 inch equals one mile, how long is a straight
road that is one and one-half inches on the map?
6. If a cricket chirps 75 times in a minute, how many times does the
cricket chirp in 8 seconds?
7. If the human heart pumps 5 quarts of blood through the human body
in a minute, how many gallons of blood will it pump in 1 hour?
8. If John walks a mile in fifteen minutes, how far can he walk in threequarters of an hour?
9. If Grace pays $3 for two pens, how many pens can she buy with $60?
3
10. A recipe for 36 cookies requires 1 cups of chocolate chips. How many
4
cups are required for 90 cookies?
11. If 7 gallons of premium gasoline costs $17.43, how much would it cost
to fill up with 25 gallons of premium gasoline? (Round to two decimal
places)
2
12. A holiday recipe calls for 1 cups of flour. If this recipe feeds 4 people,
3
how many cups of flour would you need in the altered recipe in order
to feed 10 people?
13. Cheyenne scored 20 goals during this soccer season. If she scored 5
goals for every 9 attempts, how many goals did she miss?
1
14. A recipe that makes 4 dozen oatmeal raisin cookies calls from 1 cups
4
of flour.
(a) How much flour would you need if you doubled the recipe?
(b) How much flour would you need for half of the recipe?
(c) How much flour would you need to make 5 dozen cookies?
54
CHAPTER 2. LINEAR EQUATION AND APPLICATIONS
15. Your architect has drawn up the blueprints for your new house. In the
drawing 21 inch represents 3 feet. The length of the house is 42 feet.
How many inches would this represent on the blueprint?
16. A local business produces color markers. The shop manager has found
that out of every 1200 markers produced, 3 will be defective. How
many markers were produced on this machine, if 7 markers were found
to be defective?
17. You invented a new recipe to enter into the local cooking contest. As
invented, the recipe calls for 1 21 cups of chopped onions for four servings. The contest rules state that the recipe must make six servings.
How many cups of onions must be used in order for the recipe to make
six servings?
18. If 34 inch on a map represents an actual distance of 20 miles, what is
the distance, in miles, represented by 4 18 inches on the same map?
19. The official ratio of width to length for the US flag is 10 to 19. If a
flag is 45 inches wide, what is the length of the flag?
20. East Elementary has 1500 students. The student to teacher ratio is
20 to 1. How many teachers are employed at East Elementary?
21. East Elementary has 1500 students. The ratio of female students to
male students is 16 to 9. How many female students attend East
Elementary?
22. Wildlife officials want to estimate the number of fish in Bluegill Lake.
As a result, they tag a total of 150 fish from the lake. What is the
estimated number of fish in Bluegill Lake if six months later officials
catch 250 fish and 75 are tagged?
23. The directions on a bag of cement are to mix 1 part cement, 2 parts
sand, and 3 part gravel. If you have 14 pounds of sand, how many
pounds of cement and how many pounds of gravel should be mixed
with this sand?
24. A metal tube measuring 3 feet long weighs 28 pounds. What is the
weight of a similar tube that measures 2 14 feet?
25. If there are 7.62 cm in 3 inches, how many centimeters are there in
8 inches?
2.4. APPLICATIONS OF LINEAR EQUATIONS: PROPORTIONS
55
26. A family purchases 2 gallons of milks every 3 weeks. At this rate, how
many gallons of milk can this family expect to purchase in a year?
27. According to Physics, the weight of an object on the moon is proportional to its weight on Earth. If a 144 pound woman weighs 24 pounds
on the moon, what would a 84 pound boy weigh on the moon?
28. The property tax for a home with an assessed value of $145, 000 is
$1, 373.88. What is the property tax for a house with an assessed
value of $124, 000
56
CHAPTER 2. LINEAR EQUATION AND APPLICATIONS
Chapter 3
Systems of Linear Equations
3.1
Graphing Method
In Core Mathematics I, we graphed a linear equation in two variables. (Click
here to review graphing linear equations in two variables from Core Math I.)
In this section we consider the solution to a system of linear equations. A
system of linear equations consists of two or more linear equations. A
solution of a system of linear equations is an ordered pair of numbers that
is a solution to every equation in the system.
Example 1. Determine if (βˆ’3, 2) is a solution of the following system:
(
2x + 5y =
4
4x βˆ’ 3y = βˆ’18
Solution 1. In order for (βˆ’3, 2) to be a solution for the system,
it must be a solution to each of the linear equations. To check,
we substitute x = βˆ’3 and y = 2 into each equation. Checking
the first equation, we find
?
2(βˆ’3) + 5(2) = 4
?
βˆ’6 + 10 = 4
4 = 4X
57
58
CHAPTER 3. SYSTEMS OF LINEAR EQUATIONS
Since we ended up with a true statement, (βˆ’3, 2) is a solution
of the first linear equation. However, before it can be a solution
of the system, it must also be a solution of the second equation.
Checking the second equation, we find
?
4(βˆ’3) βˆ’ 3(2) = βˆ’18
?
βˆ’12 βˆ’ 6 = βˆ’18
βˆ’18 = βˆ’18X
Thus, (βˆ’3, 2) is a solution of the given system.
(
Practice 1. Determine if (βˆ’1, 4) is a solution of
3x + 2y = 5
4x + 5y = 12
(Answer
on page 64)
The set of all ordered pairs that are solutions of a system forms the
solution set of the system. The goal is to find the solution set of a given
system. In Example 1, (βˆ’3, 2) was a solution of the given system. If we
graph each equation in the system on a common set of axes, we have the
following graph.
y
J
]J
6
5
J
J
4
c
c J
c
3
cJ
2
J
c
c
J
1
Jcc
¾
J c0 1
βˆ’5 βˆ’4 βˆ’3 βˆ’2 βˆ’1 c
J
Jβˆ’1
J
βˆ’2
J
βˆ’3
J
^
J
βˆ’4
βˆ’5
2
3
4
-
5
x
?
Notice that (βˆ’3, 2) is a point on both graphs of the linear equations.
In other words, (βˆ’3, 2) is a point of intersection of the two graphs of linear
3.1. GRAPHING METHOD
59
equations involved in the system. Therefore, one method for solving a system
of linear equations is by graphing and identifying the points of intersection.
Steps for the Graphing Method:
1. Graph each linear equation on a common set of axes.
2. Locate the points of intersection.
3. Check each proposed point of intersection to confirm that it is
a solution to each linear equation in the system.
Example 2. Solve the following system by graphing.
(
x+y =1
βˆ’x + y = 3
Solution 2. We begin by graphing each of the linear equations on
a common set of axes. Both equations can be graphed by using
the xβˆ’intercept and yβˆ’intercept. We will plot one additional
point for each line as well. Therefore, we have
x+y =1
x
1
0
βˆ’1
y
0
1
2
βˆ’x + y = 3
x
βˆ’3
0
1
y
0
3
4
Graphing these two lines by plotting the above points, we obtain
60
CHAPTER 3. SYSTEMS OF LINEAR EQUATIONS
y
6
5
@
I
@
@
4
@
@
¾
¡
¡
3
¡
@¡2
¡@1
¡
@
βˆ’5 βˆ’4 βˆ’3 βˆ’2 βˆ’1
0
¡
¡
¡
ª
¡
µ
¡
¡
βˆ’1
βˆ’2
@1 2 3
@
@
@
R
4
-
5
x
βˆ’3
βˆ’4
βˆ’5
?
It appears that the two lines intersect at (βˆ’1, 2). In order to
conclude that (βˆ’1, 2) is our solution, we must check to see if it
is a solution to each equation. Therefore, substituting x = βˆ’1
and y = 2 in both equations we get,
first equation:
?
(βˆ’1) + 2 = 1
1 = 1X
second equation:
?
βˆ’(βˆ’1) + 2 = 3
3 = 3X
Hence, (βˆ’1, 2) is the solution of the system.
(
Practice 2. Solve the following system by graphing.
3x βˆ’ y = 7
xβˆ’y =5
(Answer
on page 64)
Example 3. Solve the following system by graphing.
(
2x + y = 5
x + 3y = 5
Solution 3. We will also graph each of these equation by plotting
points. We notice that
3.1. GRAPHING METHOD
61
2x + y = 5
x
0
1
3
x + 3y = 5
y
5
3
βˆ’1
x
βˆ’1
2
5
y
2
1
0
Graphing each of these lines on a common set of axes, we obtain
y
P
i
PP
¾
βˆ’5 βˆ’4 βˆ’3 βˆ’2
KA5 6
A
4
A
A
3
A
PP2
PPA
1
P
AP
A PPP
q P
βˆ’1
0
1
2A 3
4
5
x
A
βˆ’1
A
βˆ’2
A
A
βˆ’3
A
βˆ’4
A
AU
βˆ’5
?
It appears that the graphs intersect at (2, 1). Checking this into
each equation, we see that it results in a true statement for each
since
first equation:
?
2(2) + 1 = 5
5 = 5X
second equation:
?
2 + 3(1) = 5
5 = 5X
Therefore, (2, 1) is the solution of the given system.
(
Practice 3. Solve the following system by graphing.
3x βˆ’ y = βˆ’6
βˆ’x + 2y = βˆ’3
(An-
swer on page 64)
In the previous two examples we solved a system of linear equations
that had exactly one solution. However, if we consider all the possible
62
CHAPTER 3. SYSTEMS OF LINEAR EQUATIONS
outcomes that we can have when graphing two lines, then we see that two
lines can either intersect in exactly one point, can never intersect, or intersect
everywhere. If the lines never intersect, then they must be parallel. Recall
from Core Mathematics I that two lines are parallel if they have exactly the
same slope. If the lines intersect everywhere then the lines are equivalent,
and we would have an infinite number of solutions. The next two examples
illustrate each of these cases.
(
x βˆ’ 2y = 4
Example 4. Solve the following system by graphing.
βˆ’2x + 4y = 0
Solution 4. Again, plotting points we find
x βˆ’ 2y = 4
x
0
2
4
βˆ’2x + 4y = 0
y
βˆ’2
βˆ’1
0
x
0
2
4
y
0
1
2
Therefore, plotting these points we obtain the following graphs
y
5
6
4
3
©
*
©
©
©
1
©©
*
©
©©
©
©
¾
βˆ’5 βˆ’4 βˆ’3 βˆ’2 βˆ’1 © 0
1
2
©
©3© 4 5 x
©
©
βˆ’1
©©
©©
©
βˆ’2
©©
©
©
¼
©
©© βˆ’3
©
¼
©
βˆ’4
2
βˆ’5
?
The two lines appear to be parallel, but graphs can be deceiving.
Remember that parallel lines have identical slopes. So, in order
3.1. GRAPHING METHOD
63
to guarantee these lines are parallel, we determine the slope of
each line by writing both lines in slope-intercept form.
x βˆ’ 2y = 4
βˆ’ 2x + 4y = 0
βˆ’2y = βˆ’x + 4
1
y = xβˆ’2
2
4y = 2x
1
y = x
2
Hence, the lines really are parallel since they have the same slope
of m = 12 . Therefore, the system has no solution.
(
Example 5. Solve the following system by graphing.
2x βˆ’ y =
4
βˆ’4x + 2y = βˆ’8
Solution 5. The intercepts of the first line are (2, 0) and (0, βˆ’4),
and the intercepts for the second line are also (2, 0) and (0, βˆ’4).
Since the intercepts are equal, the two lines must be the same.
Therefore, both lines are given by the following graph
y
5
6
¢Μ§
¢
4
¢
3
¢
2
βˆ’5 βˆ’4 βˆ’3 βˆ’2 βˆ’1
0
βˆ’3
¢
¢2
1
3
4
-
5
x
¢
βˆ’1
βˆ’2
¢
¢
¢
1
¾
¢
¢
¢
¢
¢
¢
¢βˆ’5
¢® ?
βˆ’4
The lines intersect infinitely many times, so the system has an
infinite number of solutions.
(
3x βˆ’ y = 4
Practice 4. Solve the following system by graphing.
βˆ’9x + 3y = βˆ’12
(Answer on page 64)
64
CHAPTER 3. SYSTEMS OF LINEAR EQUATIONS
(
Practice 5. Solve the following system by graphing.
3x βˆ’ 2y = 6
βˆ’6x + 4y = 5
(An-
swer below)
ANSWERS TO PRACTICE PROBLEMS
1. Not a solution.
4. infinite number of solutions
2. (1, βˆ’4)
3. (βˆ’3, βˆ’3)
5. no solution
SECTION 3.1 EXERCISES
(Answers are found on page 233.)
Solve each system by graphing. Plot at least two points for each line. Remember to check your solution in both equations.
(
(
x βˆ’ 3y = 10
2x + y = 5
1.
6.
x+ y = 2
3x + 9y = 15
(
2.
3x βˆ’ y = 10
(
3.
3x βˆ’ 6y = 9
2x βˆ’ 4y = 10
(
4.
xβˆ’ y =1
x + 2y = 7
(
5.
xβˆ’y = 6
3x βˆ’ 4y = 12
x+ y = βˆ’3
(
7.
4x βˆ’ 3y = βˆ’13
(
8.
x + 4y = 1
x βˆ’ 4y = 1
(
9.
5x + 5y = 45
3x + 2y = 18
(
10.
x + 5y = βˆ’9
3x βˆ’ 5y =
4
βˆ’6x + 10y = βˆ’8
3.1. GRAPHING METHOD
11.
(
4x βˆ’ y = 6
2x + 3y = 10
(
12.
βˆ’3x + 5y = βˆ’25
(
13.
14.
15.
16.
18.
19.
x+ y = 7
βˆ’2x + 3y = 16
(
3x + 4y = 26
4x + 3y = 30
(
3x βˆ’ 2y = 12
4x + y = 5
(
2x βˆ’ y = 4
4x + y = 2
(
17.
x + 2y = 12
4x βˆ’ y = 6
βˆ’4x + y = 8
(
3x + 2y = 27
xβˆ’ y = 4
(
2x βˆ’ y = 2
4x + y = 10
65
20.
(
x+y =4
xβˆ’y =2
(
21.
22.
23.
24.
25.
3x βˆ’ 2y = 10
(
2x + y = βˆ’6
4x + 2y =
2x + y =
28.
8
(
4x + y = 3
3x + 5y = 15
(
3x βˆ’ 2y = 20
2x + 7y = 5
5x + y = 2
10x + 2y = 4
(
27.
8
(
βˆ’3x + y = βˆ’2
(
26.
x+ y = 0
5x βˆ’ 4y = 8
βˆ’5x βˆ’ 4y = 8
(
2x + y = βˆ’2
2x + 3y =
6
66
3.2
CHAPTER 3. SYSTEMS OF LINEAR EQUATIONS
Substitution Method
The graphing method for solving a system of linear equations has the advantage that it is visual and that the solution corresponds to the intersection
points. However, a major weakness is that graphs can be deceiving. Some
lines that look parallel are not, or sometimes two lines clearly intersect but
in a hard to guess location. Consider the following system
(
6x βˆ’ 3y = 5
x + 2y = 0
The graphs of these two equations are given below.
y
6
5
4
¢Μ§
¢
3
¢
2
¾
H
Y
HH
¢
1
¢
H
HH¢
0
1
2
3
¢HH
βˆ’1
H
H
j
¢
¢
βˆ’2
¢
βˆ’3
¢
βˆ’5 βˆ’4 βˆ’3 βˆ’2 βˆ’1
¢
¢
¢®
¢
4
-
5
x
βˆ’4
βˆ’5
?
Can you determine the solution of this system? The solution is
but it is probably hard to determine from the graph.
¡2
1
3, βˆ’3
¢
,
In this section we will discuss an algebraic way to solve a system of linear
equations. The method to be discussed in this section is the Substitution
Method. The Substitution Method is useful when one equation can be
solved easily for one of the variables.
3.2. SUBSTITUTION METHOD
67
Steps for the Substitution Method:
1. Choose one of the equations and solve for one variable in terms
of the other variable.
2. Substitute the expression from Step 1 into the other equation.
3. Solve the equation from Step 2. (There will be one equation
with one variable).
4. Substitute the solution from Step 3 into either of the original
equations. This will give the value of the other variable.
Remember that a system of linear equations is not completely
solved until values for both x and y are found. To avoid this
mistake, write all answers as an ordered pair.
Example 1. Solve the following system using the Substitution Method.
(
6x βˆ’ 3y = 5
x + 2y = 0
Solution 1. The second equation can be easily solved for x giving
us
x + 2y = 0
x = βˆ’2y
We now substitute this into the x variable in the first equation.
This gives us
6(βˆ’2y) βˆ’ 3y = 5
βˆ’12y βˆ’ 3y = 5
βˆ’15y = 5
y=βˆ’
1
3
68
CHAPTER 3. SYSTEMS OF LINEAR EQUATIONS
Next, we need to find the value of our x variable by substituting
this into one of the equations. Since we already know that x =
βˆ’2y, substituting into this equation gives us
µ ¶
1
x = βˆ’2 βˆ’
3
2
x=
3
¡
¢
Hence, the solution of the given system is 32 , 13 .
Practice 1. Solve the following system using the Substitution Method.
(
4x + 3y = βˆ’3
x βˆ’ 3y = βˆ’2
(Answer on page 74)
Example 2. Solve the following system using the Substitution Method.
(
3x + 2y = βˆ’8
2x + y =
5
Solution 2. Notice that the second equation can be solved easily
for y, giving us
2x + y = 5
y = βˆ’2x + 5
This is what we now substitute into the y variable in our first
equation. This gives us:
3x + 2(βˆ’2x + 5) = βˆ’8
3x βˆ’ 4x + 10 = βˆ’8
βˆ’x + 10 = βˆ’8
βˆ’x = βˆ’18
x = 18
3.2. SUBSTITUTION METHOD
69
Next, we need to find the value of our y variable by substituting
x = 18 into one of the equations. Since we already know that
y = βˆ’2x + 5, substituting in this equation gives us:
y = βˆ’2(18) + 5
y = βˆ’36 + 5
y = βˆ’31
Thus, (18, βˆ’31) is the solution of the given system.
Practice 2. Solve the following system using the Substitution Method.
(
2x + y = 4
4x + 3y = 10
(Answer on page 74)
Example 3. Solve the following system using the Substitution Method.
(
x βˆ’ y = βˆ’3
4x + 3y = βˆ’5
Solution 3. Notice that the first equation can be solved quickly
for either x or y. We will solve for x.
x βˆ’ y = βˆ’3
x=yβˆ’3
We now substitute this into the x variable in our second equation.
4(y βˆ’ 3) + 3y = βˆ’5
4y βˆ’ 12 + 3y = βˆ’5
7y βˆ’ 12 = βˆ’5
7y = 7
y=1
70
CHAPTER 3. SYSTEMS OF LINEAR EQUATIONS
We now substitute y = 1 into one of our equations in order
to find the value of x. Since we already know that x = y βˆ’ 3,
substituting y = 1 into this equation yields
x=1βˆ’3
x = βˆ’2
So, the solution of the given system is (βˆ’2, 1).
Practice 3. Solve the following system using the Substitution Method.
(
βˆ’x + y = 9
3x βˆ’ 2y = 7
(Answer on page 74)
We have seen that the substitution method can be used on those systems
where one of the linear equation’s coefficients is one. However, this is not the
only situation that the Substitution Method can be easily used. In fact, the
Substitution Method can be used to solve any system of linear equations.
Example 4. Solve the following system using the Substitution Method.
(
6x + 3y = 12
3x + 2y = 5
Solution 4. Notice that the first equation can be solved for y
without fractions, giving us
6x + 3y = 12
3y = βˆ’6x + 12
y = βˆ’2x + 4
Next, we substitute this expression into our y variable in our
second equation.
3x + 2(βˆ’2x + 4) = 5
3x βˆ’ 4x + 8 = 5
βˆ’x + 8 = 5
βˆ’x = βˆ’3
x=3
3.2. SUBSTITUTION METHOD
71
We now substitute x = 3 into one of our equations in order to
find the value of y. Since we already know that y = βˆ’2x + 4,
substituting x = 3 into this equation yields
y = βˆ’2(3) + 4
y = βˆ’6 + 4
y = βˆ’2
Therefore, (3, βˆ’2) is the solution of the given system.
Practice 4. Solve the following system using the Substitution Method.
(
5x + 10y = 20
2x + 6y = 10
(Answer on page 74)
All of our previous examples were consistent systems. A system is
consistent if there is exactly one solution. However, this is not always the
situation. Recall from Section 3.1 that two lines may not intersect or they
can represent the same line. A system is inconsistent if there is no solution.
This happens when the two equations represent parallel lines. On the other
hand, a system is dependent if there is an infinite number of ordered pairs
as solutions. This occurs when the two equations represent the same line.
Example 5. Solve the following system using the Substitution Method.
(
2x βˆ’ y = 3
βˆ’6x + 3y = 9
Solution 5. Notice that the first equation can be solved easily
for y.
2x βˆ’ y = 3
βˆ’y = βˆ’2x + 3
y = 2x βˆ’ 3
72
CHAPTER 3. SYSTEMS OF LINEAR EQUATIONS
We now substitute this into the y variable in our second equation.
βˆ’6x + 3(2x βˆ’ 3) = 9
βˆ’6x + 6x βˆ’ 9 = 9
βˆ’9 = 9
Since this is a false statement, the system is inconsistent. Therefore, there is no solution.
Example 6. Solve the following system using the Substitution Method.
(
βˆ’x + 2y = βˆ’3
3x βˆ’ 6y = 9
Solution 6. Notice the first equation can easily be solved for x.
Therefore, we have
βˆ’x + 2y = βˆ’3
βˆ’x = βˆ’2y βˆ’ 3
x = 2y + 3
Next, we substitute this expression into the x variable in the second equation.
3(2y + 3) βˆ’ 6y = 9
6y + 9 βˆ’ 6y = 9
9=9
Since we arrived at a true statement, the two equations in the
system are equivalent, or the exact same line. Hence, any solution of one of them is a solution of the other. Therefore, there
are an infinite number of solutions.
Practice 5. Solve the following system using the Substitution Method.
(
2x + 4y = 6
5x + 10y = 16
(Answer on page 74)
3.2. SUBSTITUTION METHOD
73
Practice 6. Solve the following system using the Substitution Method.
(
x βˆ’ 3y =
6
βˆ’4x + 12y = βˆ’24
(Answer on page 74)
Finally, we will consider an example with fractional coefficients.
Example 7. Solve the following system using the Substitution Method.
(
1
1
4x + 2y = 3
3x βˆ’ 5y = 14
Solution 7. For this example, it is easiest to solve for the x
variable in the first equation. Isolating the x term on one side
and multiplying by four we obtain,
1
1
x+ y =3
4
2
1
1
x=3βˆ’ y
4
2
x = 12 βˆ’ 2y
Next, we substitute this into the x variable in our second equation. As a result, we obtain
3(12 βˆ’ 2y) βˆ’ 5y = 14
36 βˆ’ 6y βˆ’ 5y = 14
36 βˆ’ 11y = 14
βˆ’11y = βˆ’22
y=2
To finish the problem, we substitute y = 2 into one of our equations to obtain the value of x. Since we know that x = 12 βˆ’ 2y,
substituting into this equation yields
x = 12 βˆ’ 2(2)
x = 12 βˆ’ 4
x=8
Hence, the solution to the given system is (8, 2).
74
CHAPTER 3. SYSTEMS OF LINEAR EQUATIONS
Practice 7. Solve the following system using the Substitution Method.
(
1
1
3x βˆ’ 6y = 4
2x βˆ’ 5y = 48
(Answer below)
ANSWERS TO PRACTICE PROBLEMS
³
1.
βˆ’1, 1
3
´
5. No solution
2. (1, 2)
6. Infinite number of solutions
3. (25, 34)
4. (2, 1)
7. (9, βˆ’6)
SECTION 3.2 EXERCISES
(Answers are found on page 233.)
Solve each system using the Substitution Method.
(
(
2x + 7y = βˆ’12
βˆ’4x + 8y = 0
1.
5.
x = 3 βˆ’ 2y
2x βˆ’ 4y = 0
(
2.
3x βˆ’ y = 7
(
3.
βˆ’2x + y = 1
x βˆ’ 3y = 5
(
4.
βˆ’2x βˆ’ 4y = 0
βˆ’2x + 4y = 7
x βˆ’ 2y = βˆ’7
6.
7.
8.
(
βˆ’x + y = 5
2x βˆ’ 2y = 9
(
βˆ’2x + y = βˆ’3
2x βˆ’ y =
(
2x + 3y = 5
x βˆ’ 4y = 6
3
3.2. SUBSTITUTION METHOD
(
9.
3x + 2y = 5
(
10.
4x βˆ’ 3y = 11
8x + 4y = 12
(
11.
4x βˆ’ 2y = 6
βˆ’6x + 3y = 5
(
12.
13.
14.
15.
3
x βˆ’ 5y = 7
2x βˆ’ y = 14
(
16.
4x + 2y =
5
2x + y = βˆ’4
(
6x + 3y = 12
9x + 6y = 15
x + 7y = 3
5
2x
(
19.
x βˆ’ 12 y =
2x
20.
1
+ 5y = βˆ’4
7x βˆ’ 3y = 17
(
21.
2x + 5y = βˆ’20
x + 52 y = βˆ’10
(
22.
3x βˆ’ 4y = βˆ’12
x+ y = βˆ’3
(
23.
x + 4y = βˆ’9
4x βˆ’ 13y = βˆ’13
(
24.
2x βˆ’ y = 6
3x + y = 6
(
25.
4x βˆ’ 2y =
8
3x + 2y = βˆ’6
(
26.
+ 6y = βˆ’4
x + 12 y = βˆ’5
(1
8x βˆ’ 3y = βˆ’4
7x βˆ’ y =
(
18.
7x βˆ’ y = 38
2x + 5y = βˆ’5
(
(
βˆ’5x + 2y = 3
4x + y = 2
(
17.
4x + y = 10
75
7x βˆ’ y = 8
x + 2y = 11
76
CHAPTER 3. SYSTEMS OF LINEAR EQUATIONS
(
27.
3x βˆ’ y = 4
x + 3y = 5
(
28.
x βˆ’ 4y = 12
3x + y = βˆ’5
3.3. ELIMINATION METHOD
3.3
77
Elimination Method
Although the substitution method can be used for solving any system of
linear equations, it works the easiest when one equation can be easily solved
for one of the variables without fractions. However, if this is impossible, the
Elimination Method can be used.
Steps for the Elimination Method:
1. Align the variables.
2. Multiply one or both equations by the appropriate constant so
that the coefficients of one of the variables are opposites of each
other.
3. Add the two equations together. (If you have done everything
correctly, this will produce one equation with at most one variable).
4. Solve the equation from Step 3.
5. Substitute the solution from Step 4 into either of the original
equations. This will give the value of the other variable.
As with the substitution method, remember that a system of
linear equations is not completely solved until values for both
x and y are found. To avoid this mistake, write all answers as
ordered pairs.
Example 1. Solve the following system using the Elimination Method.
(
6x βˆ’ 5y = 25
4x + 15y = 13
Solution 1. To eliminate the y variable, we multiply the first
equation by 3 and leave the second equation alone,
3(6x βˆ’ 5y = 25)
4x + 15y = 13
78
CHAPTER 3. SYSTEMS OF LINEAR EQUATIONS
We now have
18x βˆ’ 15y = 75
4x + 15y = 13
Adding these two equations together, and solving for x, we get
22x = 88
x=4
Now, we must find the value for y by substituting x = 4 into one
of the two original equations. Substituting into the first equation
gives
6(4) βˆ’ 5y = 25
24 βˆ’ 5y = 25
βˆ’5y = 1
y=βˆ’
1
5
Hence, the solution of the given system is
¶
µ
1
4, βˆ’ .
5
Practice 1. Solve the following system using the Elimination Method.
(
5x βˆ’ y = 13
2x + y = 8
(Answer on page 83)
Notice in the first example, we only had to multiply one equation by a
constant in order to eliminate one of the variables. However, sometimes it
is necessary to multiply both equations by a constant in order to eliminate
a variable. This is illustrated in our next example.
Example 2. Solve the following system using the Elimination Method.
(
8x + 9y = 13
6x βˆ’ 5y = 45
3.3. ELIMINATION METHOD
79
Solution 2. To eliminate the x variable we will multiply the first
equation by βˆ’3 and the second equation by 4.
βˆ’3(8x + 9y = 13)
4(6x βˆ’ 5y = 45)
This gives us
βˆ’24x βˆ’ 27y = βˆ’39
24x βˆ’ 20y = 180
Adding these two equations together and solving for y yields
βˆ’47y = 141
y = βˆ’3
Next, we need to find the value for x by substituting y = βˆ’3 into
either of the two original equations. If we substitute y = βˆ’3 into
the first equation, we get
8x + 9(βˆ’3) = 13
8x βˆ’ 27 = 13
8x = 40
x=5
Therefore, (5, βˆ’3) is the solution of the given equation.
Practice 2. Solve the following system using the Elimination Method.
(
4x βˆ’ 5y = 35
3x βˆ’ 4y = 24
(Answer on page 83)
80
CHAPTER 3. SYSTEMS OF LINEAR EQUATIONS
In Section 3.2 we discussed an inconsistent system and a dependent system. Recall that a system is inconsistent if there is no solution. This happens when the two equations represent parallel lines. On the other hand,
a system is dependent if there is an infinite number of ordered pairs as
solutions. This occurs when the two equations represent the same line.
Example 3. Solve the following system using the Elimination Method.
(
βˆ’6x + 9y = 12
2x βˆ’ 3y = βˆ’4
Solution 3. In order to eliminate the x variable, we multiply
the second equation by 3.
βˆ’6x + 9y = 12
3(2x βˆ’ 3y = βˆ’4)
This gives us
βˆ’6x + 9y = 12
6x βˆ’ 9y = βˆ’12
When we add these two equations together, we get
0=0
Since this is a true statement, we know that the system is dependent. Therefore, there are an infinite number of solutions.
Example 4. Solve the following system using the Elimination Method.
(
βˆ’5x + 4y = 1
15x βˆ’ 12y = 4
3.3. ELIMINATION METHOD
81
Solution 4. If we multiply the first equation by 3
3(βˆ’5x + 4y = 1)
15x βˆ’ 12y = 4
we will be able to eliminate the x variable. This gives us
βˆ’15x + 12y = 3
15x βˆ’ 12y = 4
Adding these two equations together results in
0=7
Since this is a false statement, we know that the system is inconsistent. Therefore, there is no solution.
Practice 3. Solve the following system using the Elimination Method.
(
3x βˆ’ 5y = 2
βˆ’6x + 10y = βˆ’4
(Answer on page 83)
Practice 4. Solve the following system using the Elimination Method.
(
4x βˆ’ 6y = 5
βˆ’2x + 3y = 7
(Answer on page 83)
Next, we turn our attention to fractional coefficients.
Example 5. Solve the following system using the Elimination Method.
(
3x + 72 y = 34
βˆ’ 12 x + 53 y = βˆ’ 45
82
CHAPTER 3. SYSTEMS OF LINEAR EQUATIONS
Solution 5. We will solve this problem by eliminating the x
variable. In order to eliminate the x variable and also eliminate
all fractions, we will multiply the first equation by 4 and the
second equation by 24.
¡
¢
4 3x + 72 y = 34
¡
¢
24 βˆ’ 21 x + 53 y = βˆ’ 45
This gives us
12x + 14y =
3
βˆ’12x + 40y = βˆ’30
Adding these two equations together gives us
54y = βˆ’27
1
y=βˆ’
2
To find the xβˆ’variable, we substitute y = βˆ’ 12 into either of the
equations. Substituting into the first equation gives us
µ ¶
7
1
3
3x +
βˆ’
=
2
2
4
7
3
3x βˆ’ =
4
4
5
3x =
2
5
x=
6
¡5 1¢
So, 6 , βˆ’ 2 is the solution of the given system.
Practice 5. Solve the following system using the Elimination Method.
(
1
1
3x + 6y = 1
1
2x
(Answer on page 83)
βˆ’ 14 y = 0
3.3. ELIMINATION METHOD
83
ANSWERS TO PRACTICE PROBLEMS
1. (3, 2)
4. no solution
2. (20, 9)
³
3. infinite number of solutions
5.
´
3,3
2
SECTION 3.3 EXERCISES
(Answers are found on page 234.)
Solve each system using the Elimination Method.
(
(
βˆ’2x + 3y = βˆ’1
9x + 4y = βˆ’3
1.
8.
x βˆ’ 2y = 3
6x + 7 = βˆ’6y
(
2.
8x + 4y = 12
(
3.
4.
5.
7x βˆ’ y =
3x + 5y =
6.
11.
6
xβˆ’y =2
4x + y = 24
x + 2y = 2
(1
2x
12.
9x + 2y = 16
13.
14.
x + 8y = βˆ’18
1
2x
(
4x βˆ’ 6y = βˆ’1
βˆ’ 5y = 6
3x βˆ’ 2y = 8
(
3x βˆ’ 4y = βˆ’2
5
2x βˆ’ y = βˆ’3
(
8
9x + 2y = 4
(
3x + 3y = βˆ’1
2x βˆ’ 2y =
10.
5x βˆ’ 3y = βˆ’7
(
9.
(
2x βˆ’ 2y = βˆ’3
(
(
2x βˆ’ 5y = 4
3x βˆ’ 2y = 4
(
7.
4x βˆ’ 3y = 11
+ 3y =
6
4x = 7y βˆ’ 6
9y + 12x = 12
84
CHAPTER 3. SYSTEMS OF LINEAR EQUATIONS
(
15.
4x + 6y βˆ’ 6 = 0
3y βˆ’ 3 = βˆ’2x
(
16.
3x + 13 y = 13
4x βˆ’
(
17.
21.
3
2y
= βˆ’6
0.2x + 0.6y =
22.
23.
18.
(
19.
1
10 x
+
1
6y
=
1
2
βˆ’
3
10 y
=
3
10
24.
(
3x + 5y = 1
2x + 3y = 0
(
2x βˆ’ 3y = 5
x + 4y = 3
(
4x βˆ’ 2y = βˆ’1
6x βˆ’ 4y = βˆ’3
2x βˆ’ y = 10
3x + 4y = βˆ’7
(
20.
1
2x
4x + 15y = 13
1
0.5x + 0.6y = βˆ’0.2
(
(
6x βˆ’ 5y = 25
(
25.
3x + 4y = βˆ’10
2x + 3y = 2
4x βˆ’ 5y = βˆ’1
2x βˆ’ 3y = βˆ’1
Solve the following systems of equations using either the Substitution Method
or the Elimination Method.
(
4x βˆ’ 5y = 35
26.
3x βˆ’ 4y = 24
(
x + 3y = βˆ’16
29.
y = 6x + 1
(
4x βˆ’ 3y = 5
27.
3x + 4y = βˆ’2
(
x = 4y βˆ’ 14
30.
(
x+y =1
3x βˆ’ 2y = 14
28.
y = 2x βˆ’ 8
3.3. ELIMINATION METHOD
(
31.
85
3x + 4y = 2
(
6x + 8y = 1
40.
(
32.
(
41.
33.
35.
36.
βˆ’5x βˆ’ 4y = 8
(
2x + y = βˆ’2
2x + 3y =
2x βˆ’ 7y =
39.
βˆ’x + 8y =
6
2x
3
44.
1
4x βˆ’ 23 y = βˆ’1
4x βˆ’ 2y = 21
3x + y = βˆ’10
47.
15
2x + 3y = βˆ’11
1
2x βˆ’ 5y
1
4 x + 3y
3
=1
=2
5x + 2y = 1
βˆ’3x + 5y = 0
(
7x βˆ’ y =
3
4x + 5y = βˆ’4
(
46.
(
βˆ’x + 8y =
4x + 6y =
(
45.
βˆ’ 13 y =
(
8x βˆ’ 2y = βˆ’3
43.
4x βˆ’ 3y = 10
(
3x + 4y = βˆ’4
(1
(
9x + 2y = βˆ’5
(5
38.
6
(
βˆ’3x + 8y = βˆ’7
37.
5
2
5x βˆ’ 4y = 8
42.
34.
βˆ’3x + y =
4x βˆ’ 3y = 5
12x βˆ’ 9y = 15
(
6x + 2y = βˆ’3
8x + 3y = 5
βˆ’2x + 3y = 6
(
3x βˆ’ 5y =
1
6x + 7y = βˆ’2
86
3.4
CHAPTER 3. SYSTEMS OF LINEAR EQUATIONS
Applications of Systems of Linear Equations
In this section we will cover a few applications of linear systems. The typical
problem will involve describing a situation with two linear equations and
then solving the system using your favorite method.
Example 1. The sum of two numbers is 53 and their difference is 15. Find
both numbers.
Solution 1. If we let x be one of the numbers and y represent
the other number, we have the following system to solve
(
x + y = 53
x βˆ’ y = 15
Notice that the system can be solved using the elimination method
without the need to multiply either equation by a constant. Thus,
adding the two equations together and solving for x, we get
2x = 68
x = 34
Substituting x = 34 into the first equation gives us
34 + y = 53
y = 19
Therefore, the two numbers are 19 and 34.
Practice 1. The sum of two numbers is 79 and their difference is 33. Find
both numbers. (Answer on page 93)
Example 2. Suzie purchased only apples and oranges at the local market.
Each apple costs 25 cents and each orange costs 30 cents. If Suzie purchased
15 pieces of fruit for $4.05, determine the number of apples and the number
of oranges she purchased.
3.4. APPLICATIONS OF SYSTEMS OF LINEAR EQUATIONS
87
Solution 2. Let
x = the number of apples purchased
and
y = the number of oranges purchased.
Since Suzie purchased a total of 15 pieces of fruit, The first equation of the system is x + y = 15. Since the apple cost is 0.25x
and the orange cost is 0.30y, the second equation of the system
is 0.25x + 0.30y = 4.05. Thus, the system to be solved is
(
x+
y = 15
0.25x + 0.30y = 4.05
Solving the first equation for x, we get x = 15βˆ’y. If we substitute
this into the second equation, we obtain
0.25(15 βˆ’ y) + 0.30y = 4.05
3.75 βˆ’ 0.25y + 0.30y = 4.05
3.75 + 0.05y = 4.05
0.05y = 0.30
y=6
Substituting y = 6 into x = 15 βˆ’ y results in x = 15 βˆ’ 6 = 9.
Therefore, Suzie purchased 9 apples and 6 oranges.
Practice 2. Peter purchased Matchbox cars and Star Wars figures at a
garage sale for a total of $5.50. Each Star Wars figure cost 50 cents and
each Matchbox car cost 25 cents. If Peter purchased a total of 17 items, how
many Star Wars figures and how many Matchbox cars did he buy? (Answer
on page 93)
Next, let us consider an example involving simple interest. Recall the
following simple interest formula from Section 2.3.
Formula for simple interest is given by
I = prt
where I is the interest, p is the principal, r is the annual interest
rate, and t is the time in years.
88
CHAPTER 3. SYSTEMS OF LINEAR EQUATIONS
The word β€œsimple” means that the interest is paid only once, so no
compounding occurs like in a savings account where interest might be paid
monthly.
Example 3. Erin took an early retirement incentive of $500, 000. Her plan
is to invest this money and live off the interest. Some of the money will
be the principal for a savings account that pays 3% simple interest. The
remainder of the money will serve as the principal for a stock fund that pays
on average 10% simple interest. If the goal is to achieve $35, 000 in interest
per year for living expenses, how much principal should be invested at 3%
and 10% respectively?
Solution 3. We know that there are two different principals
which sum to $500, 000. So if we let
x = the principal invested at 3%
and
y = the principal invested at 10%
the first equation of the system is x + y = 500, 000.
Next, we calculate the interest on each account. The interest for
one year from the principal invested at 3% would be (x)(0.03)(1)
or just 0.03x. Similarly, the interest for one year from the principal invested at 10% would be (y)(0.10)(1) or just 0.10y. Since
the interests must sum to $35, 000, the second equation of the
system is 0.03x + 0.10y = 35, 000. Therefore, we need to solve
the system
(
x+
y = 500, 000
0.03x + 0.10y = 35, 000
Since both x and y are easy to solve for in our first equation, we
will use the Substitution Method to solve this system. If we solve
for x in the first equation, we obtain
x + y = 500, 000
x = 500, 000 βˆ’ y
Substituting this into the x variable in our second equation and
3.4. APPLICATIONS OF SYSTEMS OF LINEAR EQUATIONS
89
solving for y (by rounding to two decimal places), we obtain
0.03(500, 000 βˆ’ y) + 0.10y = 35, 000
15, 000 βˆ’ 0.03y + 0.10y = 35, 000
15, 000 + 0.07y = 35, 000
0.07y = 20, 000
y = 285, 714.29
We find the value of x by substituting this value into the y variable in x = 500, 000 βˆ’ y to get x = 500, 000 βˆ’ 285714.29 =
214, 285.71. Therefore, Erin needs to invest $214, 285.71 in the
account paying 3% interest, and invest $285, 714.29 in the account paying 10% interest.
Practice 3. Suppose instead that Erin wants to invest a portion of her
$500, 000 incentive bonus in a stock account earning 6% interest and the
rest in a bond fund earning 9% interest. If her goal is to achieve $35, 000 in
interest per year for living expenses, how much principal should be invested
at 6% and 9% respectively? (Answer on page 93)
Next, we consider some examples involving distance. Here the key formula is d = r · t, where d is the distance an object travels at an average rate
r over a time period t. The units for d and t must agree with the units of r.
For example, if r is measured in miles per hour then d must be measured in
miles and t in hours. If r is given in feet per minute and t in hours then we
would need to convert t to minutes or r to feet per hour.
Example 4. Roberta and Gene recently completed a two day 1050 mile trip
to Daytona. On the first day of the trip they averaged 60 miles per hour.
On the second day of the trip, they averaged 65 miles per hour. If the trip
took a total of 17 hours to complete, determine their driving time on each
day of the trip.
Solution 4. If we let
x = the driving time for day one of the trip
and
y = the driving time for day two of the trip
90
CHAPTER 3. SYSTEMS OF LINEAR EQUATIONS
then the first equation of the system is x + y = 17 since their trip
took a total of 17 hours.
The distances from each day sum to 1050. Using d = rt, the
distance traveled on day one is 60x and the distance traveled on
day two is 65y. So, the second equation of the system would be
60x + 65y = 1050. Thus, we need to solve
(
x + y = 17
60x + 65y = 1050
Solving the first equation for y, we find that y = 17 βˆ’ x. Substituting this into the second equation, we get
60x + 65(17 βˆ’ x) = 1050
60x + 1105 βˆ’ 65x = 1050
1105 βˆ’ 5x = 1050
βˆ’5x = βˆ’55
x = 11
To solve for y, the driving time on day two, we substitute x = 11
into y = 17 βˆ’ x = 17 βˆ’ 11 = 6. Therefore, Gene and Roberta
traveled 11 hours on day one and 6 hours on day two.
Practice 4. The Miller family traveled a total of 1075 miles to see the Grand
Canyon over a two day period. They averaged 55 miles per hour on day one
and 60 miles per hour on day two. If the trip took 18 12 hours, determine
their driving time on each day of the trip. (Answer on page 93)
Example 5. Roberta and Gene are 28 miles apart and begin to bicycle towards each other at the same moment. Gene’s rate is 5 miles per hour
greater than Roberta’s rate, and after 1.5 hours they reach each other. Determine the distance each will travel before reaching the other.
Solution 5. The key is that Gene’s and Roberta’s unknown distances total 28 miles. Now even though we are looking for distances, the more crucial unknowns are the rates of Gene and
Roberta. After all, distance comes from d = rt. So, if we let
x = Roberta’s rate
and
y = Gene’s rate
3.4. APPLICATIONS OF SYSTEMS OF LINEAR EQUATIONS
91
then the first equation of the system is y = x + 5.
The second equation comes from the total distance being 28 miles.
Since the time for both Roberta and Gene is 1.5 hours, the distance traveled by Roberta is 1.5x and the distance traveled by
Gene is 1.5y. So, the second equation of the system is 1.5x +
1.5y = 28. Therefore, the system that needs solved is
(
y =x+5
1.5x + 1.5y = 28
Since the first equation is already solved for y, we use the substitution method. Thus,
1.5x + 1.5(x + 5) = 28
1.5x + 1.5x + 7.5 = 28
3x + 7.5 = 28
3x = 20.5
20.5
x=
3
41
x=
6
In the last step, the decimal was converted into a fraction to avoid
a repeating decimal answer. Now that we know that Roberta’s
rate is 41
6 miles per hour, we find the distance she traveled to be
41
3
41
1
6 × 2 = 4 = 10 4 miles. Therefore, Genes’s distance traveled
3
1
is 28 βˆ’ 10 4 = 17 4 .
Practice 5. Roberta and Gene are in cities that are 600 miles apart and
decide to leave at the same moment, traveling towards each other. Gene’s
rate is 30 mph slower than Roberta’s rate. If they reach each other after 6
hours, determine the rate of each car. (Answers on page 93)
Finally, we consider a mixture problem.
Example 6. A 40% dye solution is to mixed with a 70% dye solution to
get 120 liters of a 50% dye solution. How many liters of the 40% and 70%
solutions will be needed to produce the desired result?
92
CHAPTER 3. SYSTEMS OF LINEAR EQUATIONS
Solution 6. Let
x = the number of liters of 40% dye solution
and y = the number of liters of 70% dye solution
We will summarize the given information in the following table
where the percents are written as a decimal.
% dye solution
0.40
0.70
0.50
Liters
x
y
120
Liters of dye solution
0.40x
0.70y
0.50(120)
Since 0.50(120) = 60, this yields the following system to solve
(
x+
y = 120
0.40x + 0.70y = 60
Solving the first equation for x we obtain x = 120 βˆ’ y. If we
substitute this into the second equation, we find
0.40(120 βˆ’ y) + 0.70y = 60
48 βˆ’ 0.40y + 0.70y = 60
48 + 0.30y = 60
0.30y = 12
y = 40
Using y = 40, we find x = 120 βˆ’ y = 120 βˆ’ 40 = 80. Therefore,
40 liters of the 70% dye solution must be mixed with 80 liters of
the 40% dye solution to obtain the desired result.
Practice 6. A 30% alcohol solution is to be mixed with a 80% alcohol solution to produce 100 gallons of a 50% alcohol solution. How many gallons
of the 30% and 80% solutions must be mixed to produce the desired result?
(Answer on page 93)
3.4. APPLICATIONS OF SYSTEMS OF LINEAR EQUATIONS
93
ANSWERS TO PRACTICE PROBLEMS
1. 23 and 56
5. Gene’s rate is 35 mph;
65 mph
Roberta’s rate is
2. 5 Star Wars figures; 12 Matchbox cars
3. $166, 666.67 at 9%; $333, 333.33 at 6%
4. 7 hrs on day one; 11.5 hrs on day two
6. 60 gallons of 30%; 40 gallons of 80%
SECTION 3.4 EXERCISES
(Answers are found on page 235.)
1. A shopper pays $20.75 for four pounds of a nut mixture consisting of
almonds and pecans. If almonds cost $3.50 per pound and pecans cost
$6.00 per pound. How much of each were purchased?
2. Kent Cinema took in $1, 270 in ticket sales for a movie that 220 people
attended. If a childs ticket sold for $4.50 and an adult ticket sold for
$6.50. How many children attended the movie?
3. A recent mailing of 56 letters and postcards cost $21.72. If the letters
cost 42 cents each to mail and postcards cost 27 cents each, how many
of each were mailed?
4. There is a total of 22 coins in kitchen drawer consisting of only quarters
and nickels. The total value of the coins is $3.90. How many nickels
and how many quarters are there?
5. Three times the larger of the two numbers is 10 more than twice the
smaller. Five times the smaller is 11 less than four times the larger.
What are the numbers?
6. The tuition plus room and board comes to $20, 800 per year. The
room and board is $550 more than half the tuition. How much is the
tuition, and what does room and board cost?
7. Suppose $6, 000 is invested in two accounts. One account yields 8%
annual interest while the other account yields 7 12 % annual interest.
If the annual return on the two investments is $464, how much was
invested at each rate?
8. Suppose $9, 000 was invested one year ago. Part of the money was
invested at 6% annual interest and the rest at 10%. If the total interest
for the year was $652.80, how much was invested at each rate?
94
CHAPTER 3. SYSTEMS OF LINEAR EQUATIONS
9. Walter invested some money at 5% annual interest and twice that
amount at 6%. If his total annual interest was $127.50, how much was
invested at each rate?
10. Janice invested some of $1, 500 bonus into an account that paid 4 21 %
interest after one year. The rest of her bonus was invested in a mutual
fund that suffered a 3% loss for the same year. If the net profit from
both accounts was only $15, how much was invested in each account?
11. Suppose $10, 000 is invested in two accounts paying 5% and 4% annual
interest. The amount of interest at the end of one year is the same for
both accounts. How much was invested in each account?
12. Suppose $2, 000 is invested in two accounts paying 3% and 5% annual
interest. After 1 year the amount of interest earned in the 5% account is double the amount earned in the 3% account. How much was
invested in each account?
13. A jet takes 2 hours and 40 minutes to travel 1,120 miles with a tail
wind. The return trip against the wind takes 2 hours and 48 minutes.
What is the speed of the jet in still air?
14. A salmon travels downstream in 1 hour and 20 minutes. The return
trip takes him 4 hours against the current. If the river flows a 1.5 mph,
find the distance the salmon travels in one direction.
15. Meg traveled 7 miles yesterday walking for one half-hour and biking
for one half-hour. Today traveling at the same rates she traveled a
total of 6 miles by walking for 40 minutes and biking for 20 minutes.
What was her walking and biking rates?
16. A plane flying cross-country 2,400 miles with the wind takes 3.75 hours.
Against the wind it takes 4 hours. Determine the wind speed.
17. If Will can row a boat 1 mile upstream in 24 minutes and can row the
same distance downstream in 12 minutes find Wills rowing speed and
the speed of the current.
18. Steve invests a $20, 000 bonus, part in a stock account and the rest
in a bond fund. If the stock account grew at a yearly simple interest
rate of 12% and the bond fund grew at a yearly simple interest rate of
7% and the total interest earned in one year is $2, 000, determine the
amount invested in the stock fund.
3.4. APPLICATIONS OF SYSTEMS OF LINEAR EQUATIONS
95
19. Johnny has a piggy bank with just nickels and dimes. If he dumps his
coins into a change counter and the counter says he has a total of 300
coins with a total value of $27.30, how many nickels were in his piggy
bank?
20. The Andersons travel from their home to Disney, a total distance of
850 miles, over two days. If on the first day they averaged 55 miles per
hour and on the second day 65 miles per hour and the total driving
time was 15 hours, determine the time traveled on day one.
21. Sue began her trip to the beach in a big hurry averaging 70 miles per
hour. After a ticket for speeding, she dropped her speed to 65 miles
per hour on the average. If the total distance to the beach was 265
miles and the total driving time was 4 hours, how long did she drive
at the faster rate?
22. Roberta and Gene have the same starting point for a bike trip, but
Roberta leaves 30 minutes before Gene. If Gene’s rate is 12 miles per
hour and Roberta’s rate is 10 miles per hour, how long will it take
Gene to reach Roberta?
23. Jen and Sarah live 400 miles apart and decide to meet at a predetermined point. If they meet after 4 hours, find the speed of each car if
Jen’s rate is 20 miles per hour faster than Sarah’s rate.
24. A 90% antifreeze solution is to be mixed with a 75% antifreeze solution
to make a 78% antifreeze solution. How many gallons of each solution
must be mixed to finish with 120 gallons of the 78% solution?
25. A 40% saline solution is to be mixed with a 70% saline solution to
yield 90 liters of a 50% saline solution. How many liters of the 40%
and 70% solutions will be needed?
26. A lab technician needs 8 liters of a 55% alcohol solution. How many
liters of a 40% alcohol solution and how many liters of a 60% alcohol
solution does he need to mix?
27. Commemorative coins are to be forged from an alloy that is 40% silver.
The plant has one alloy with 50% silver content and another one with
25% silver content. How many grams of each alloy should be used to
obtain 20 grams of the alloy with 40% silver content?
96
CHAPTER 3. SYSTEMS OF LINEAR EQUATIONS
28. How much of a drink with 40% grape juice must be mixed with a 70%
grape juice drink in order to obtain 20 gallons of a 50% grape juice
drink?
29. How much of a 4% hydrochloric acid solution must be mixed with a
12% hydrochloric acid solution to obtain 12 ounces of a 9% hydrochloric acid solution?
Chapter 4
Exponents and Polynomials
4.1
Positive Integer Exponents
In Core Math I, it was discussed that exponents can be used for repeated
multiplication. (Click here to review exponents from Core Math I) For
example,
34 = 3| · 3{z
· 3 · 3}
4 factors
where 3 is the base, 4 is the exponent, and 34 is a power. The term power
is also used to refer to just the exponent itself. An algebraic expression
containing exponents is an exponential expression. When working with
exponents it is very important to determine the base.
Example 1. Determine the base of each exponential expression and evaluate.
(a) 24
(b) (βˆ’2)4
(c) βˆ’ 24
Solution 1.
(a) Here 2 is the base and 4 is the exponent. Hence,
24 = 2 · 2 · 2 · 2 = 16.
(b) Now, βˆ’2 is the base while 4 is still the exponent. Thus,
(βˆ’2)4 = βˆ’2 · βˆ’2 · βˆ’2 · βˆ’2 = 16.
97
98
CHAPTER 4. EXPONENTS AND POLYNOMIALS
(c) It is important to note that 2 is the base. The lack of parentheses shows that the exponent of 4 applies only to that base
of 2 and not βˆ’2. Therefore,
βˆ’24 = βˆ’1 · 2 · 2 · 2 · 2 = βˆ’16.
CAUTION: βˆ’an and (βˆ’a)n do not always yield the same answers. While both have n as the exponent, βˆ’an has base a;
whereas, (βˆ’a)n has base βˆ’a.
Practice 1. Evaluate each expression. (Answers on page 104 )
(a) βˆ’42
(b) (βˆ’4)2
(c) βˆ’33
(d) (βˆ’3)3
We next would like to develop the exponent rules for the integer exponents. To do this, we will begin with positive integer exponents and recall
our definition of exponents as repeated multiplication. Consider,
32 · 35 = |{z}
3 · 3 · |3 · 3 ·{z
3 · 3 · 3}
2 factors
5 factors
= 3| · 3 · 3 ·{z
3 · 3 · 3 · 3}
7 factors
7
=3
Also,
74 · 75 = 7| · 7{z
· 7 · 7} · |7 · 7 ·{z
7 · 7 · 7}
4 factors
5 factors
= 7| · 7 · 7 · 7 ·{z
7 · 7 · 7 · 7 · 7}
9 factors
= 79
We see that 32 · 35 = 32+5 = 37 and 74 · 75 = 74+5 = 79 . This suggests
that when we multiply like bases we add the exponents. This is the product
rule for exponents.
4.1. POSITIVE INTEGER EXPONENTS
99
Product Rule for Exponents: Let m and n be integers. Then for
any real number a,
am · an = am+n .
(When multiplying powers with like bases, we keep the base and
add the exponents.)
CAUTION: The bases must be the same before applying the
product rule for exponents. For example, x2 · y 4 6= (xy)6 . Remember, keep the base the same and only add the exponents.
DO NOT multiply the bases together. For example, 32 · 34 6= 96 ;
instead 32 · 34 = 32+4 = 36 .
Example 2. Use the rules of exponents to simplify each product.
(a)
54 · 59
(b)
(βˆ’2)2 · (βˆ’2)3
(c)
32 · 36 · 37
(d)
x3 · x2 · x4
Solution 2.
(a)
54 · 59 = 54+9 = 513
(b)
(βˆ’2)2 · (βˆ’2)3 = (βˆ’2)2+3 = (βˆ’2)5 = βˆ’32
(c)
32 · 36 · 37 = 32+6+7 = 315
(d)
x3 · x2 · x4 = x3+2+4 = x9
Practice 2. Use the rules of exponents to simplify each product. (Answers
on page 104 )
(a)
24 · 25
(b)
72 · 73 · 76
(c)
y5 · y9 · y2
We use the commutative and associative properties to multiply more
complicated expressions.
100
CHAPTER 4. EXPONENTS AND POLYNOMIALS
Example 3. Use the rules of exponents to simplify each product.
(a)
2x4 · 5x3
(b)
(x3 y 5 )(x5 y 8 )
(c)
(βˆ’2x4 y 7 )(3x5 y 2 )(βˆ’4xy 5 )
Solution 3.
(a)
(b)
(c)
¡
¢
2x4 · 5x3 = (2 · 5) x4 · x3 = 10x7
¡
¢¡
¢
(x3 y 5 )(x5 y 8 ) = x3 · x5 y 5 · y 8 = x8 y 13
¡
¢¡
¢
(βˆ’2x4 y 7 )(3x5 y 2 )(βˆ’4xy 5 ) = (βˆ’2 · 3 · βˆ’4) x4 x5 x y 7 y 2 y 5 = 24x10 y 14
Practice 3. Use the rules of exponents to simplify each expression. (Answers
on page 104 )
(a)
(βˆ’2x3 y 2 z 5 )(4xy 9 z 2 )
(b)
(4x4 y 5 )(3x2 y)(5x3 y 4 )
Now that we know the product rule for exponents, let us examine what
happens when we have a base to a power raised to another power. For
example, consider (34 )3 and (53 )4 . Using the definition of an exponent along
with the product rule for exponents, we get
(34 )2 = 3| 4{z
· 3}4 = 34+4 = 38
2 factors
and
(53 )4 = 5| 3 · 53{z
· 53 · 5}3 = 53+3+3+3 = 512 .
4 factors
Notice in the first example, the product of the exponents, namely 4 · 2, gives
us the exponent in the answer. In the second example, 3 · 4 gives us the exponent of 12 in the answer. This leads us to our next property of exponents.
Power Rule for Exponents: Let m and n be integers. Then for
any real number a,
(am )n = amn .
(When raising a base to a power to another power, we multiply the
exponents.)
4.1. POSITIVE INTEGER EXPONENTS
101
CAUTION: The powers are to be multiplied together. Do not
raise the power inside the parenthesis to the outside power. We
will see in the exercises that these are not the same.
Example 4. Use the rules of exponents to simplify each product.
(a)
(43 )4
(b)
(25 )4
(c)
(x5 )3 · (x4 )2
(d)
(23 )2 · (26 )3 · (25 )4
Solution 4.
(a)
(43 )4 = 43·4 = 412
(b)
(25 )4 = 25·4 = 220
(c)
(x5 )3 · (x4 )2 = x5·3 x4·2 = x15 x8 = x15+8 = x23
(d)
(23 )2 · (26 )3 · (25 )4 = 23·2 · 26·3 · 25·4 = 26 · 218 · 220
= 26+18+20 = 244
Practice 4. Use the rules of exponents to simplify each expression. (Answers
on page 104 )
(a)
¡ 2 ¢5 ¡ 6 ¢2 ¡ 3 ¢4
y
· y
· y
(b)
¡ 2 ¢3 ¡ 4 ¢2
3
· 3
If we recall our commutative and associative rules for multiplication we
can obtain two more power rules for exponents. Consider
(2x)3 = (2x)(2x)(2x)
=2·2·2·x·x·x
= 23 · x3
= 8x3
and
102
CHAPTER 4. EXPONENTS AND POLYNOMIALS
(2x4 )3 = (2x4 )(2x4 )(2x4 )
= 2 · 2 · 2 · x4 · x4 · x4
= 23 · x4+4+4
= 8x12
Notice in the second example that 23 = 8 and (x4 )3 = x12 . This leads
to our second power rule for exponents.
Product to a Power Rule for Exponents: Let n be any integer.
Then for any real numbers a and b,
(ab)n = an bn .
(When we raise a product to a power, we raise each factor to the
given power.)
CAUTION: Raising a Product to a Power Rule for Exponents
does NOT apply to a sum. For example, (3 + 4)2 = 72 = 49 but
32 + 42 = 9 + 16 = 25. Hence, (3 + 4)2 6= 32 + 42 .
Furthermore, using our properties of fractions we can obtain the third
power rule of exponents. Consider
µ ¶4
2
2·2·2·2
24
2 2 2 2
16
= · · · =
= 4 = .
3
3 3 3 3
3·3·3·3
3
84
The resulting rule is stated below.
Quotient to a Power Rule for Exponents: Let n be any integer.
Then for any real numbers a and b where b 6= 0,
³ a ´n
b
=
an
.
bn
(When we raise a quotient to a power, we raise both the numerator
and denominator to the given power.)
4.1. POSITIVE INTEGER EXPONENTS
103
Example 5. Use the rules of exponents to simplify each expression. Assume
all variables represent non-zero real numbers.
µ 2 ¶4
¡ 2 3 ¢5
x
x5
(a)
x y
(b)
y3
µ 7 ¶3 µ 2 ¶2
µ
¶
x
x
1 5 2 ¡ 6¢
2
3
(c)
(d) (2x )
x
4x
y3
y5
2
Solution 5.
¡ 2 3 ¢5
x y
= (x2 )5 (y 3 )5 = x2·5 y 3·5 = x10 y 15
µ 2 ¶4
2 4
8 5
8+5
x
x13
5 = (x ) x5 = x x = x
(b)
x
=
y3
(y 3 )4
y 12
y 12
y 12
µ 7 ¶3 µ 2 ¶2
x
x
(x7 )3 (x2 )2
x21 x4
x21+4
x25
(c)
=
·
=
·
=
=
y3
y5
(y 3 )3 (y 5 )2
y 9 y 10
y 9+10
y 19
¶
µ ¶2
µ
1
1 5 2 ¡ 6¢
3
2
3
2
3
x
4x = 2 (x )
(d)
(2x )
(x5 )2 · 4x6
2
2
1
= 8 · · 4 · x6 x10 x6 = 8x6+10+6 = 8x22
4
(a)
Practice 5. Use the rules of exponents to simplify each expression. (Answers
on page 104 )
(a)
¡ 3 2 ¢2
2x y z
³
(b)
(c)
y2 z
x4
´3 ³
z2 y
x3
´4
¡ 3 ¢3 ¡ 1 4 ¢2 ¡
¢
3x
βˆ’3x2
3x
Now we turn our attention to the meaning of a0 for a 6= 0. It is important
that we define a0 to be sure that it still satisfies all of the integer properties
we have discussed so far. For example, we still want 20 ·24 = 20+4 = 24 . Since
24 6= 0, 20 must be 1. Note that it is important here that 24 6= 0 because
otherwise 20 could have been any number. The same reasoning would apply
to any nonzero base so we are led to make the following definition.
104
CHAPTER 4. EXPONENTS AND POLYNOMIALS
Zero Exponent: For any nonzero real number a,
a0 = 1.
Therefore, 80 = 1, (βˆ’3)0 = 1, and (2x2 y 3 )0 = 1 for nonzero variables x
and y.
We will discuss the negative integer exponents in Section 4.3.
ANSWERS TO PRACTICE PROBLEMS
1.
(b) 711
(a) βˆ’16
(c) y
(b) 16
(c) βˆ’27
3.
(d) βˆ’27
2.
(a) 29
4.
(b) 314
16
5.
(a) βˆ’8x4 y 11 z 7
(a) 4x6 y 4 z 2
y 10 z 11
x24
(b) 60x9 y 10
(b)
(a) y 34
(c) βˆ’9x19
SECTION 4.1 EXERCISES
(Answers are found on page 236.)
Using exponents, rewrite the following expressions in a simpler form.
1.
3·3·3·3·3·3·3
3.
3·2·3·2·6
2.
2·5·5·2·2·5
4.
1 1 1 1 1 1
· · · · ·
2 2 2 2 2 2
Evaluate each expression.
11.
µ ¶3
1
2
12.
(βˆ’1)34
13.
(1.5)2
14.
(βˆ’3)4
βˆ’26
5.
βˆ’240
6.
(βˆ’24)0
7.
(βˆ’2)5
8.
63
9.
10.
17.
βˆ’44
18.
(βˆ’138)0
19.
βˆ’(βˆ’1)60
βˆ’72
20.
βˆ’80
15.
013
21.
βˆ’(βˆ’5)2
16.
(βˆ’2)4
22.
(0.001)2
4.1. POSITIVE INTEGER EXPONENTS
105
Use the laws of exponents to calculate the following expressions mentally.
µ ¶4
1
·
8
23.
244
24.
207 (0.5)7
25.
26.
µ ¶8
µ ¶3
1
2
11
·9 ·
3
6
µ ¶3
µ ¶3
1
1
3
3
20 ·
·2 ·
5
4
Simplify the following, leaving your answer as a single base raised to a single
exponent. Assume all variables represent non-zero real numbers.
27.
x7 · x3 · x4
31.
92 · 123 · 2
28.
128 · (24 )3
32.
167 · 48 · 83
29.
23 · 45 · 83
33.
1252 · 94 · 272 · 254
30.
(52 )4 · 53 · 52
34.
276 · 38 · 94 · 81
Use a combination of the rules of exponents to simplify each expression.
Assume all variables represent nonzero real numbers.
35.
2x2 (2x)2
41.
¡
¢3
βˆ’3xy 3 z 2
36.
(7x5 )2
42.
( 12 x4 )(16x5 )
37.
(4x2 )3
43.
(2x3 )(βˆ’3x5 )(4x7 )
38.
(3x4 y 5 )3
44.
(βˆ’4b3 )( 61 b2 )(βˆ’9b4 )
39.
(βˆ’6x3 y 9 )2
45.
(x2 y)(x5 y 3 )
40.
(2x2 y 5 z 9 )4
46.
(2x2 y)
¡1
3 5
4x y
¢ ¡1
3x
4 y2
¢
106
CHAPTER 4. EXPONENTS AND POLYNOMIALS
µ
¶
1 5 ¡ 3 2 ¢2
x y 8x y
16
47.
¡ 2 ¢2 ¡ 4 5 ¢
xy
x y
54.
¡ 3 ¢3
2x
48.
¡ 3 4 ¢2 ¡ 2 5 ¢3
x y
2x y
55.
¡ 4 3 ¢2 ¡
¢
3x y
7xy 5
49.
(3x5 y 2 )(βˆ’2x3 y 4 )2
µ
50.
µ
51.
µ
52.
µ
53.
βˆ’3x4
2y 5
5y 4 z
2x3
x4
9y 3
x3
2y
µ
56.
¶2
57.
¶3
¶µ
¶3 µ
4x
y2
¶µ
3x
y2
¶2
¡ 4 5 ¢3
2x y
58.
1 3
xy
4
59.
¡ 2 3 ¢2
3x y
60.
¡
¢
49x6 y 9 z
¶
¶
¶2
¡
¢2 ¡
¢3
βˆ’3x4 y
βˆ’2x2 y 5
µ
3x6
y4
4x2
y
µ
1 4 5
x y
9
µ
¶µ
βˆ’xy 2 z
7
1 3
xy
4
¶
¶2
Using the properties of exponents, determine which is larger. State why.
61.
428
62.
39 + 39 + 39
63.
279
or
914
64.
618
or
336
or
818
96
or
2
65. Is it true that (34 )2 = 3(4 ) ? Explain why or why not.
66. Is (3 + 4)3 = 33 + 43 ? Are there any combinations of values of a and b
for which (a + b)3 = a3 + b3 ? Are there any positive integer values of
n for which (a + b)n = an + bn ?
4.2. POLYNOMIALS
4.2
107
Polynomials
A polynomial in x is any expression which can be written as
an xn + anβˆ’1 xnβˆ’1 + · · · + a1 x + a0
where an , anβˆ’1 , . . . , a1 , a0 are any numbers and each exponent is a whole
number. In other words, a polynomial in x is a sum of a fixed number of
terms of the form axn where a can be any number and n is a whole number.
The degree of a polynomial is the highest exponent of the polynomial.
Examples of polynomials in one variable are:
3x2 βˆ’ 5x + 1
7x5 βˆ’ 8x3 + 2x2 βˆ’ 9
βˆ’ 9y 5 βˆ’ 3
Furthermore,
3x2 βˆ’ 5x + 1 has degree 2
7x9 βˆ’ 8x3 + 2x2 βˆ’ 4 has degree 9
and
βˆ’ 9y 5 βˆ’ 3 has degree 5
Polynomials can also have more than one variable. A polynomial with
more than one variable is the sum of terms in which all variables have whole
number exponents. The degree of a term of a polynomial, is the sum of
the exponents on the variables. Therefore, the degree of a polynomial with
more than one variable is the highest degree of any term of the polynomial.
Examples of polynomials with more than one variable are:
2x2 y βˆ’ 3xy + 5
4x3 y 2 βˆ’ 3x2 y 4
5x βˆ’ 7xy + 2y.
Also, we see that
2x2 y βˆ’ 3xy + 5 has degree 3
4x3 y 2 βˆ’ 3x2 y 4 has degree 6
and
5x βˆ’ 7xy + 2y has degree 2
Polynomials can be classified according to the number of terms they
have. For example, a polynomial with only one term is called a monomial.
A polynomial with exactly two terms is called a binomial, and a polynomial
with three terms is called a trinomial.
108
CHAPTER 4. EXPONENTS AND POLYNOMIALS
Example 1. Determine the degree of each polynomial.
(a) 8x4 βˆ’ 3x3 + 5x2 βˆ’ 1
(b) 7x3 y 4 + 3x2 y 3 βˆ’ 6x5 y
(c) 9x4 y 2 βˆ’ 5x9 y
Solution 1. (a) The degree of 8x4 βˆ’ 3x3 + 5x2 βˆ’ 1 is 4 since
this is the largest exponent.
(b) The degree of 7x3 y 4 + 3x2 y 3 βˆ’ 6x5 y is the same as the highest degree of any term in the polynomial. The term 7x3 y 4
has degree 7, 3x2 y 3 has degree 5 and 6x5 y has degree 6.
Therefore, the degree of the polynomial is 7.
(c) The degree of 9x4 y 2 βˆ’ 5x9 y is also the same as the highest
degree of any term in the polynomial. The term 9x4 y 2 has
degree 6 and 5x9 y has degree 10. Thus, the degree of the
polynomial is 10.
Practice 1. Determine the degree of each polynomial. (Answer on page 114.)
(a) 10x6 + 3x4 βˆ’ 4x3 + 7
(b) 3a4 b2 βˆ’ 2a5 b3 + 6ab4
A polynomial can assume different values depending on the value of the
variable. We will use the function notation that was discussed in Section 1.1.
Example 2. Let P (x) = 4x2 βˆ’ 2x + 1. Find
(a) P (3)
(b) P (βˆ’2)
Solution 2. (a) To evaluate this polynomial when x = 3, we
replace every variable x with the value 3. Recalling the order
of operations, we obtain
P (3) = 4(3)2 βˆ’ 2(3) + 1 = 4(9) βˆ’ 6 + 1 = 36 βˆ’ 6 + 1 = 31
as the value of the polynomial when x = 3.
4.2. POLYNOMIALS
109
(b) To evaluate this polynomial when x = βˆ’2, we replace every
variable x with the value βˆ’2. For this value, we obtain
P (βˆ’2) = 4(βˆ’2)2 βˆ’2(βˆ’2)+1 = 4(4)+4+1 = 16+4+1 = 21
Practice 2. Let P (x) = 3x2 +5xβˆ’6. Find (a) P (2)
(b) P (βˆ’1) (Answers
on page 114.)
In order to add or subtract polynomials, we combine like terms – terms
with the same variable and same exponent. When we combine, we keep
the variable the same and add (or subtract) the coefficients. For example,
3x2 and 8x2 are like terms (same variable and same exponent). As a result,
3x2 + 8x2 = 11x2 and 3x2 βˆ’ 8x2 = βˆ’5x2 . On the other hand, 3x4 and 8x2
are unlike terms (same variable but different exponents). Hence, these two
terms cannot be combined. To add or subtract two or more polynomials
with more than one term, we combine all like terms.
¡ 2
¢ ¡
¢
Example 3. Simplify
4x βˆ’ 3x + 2 + 6x2 + 7x βˆ’ 5
Solution 3.
¡ 2
¢ ¡
¢
4x βˆ’ 3x + 2 + 6x2 + 7x βˆ’ 5 = (4x2 + 6x2 ) + (βˆ’3x + 7x) + (2 βˆ’ 5)
= 10x2 + 4x βˆ’ 3
¡
¢ ¡
¢
Practice 3. Simplify 3x2 βˆ’ 5x βˆ’ 7 + 8x2 βˆ’ 9x + 4 (Answer on page 114.)
To subtract polynomials, first distribute the negative and then combine like
terms.
¡ 2
¢ ¡
¢
Example 4. Simplify
5x βˆ’ 3x + 2 βˆ’ 2x2 βˆ’ 7x βˆ’ 3
Solution 4.
¡ 2
¢ ¡
¢
5x βˆ’ 3x + 2 βˆ’ 2x2 βˆ’ 7x βˆ’ 3 = 5x2 βˆ’ 3x + 2 βˆ’ 2x2 + 7x + 3
= (5x2 βˆ’ 2x2 ) + (βˆ’3x + 7x) + (2 + 3)
= 3x2 + 4x + 5
110
CHAPTER 4. EXPONENTS AND POLYNOMIALS
¡
¢ ¡
¢
Practice 4. Simplify 3x2 + 9x βˆ’ 6 βˆ’ 7x2 + 4x βˆ’ 3 (Answer on page 114.)
Example 5. Simplify
¡ 2
¢ ¡
¢ ¡
¢
3x βˆ’ 9x + 7 βˆ’ 4x3 + 2x2 + 4 + 9x3 + 7x + 1
Solution 5.
¡ 2
¢ ¡
¢ ¡
¢
3x βˆ’ 9x + 7 βˆ’ 4x3 + 2x2 + 4 + 9x3 + 7x + 1
= 3x2 βˆ’ 9x + 7 βˆ’ 4x3 βˆ’ 2x2 βˆ’ 4 + 9x3 + 7x + 1
= (βˆ’4x3 + 9x3 ) + (3x2 βˆ’ 2x2 ) + (βˆ’9x + 7x) + (7 βˆ’ 4 + 1)
= 5x3 + x2 βˆ’ 2x + 4
¡
¢ ¡
¢ ¡
¢
Practice 5. Simplify 5x3 βˆ’ 7x2 + 8x βˆ’ 3 + 4x3 βˆ’ 5x + 7 βˆ’ 9x2 + 6x βˆ’ 11
(Answer on page 114.)
Example 6. Simplify
¡ 2
¢ ¡
¢ ¡
¢
7x y + 3xy 2 βˆ’ 6xy + 4xy βˆ’ 3x2 y + 9xy 2 βˆ’ 2x2 y βˆ’ 5xy
Solution 6.
¡ 2
¢ ¡
¢ ¡
¢
7x y + 3xy 2 βˆ’ 6xy + 4xy βˆ’ 3x2 y + 9xy 2 βˆ’ 2x2 y βˆ’ 5xy
= 7x2 y + 3xy 2 βˆ’ 6xy + 4xy βˆ’ 3x2 y + 9xy 2 βˆ’ 2x2 y + 5xy
= 2x2 y + 12xy 2 + 3xy
Practice 6. Simplify
¡ 2 3
¢ ¡
¢ ¡
¢
4x y + 5x3 y 2 βˆ’ 8xy 2 βˆ’ 6xy 2 βˆ’ 3x2 y 3 βˆ’ 3x3 y 2 βˆ’ 7x2 y 3 + 3x3 y 2
(Answer on page 114.)
4.2. POLYNOMIALS
111
To multiply polynomials we multiply each term in the first polynomial
by each term in the second polynomial. In order to do this, we need to
recall the distributive property and the product rule for exponents: For any
integers m and n, am · an = am+n .
¡
¢
Example 7. Simplify 4x 3x2 + 2
Solution 7. We need to distribute 4x to every term in our parenthesis. Hence,
¡
¢
4x 3x2 + 2 = (4x)(3x2 ) + (4x)(2)
= 12x3 + 8x
Practice 7. Simplify
¡
¢
βˆ’2x2 3x2 + 4x βˆ’ 2 (Answer on page 114.)
Example 8. Simplify
(4x βˆ’ 3) (5x + 2)
Solution 8.
(4x βˆ’ 3) (5x + 2) = 4x(5x + 2) βˆ’ 3(5x + 2)
= (4x)(5x) + (4x)(2) βˆ’ (3)(5x) βˆ’ 3(2)
= 20x2 + 8x βˆ’ 15x βˆ’ 6
= 20x2 βˆ’ 7x βˆ’ 6
Be careful when combining like terms. Remember that the variable part
remains unchanged.
Practice 8. Simplify (7x βˆ’ 3)(5x + 4) (Answer on page 114.)
There is also a rectangular array approach to multiplication of polynomials. To set up a rectangular array, consider (3x + 2)(4x βˆ’ 5). The terms
of the first polynomial become the labels on each of the rows; whereas, the
terms of the second polynomial become the labels on each of the columns,
as shown in the next diagram.
112
CHAPTER 4. EXPONENTS AND POLYNOMIALS
4x
βˆ’5
3x
2
To finish the problem and find the solution, we then multiply the term at the
beginning of the row with the term at the top of the corresponding column.
We then record the result in the appropriate cell of the table. This would
look like
3x
2
4x
12x2
8x
βˆ’5
βˆ’15x
βˆ’10
Now to find the solution, we combine like terms of the cells inside the table
to get 12x2 βˆ’ 7x βˆ’ 10. The benefits of the rectangular array is that it insures
that you multiply every term in the first polynomial with every term in the
second polynomial.
Example 9. Use the rectangular array approach to multiply
(3x2 + 2x βˆ’ 4)(2x2 βˆ’ 5x + 3).
Solution 9. The set-up for the rectangular array approach is the
following:
2x2
βˆ’5x
3
3x2
2x
βˆ’4
Therefore, once the multiplication has been performed and recorded
we have
3x2
2x
βˆ’4
2x2
6x4
4x3
βˆ’8x2
βˆ’5x
βˆ’15x3
βˆ’10x2
20x
3
9x2
6x
βˆ’12
To finish, we combine like terms to get a solution of
6x4 βˆ’ 11x3 βˆ’ 9x2 + 26x βˆ’ 12.
4.2. POLYNOMIALS
113
Practice 9. Use the rectangular array approach to multiply
¡ 3
¢¡
¢
3x βˆ’ 4x + 2 4x2 βˆ’ 5x βˆ’ 3 .
(Answer on page 114.)
The rectangular array approach is quite beneficial when the problem is
fairly long and it is more likely that a mistake in the distributive property
could arise. Once again, it guarantees that each term of the first polynomial
will be multiplied by each term of the second polynomial.
Recall from Section 4.1 that it was determined that (a + b)n 6= an + bn
for all integers n β‰₯ 2. So, how do we simplify (3x βˆ’ 2)2 ? Recall, that
the exponent tells us how many times the multiplication is to be repeated.
Hence, using our definition of exponents we must rewrite the expression
before multiplying it out.
Example 10. Multiply (3x βˆ’ 2)2
Solution 10.
(3x βˆ’ 2)2 = (3x βˆ’ 2) (3x βˆ’ 2)
= 9x2 βˆ’ 6x βˆ’ 6x + 4
= 9x2 βˆ’ 12x + 4
Practice 10. Multiply (6x + 5)2 (Answer on page 114.)
This same procedure can be used for any integer exponent value.
Example 11. Multiply (3x βˆ’ 2)3
Solution 11. From Example 10, we found that by multiplying the first two quantities together we have (3x βˆ’ 2) (3x βˆ’ 2) =
9x2 βˆ’ 12x + 4. Hence,
(3x βˆ’ 2)3 = (3x βˆ’ 2) (3x βˆ’ 2) (3x βˆ’ 2)
= (9x2 βˆ’ 12x + 4)(3x βˆ’ 2)
= 27x3 βˆ’ 18x2 βˆ’ 36x2 + 24x + 12x βˆ’ 8
= 27x3 βˆ’ 54x2 + 36x βˆ’ 8
Practice 11. Multiply (6x + 5)3 (Answer on the next page.)
114
CHAPTER 4. EXPONENTS AND POLYNOMIALS
ANSWERS TO PRACTICE PROBLEMS
1.
6. 2x2 y 3 + 5x3 y 2 βˆ’ 14xy 2
(a) 6
(b) 8
2.
7. βˆ’6x4 βˆ’ 8x3 + 4x2
(a) 16
8. 35x2 + 13x βˆ’ 12
(b) βˆ’8
9. 12x5 βˆ’ 15x4 βˆ’ 25x3 + 28x2 + 2x βˆ’ 6
3. 11x2 βˆ’ 14x βˆ’ 3
4. βˆ’4x2 + 5x βˆ’ 3
10. 36x2 + 60x + 25
5. 9x3 βˆ’ 16x2 βˆ’ 3x + 15
11. 216x3 + 540x2 + 450x + 125
SECTION 4.2 EXERCISES
(Answers are found on page 237.)
1. Is it possible for the sum of two polynomials, both of degree three, to
be a polynomial of degree two? If so, give an example. If not, explain
why.
2. If we multiply a monomial (a polynomial with one term) by another
monomial will we always get a monomial?
3. If we multiply a monomial by a binomial (a polynomial with two terms)
will we ever get a trinomial (a polynomial with three terms)?
4. If we consider the product of a polynomial of degree two and a polynomial of degree three, what is the degree of the product? In general,
if an n-th degree polynomial and an m-th degree polynomial are multiplied together, what is the degree of the product?
Determine the degree of each polynomial.
5. P (x) = 6x8 βˆ’ 3x5 + 2x + 9
6. P (x) = 4x7 βˆ’ 3x6 + 2x3 + 5x2
7. P (x, y) = 2xy 3 βˆ’ 3x2 y 4 + 5xy
8. P (a, b) = 3ab2 βˆ’ 2ab + 7a2 b
9. P (x, y) = 3x3 y 2 + 5xy 2 + 7x + 10
4.2. POLYNOMIALS
115
10. P (a, b) = 9a + 3a2 b βˆ’ 6ab2 + 4
Evaluate each polynomial.
11. If P (x) = 4x + 7, find P (βˆ’8).
¡ ¢
12. If P (x) = 3x βˆ’ 4, find P βˆ’ 52 .
13. If P (x) = 4x2 βˆ’ 6x + 8, find P
¡1¢
2 .
14. If P (x) = 5x2 + 3x + 9, find P (2).
15. If P (x) = x3 βˆ’ 4x2 βˆ’ 2x + 5, find P (βˆ’2).
16. If P (x) = x2 + x βˆ’ 6, find P (βˆ’1).
17. If P (x) = x2 βˆ’ x βˆ’ 12, find P (βˆ’3).
18. If P (x) = 11x2 βˆ’ 3x βˆ’ 11, find P (2).
19. If P (x) = βˆ’2x2 βˆ’ 8x + 6, find P (βˆ’3).
20. If P (x) = x3 βˆ’ 10x2 + 22x βˆ’ 5, find P (βˆ’2).
21. If P (x) = 7x3 βˆ’ 11x2 + 6x βˆ’ 9, find P (βˆ’1).
22. If P (x) = βˆ’8x3 + 6x2 βˆ’ 10x, find P (βˆ’5).
23. If P (x) = 15x4 βˆ’ 6x3 βˆ’ 21x2 , find P (2).
Perform the indicated operations and simplify.
24.
(3x + 7) + (βˆ’4x + 9)
25.
(2x + 7) βˆ’ (βˆ’3x + 9)
26.
(8x3 + 12x2 ) βˆ’ (8x3 + 8x2 )
27.
(7a2 + 8a βˆ’ 9) + (βˆ’8a2 βˆ’ 9a + 8)
¡ 2
¢ ¡
¢
5x βˆ’ 7x βˆ’ 2 + 6x2 + 4x βˆ’ 9
¡ 2
¢ ¡
¢
6x βˆ’ 3x + 4 βˆ’ 8x2 + 5x βˆ’ 2
¡ 3
¢ ¡
¢ ¡
¢
3x βˆ’ 5x2 + 6x βˆ’ 7 βˆ’ 4x3 βˆ’ 7x + 1 + 2x3 βˆ’ 5x2 + 9x + 3
28.
29.
30.
116
CHAPTER 4. EXPONENTS AND POLYNOMIALS
31.
¡ 3
¢ ¡
¢ ¡
¢
5x βˆ’ 9x2 + 2x βˆ’ 3 βˆ’ βˆ’6x3 + 2x2 βˆ’ 8x βˆ’ 1 βˆ’ 4x3 + 4x + 7
32.
(5a3 b2 + 2a2 b2 βˆ’ 3ab2 ) + (a3 b2 βˆ’ a2 b2 + ab2 )
33.
(17mn2 βˆ’ 6m2 n) + (2mn2 + 10) βˆ’ (10m2 n + 15)
34.
4x (3x βˆ’ 2)
35.
38.
5x2 (6x + 3)
¡
¢
βˆ’2x 4x2 βˆ’ 3x + 5
¡
¢
3x2 5x2 βˆ’ 2x βˆ’ 7
¡
¢
βˆ’3x2 y 5xy 2 βˆ’ 7xy + 3x2
39.
βˆ’2x2 y 4 (5x2 y 2 βˆ’ 3xy 3 + 2x3 y 4 βˆ’ 7y)
40.
(2x βˆ’ 3)(2x + 5)
41.
(3x βˆ’ 2)(5x βˆ’ 4)
42.
(4x + 3)(7x βˆ’ 2)
43.
(5x βˆ’ 3) (2x + 1)
44.
(6x βˆ’ 1) (4x + 2)
45.
(2x βˆ’ 3) (2x + 3)
46.
(4x + 1) (4x βˆ’ 1)
47.
(5x + 2) (5x βˆ’ 2)
48.
(3x βˆ’ 4) (3x + 4)
¶µ
µ
3
xβˆ’5
3x βˆ’
2
µ
¶µ
2
3x βˆ’
2x +
3
36.
37.
49.
50.
51.
(7x βˆ’ 2)2
52.
(4x + 3)2
53.
(2x βˆ’ 3)2
2
5
1
4
¶
¶
4.2. POLYNOMIALS
54.
(x βˆ’ 2)3
55.
57.
(2x + 1)3
¡
¢
(4x + 1) 3x2 βˆ’ 5x βˆ’ 2
¡
¢
(x βˆ’ 4) 2x2 βˆ’ 3x βˆ’ 2
58.
x (3x βˆ’ 4) (7x + 3)
59.
2x2 (4x βˆ’ 3) (2x + 5)
60.
2x(3x βˆ’ 4)(2x + 3)
¡
¢
(5x βˆ’ 2) 2x2 βˆ’ 3x + 4
¡
¢
(3x βˆ’ 5) 2x3 βˆ’ 4x2 + 7x βˆ’ 2
56.
61.
62.
117
Use a rectangular array to multiply.
65.
(3x βˆ’ 5)(2x3 βˆ’ 6x2 βˆ’ 3x + 2)
¡ 2
¢¡
¢
x βˆ’ 3x βˆ’ 1 2x2 + 5x βˆ’ 7
¡ 2
¢¡
¢
6x βˆ’ x + 3 3x2 βˆ’ 2x βˆ’ 4
66.
Give a counterexample to show that x + 2(x βˆ’ 3) 6= (x + 2)(x βˆ’ 3)
63.
64.
118
4.3
CHAPTER 4. EXPONENTS AND POLYNOMIALS
Negative Integer Exponents
In Section 4.1, we discussed how to work with zero and positive integer exponents. So, what about negative exponents? Our old definition of viewing
exponents as repeated multiplication no longer holds for negative exponents.
For example, we cannot evaluate 3βˆ’2 by taking the base 3 and multiplying
it to itself βˆ’2 times. This makes no sense. However, we need to be sure
that we define aβˆ’n so that it still obeys all of the exponent rules we have
discussed thus far. If a 6= 0,
an · aβˆ’n = an+(βˆ’n) = a0 = 1.
Thus, aβˆ’n must be the reciprocal of an .
Negative Exponent Rule: For any nonzero real number
a,
1
aβˆ’n = n .
a
CAUTION: A positive constant raised to a negative power does
NOT yield a negative number. For example, 3βˆ’2 6= βˆ’6, instead,
3βˆ’2 =
1
1
= .
2
3
9
Furthermore, the above rule does not apply to negative constants. For example, βˆ’4 6= 14 . The rule only applies to negative
exponents.
We leave it as an exercise for the reader to verify that the product rule
and power rules for exponents remain true with negative exponents. Thus,
all the rules we discussed in Section 4.1 still hold for any integer exponent
whether it is positive, negative, or zero.
Example 1. Use the rules of exponents to simplify each expression. Write
answers with positive exponents only.
(a)
3βˆ’1 + 3βˆ’2
(b)
5βˆ’1 βˆ’ 50
4.3. NEGATIVE INTEGER EXPONENTS
(c)
(d)
(e)
(f )
119
¡
¢0
(2xy)2 3xy 7
¡ βˆ’4 3 ¢βˆ’2
x y
¡
¢βˆ’2
βˆ’2x4 y 3
µ
¶
¡ 4 βˆ’3 ¢ 1 βˆ’5 7
x y
8x y
2
Solution 1.
(a)
(b)
(c)
(d)
(e)
(f )
1
1
1 1
3 1
4
+ 2 = + = + =
3 3
3 9
9 9
9
1
1
5
4
5βˆ’1 βˆ’ 50 = βˆ’ 1 = βˆ’ = βˆ’
5
5 5
5
¡
¢0
(2xy)2 3xy 7 = 22 x2 y 2 · 1 = 4x2 y 2
¡ βˆ’4 3 ¢βˆ’2
1
x8
x y
= xβˆ’4·βˆ’2 · y 3·βˆ’2 = x8 y βˆ’6 = x8 6 = 6
y
y
¡
¢
1
1
1
βˆ’2
βˆ’2x4 y 3
=
=
= 8 6
4
3
2
2
4·2
3·2
(βˆ’2x y )
(βˆ’2) x y
4x y
µ
¶
¡ 4 βˆ’3 ¢ 1 βˆ’5 7
1 4 βˆ’5 βˆ’3 7
8x y
x y = 8· ·x x y y = 4x4+(βˆ’5) y (βˆ’3)+7
2
2
4
1
4y
= 4xβˆ’1 y 4 = 4 y 4 =
x
x
3βˆ’1 + 3βˆ’2 =
Practice 1. Use the rules of exponents to simplify each expression. Write
answers with positive exponents only. (Answers on page 123)
¡ βˆ’2 3 2 ¢βˆ’2
r s t
(a)
¡
¢2 ¡
¢
(b)
βˆ’2x3 y βˆ’2 z βˆ’1
βˆ’4xβˆ’2 yz 3
(c)
4βˆ’3 + 4βˆ’2
Consider the example
2βˆ’4
=
3βˆ’5
2βˆ’4
. Using our negative exponent rule, we obtain
3βˆ’5
1
24
1
35
=
1
1
1 35
35
÷
=
·
=
.
24 35
24 1
24
120
CHAPTER 4. EXPONENTS AND POLYNOMIALS
Therefore, the negative exponent rule allows us to move factors in a fraction
µ ¶βˆ’3
2
5
just by changing the sign of the exponent. Also, consider
. Since
5
2
2
is the reciprocal of ,
5
à !3 µ ¶
µ ¶βˆ’3
2
5 3
1
13
1
=
= ¡ ¢3 = ¡ ¢3 =
.
2
2
2
5
2
5
5
5
Thus, if we have a fraction raised to a power, we can change the sign of
the exponent by using the reciprocal of the fraction. We summarize these
rules as follows:
Converting from Negative to Positive Exponents: For any
nonzero numbers a and b, and any integers m and n,
³ a ´βˆ’n µ b ¶n
aβˆ’m
bn
= m
and
=
.
bβˆ’n
a
b
a
Before examining some exercises we need to discuss our final exponent
26
22
rule – the quotient rule. Consider, 2 and 6 . Rewriting both expressions
2
2
using the definition of an exponent, we obtain
2 · 2 · 2 · 2· 6 2· 6 2
26
=
= 2 · 2 · 2 · 2 = 24 ,
2
2
6 2· 6 2
and
22
1
1
6 2· 6 2
=
= 4 = 2βˆ’4 .
=
6
2
6 2· 6 2 · 2 · 2 · 2 · 2
2·2·2·2
2
For the first example, subtracting the exponents we have 6 βˆ’ 2 = 4 which is
the resulting exponent. In the second example, subtracting the exponents,
we have 2 βˆ’ 6 = βˆ’4 which again is the resulting exponent. These examples
suggest the quotient rule for exponents.
Quotient Rule for Exponents: For any nonzero real number a
and any integers m and n,
am
= amβˆ’n .
an
(When dividing powers with like bases, we keep the base and subtract
the exponents.)
4.3. NEGATIVE INTEGER EXPONENTS
121
CAUTION: When using the quotient rule, the bases must be the
same. Remember to keep the base the same and only subtract
the exponents. The order of the subtraction is important since
subtraction does not satisfy the commutative property. Therefore, when applying the quotient rule for exponents, remember
that it is
(exponent in the numerator) βˆ’ (exponent in the denominator).
When working with the rules of exponents, note that the steps can be
performed in many different orders. As such, the examples below show one
method of simplifying each expression. However, there are many equally
correct ways to begin your solution. Regardless of what order you perform
the steps, you will always get the same answer.
Example 2. Use a combination of the exponent rules to simplify each exponential expression. Write all answers with only positive exponents. Assume
all variables represent nonzero real numbers.
(a)
(2x2 )3
4x4
(b)
(6x3 )2
· (3x2 )0
(2x2 )3
¡ 2 βˆ’3 ¢4
xy w
(c)
(xβˆ’3 y βˆ’2 w)3
µ
(d)
(e)
¶3
βˆ’3a3 bβˆ’5
6aβˆ’2 bβˆ’2
¡ 2 βˆ’3 ¢βˆ’2
2m n
(βˆ’3mβˆ’3 n4 )βˆ’1
Solution 2.
(a)
(2x2 )3
23 x2·3
8x6
=
=
= 2x6βˆ’4 = 2x2
4x4
4x4
4x4
122
CHAPTER 4. EXPONENTS AND POLYNOMIALS
2 3·2
36x6
36
9
(6x3 )2
2 )0 = 6 x
·
(3x
·
1
=
=
=
2
3
3
2·3
6
(2x )
2 x
8x
8
2
¡ 2 βˆ’3 ¢4
xy w
x4 y 2·4 wβˆ’3·4
x4 y 8 wβˆ’12
x4 x9 y 8 y 6
(c)
=
=
=
xβˆ’3·3 y βˆ’2·3 w3
xβˆ’9 y βˆ’6 w3
w12 w3
(xβˆ’3 y βˆ’2 w)3
x4+9 y 6+8
x13 y 14
=
=
12+3
w
w15
(b)
¶3 µ
¶3 µ
¶3 µ 5 βˆ’3 ¶3
βˆ’3a3 a2 b2
βˆ’3a3+2 b2βˆ’5
βˆ’3a3 bβˆ’5
βˆ’a b
=
=
(d)
=
βˆ’2
βˆ’2
5
6a b
6b
6
2
µ 5 ¶3
5·3
15
βˆ’a
βˆ’a
βˆ’a
=
= 3 3·3 =
3
2b
2 b
8b9
¡ 2 βˆ’3 ¢βˆ’2
¡
¢
2m n
βˆ’3mβˆ’3 n4
βˆ’3n4 n6
βˆ’3mβˆ’3 n4
(e)
=
=
=
4m4 nβˆ’6
4m4 m3
(βˆ’3mβˆ’3 n4 )βˆ’1
(2m2 nβˆ’3 )2
10
βˆ’3n
=
4m7
µ
Practice 2. Use a combination of the exponent rules to simplify each exponential expression. Write all answers with only positive exponents. Assume
all variables represent nonzero real numbers. (Answers on page 123 )
¡ 3 βˆ’1 2 ¢ ¡ βˆ’2 βˆ’3 3 ¢
(a)
x y z
x y z
¡ 2 3 ¢βˆ’2
3x y
(b)
(2x3 y 4 )βˆ’3
µ
¶βˆ’3
βˆ’2x3 y βˆ’2 z
(c)
3xβˆ’5 y βˆ’1 z 3
In addition to simplifying expressions, the exponent rules can also be
used to calculate expressions mentally by making the number smaller, and
easier, to work with.
Example 3. Use the laws of exponents to calculate the following expressions
mentally.
273
(a)
(b) 164 · (0.125)4
(c) 1255 · (25)βˆ’7
93
4.3. NEGATIVE INTEGER EXPONENTS
123
Solution 3.
µ ¶3
273
27
(a) 3 =
= 33 = 27.
9
9
µ ¶4 µ ¶4
1
16
4
4
4
(b) 16 · (.125) = 16 ·
=
= 24 = 16.
8
8
¡ ¢βˆ’7
(c) 1255 · (25)βˆ’7 = (53 )5 52
= 515 5βˆ’14 = 51 = 5.
Practice 3. Use the laws of exponents to calculate the following expressions
mentally. (Answers below )
(a)
49
643
(b)
38 · 93
275
ANSWERS TO PRACTICE PROBLEMS
1.
(a)
r4
s6 t4
(b)
βˆ’16x4 z
y3
(b)
(c)
5
64
(c)
2.
(a)
xz 5
y4
3.
(a) 1
(b)
8x5 y 6
9
βˆ’27y 3 z 6
8x24
1
3
SECTION 4.3 EXERCISES
(Answers are found on page 239.)
Evaluate each expression.
1.
3βˆ’4
2.
8βˆ’2
3.
βˆ’5βˆ’4
µ ¶βˆ’3
1
2
8.
(βˆ’4)βˆ’2
9.
4.
5.
6.
βˆ’2βˆ’2
7.
(βˆ’2)βˆ’2
1
10.
µ ¶βˆ’4
4
3
11.
µ ¶βˆ’3
1
6
12.
µ ¶βˆ’2
2
9
5βˆ’2
1
7βˆ’2
124
CHAPTER 4. EXPONENTS AND POLYNOMIALS
Evaluate each expression.
13.
2βˆ’2 + 4βˆ’2
14.
(βˆ’3)βˆ’2 + (βˆ’4)βˆ’1
15.
3βˆ’1 + 2βˆ’2
16.
(βˆ’4)βˆ’2 βˆ’ 2βˆ’1
17.
22 βˆ’ 2 + 2βˆ’1 βˆ’ 2βˆ’2
18.
(βˆ’3)2 + 3 βˆ’ 3βˆ’1 + 3βˆ’2
19.
2βˆ’1 + 3βˆ’1 βˆ’ 4βˆ’1
20.
(βˆ’2)βˆ’1 βˆ’ (βˆ’2)βˆ’2 + (βˆ’2)βˆ’3
µ ¶βˆ’2 µ ¶βˆ’1
4
2
+
3
5
µ ¶βˆ’1
µ ¶βˆ’2
1
1
+ 3βˆ’1 +
+ 3βˆ’2
3
3
µ ¶βˆ’2 µ ¶βˆ’2 µ ¶2 µ ¶2
3
1
1
2
+
βˆ’
βˆ’
3
2
3
2
µ ¶2
µ ¶βˆ’1 µ ¶βˆ’2
1
1
1
1
βˆ’ +
βˆ’
2
2
2
2
21.
22.
23.
24.
Simplify the following, leaving your answer as a single base raised to a single
exponent. Assume all variables represent non-zero real numbers.
25.
x7 · x3 ÷ x4
28.
(52 )4 ÷ 53 · 52
26.
128 ÷ 24
29.
167 · 48 · 83 ÷ 212
27.
23 · 45 ÷ 83
30.
276 ÷ 38 · 94 · 81
4.3. NEGATIVE INTEGER EXPONENTS
125
Use a combination of the rules of exponents to simplify each expression.
Write all answers using positive exponents only. Assume all variables represent nonzero real numbers.
31.
x4 y 5
x3 y 2
44.
¡ βˆ’4 3 βˆ’6 ¢βˆ’3
5x y z
32.
x4 y 3
x9 y 2
45.
18xβˆ’5 y βˆ’2
10xβˆ’2 y βˆ’8
33.
x5 y 12 x9
xy 4
46.
14xβˆ’3 y βˆ’2
24xβˆ’8 y 3
34.
x2 y 4 x5
x9 y 8
47.
¡ βˆ’3 4 ¢βˆ’1
2x y
6xβˆ’5 y βˆ’2
35.
(x2 y 3 )2
xy 5
48.
3βˆ’1 x4 y βˆ’2
3βˆ’2 xβˆ’2 y
36.
x5 y 12 x9
(xy)4
49.
6βˆ’2 x7 y βˆ’3 z 2
6βˆ’3 xβˆ’4 y 5 z 9
37.
(2x3 )(3x2 )
(x2 )3
38.
4x3
(2x4 )(3x)
39.
40.
µ
50.
µ
51.
(4x2 y)(3x3 y 2 )
x7 y 4
µ 2 ¶µ 2 ¶
4a b
5a b
3
2
a b
2b4
41.
¡ βˆ’2 3 5 ¢3
2x y z
42.
¡
¢2
βˆ’4x3 y βˆ’5 z βˆ’1
43.
¡ 4 2 βˆ’3 ¢βˆ’2
3x y z
µ
52.
µ
53.
µ
54.
µ
55.
x2
x4 y 3
xy 6
x4 y 2
¶2
¶3
4x5 y 2
8x4 y 8
¶2
a2 b3 c
a4 bβˆ’1 c2
3x2 y 8
9x4 y 5
¶2
¶βˆ’3
x3 y βˆ’2 z 5
xβˆ’2 y 5 z
¶4
126
56.
CHAPTER 4. EXPONENTS AND POLYNOMIALS
¡ βˆ’3 2 ¢3
2x y
61.
6x5 y βˆ’2
15xβˆ’3 y 8
62.
¡ βˆ’3 5 ¢βˆ’1
4x y
8x5 y βˆ’3
57.
¡ βˆ’1 βˆ’3 ¢2 ¡ βˆ’1 3 ¢βˆ’1
4x y
5x y
58.
¡ 3 2 ¢2 ¡ 2 3 ¢βˆ’3
4x y
2x y
59.
¡ βˆ’2 4 ¢βˆ’2
3x y
63.
9x5 y βˆ’3
24x7 y 8
60.
¡ 4 βˆ’3 ¢βˆ’1 ¡ 6 βˆ’2 ¢
6x y
8x y
64.
¡ βˆ’3 4 ¢3 ¡ 2 βˆ’1 ¢βˆ’2
2x y
3x y
Use a combination of the rules of exponents to simplify each expression.
Write all answers using positive exponents only. Assume all variables represent nonzero real numbers.
65.
(2x2 y βˆ’5 )(6xβˆ’3 y)( 13 xβˆ’1 y 3 )
µ
66.
µ
67.
68.
¶βˆ’2
βˆ’2x4 y βˆ’4
3xβˆ’1 y βˆ’2
¶4
(2x3 y βˆ’2 )3 (βˆ’3xy 5 )βˆ’2
4xβˆ’5 y 7
µ
69.
xy βˆ’2 z βˆ’3
2xβˆ’2 yz 2
βˆ’4x5 y βˆ’2 z βˆ’3
8xβˆ’4 y 6 z βˆ’4
¶βˆ’3
70.
¡ 3 βˆ’2 ¢βˆ’2 ¡ 2 3 ¢2
4x y
2x y
71.
(2x3 y 6 z βˆ’1 )βˆ’2
(3xβˆ’3 y 6 z 2 )βˆ’3
µ
1 4 βˆ’2
x y
3
¶βˆ’1
72. A fellow student said that 85 ÷ 22 = 43 . How would you convince this
student that this is incorrect? What is the correct answer?
4.4. SCIENTIFIC NOTATION
4.4
127
Scientific Notation
One application of exponents is in the use of scientific notation. Scientific
notation is convenient for writing very large numbers. When negative exponents are used with scientific notation, it can be used to represent very
small numbers as well.
A number in scientific notation is written
a × 10n
where 1 ≀ |a| < 10 and n is an integer.
Two examples of scientific notation for small numbers are:
β€’ The width of a strand of DNA is 2 × 10βˆ’9
β€’ The mass of a proton is 1.6724 × 10βˆ’27 kg
In scientific notation there is always exactly one digit before the decimal
point. To rewrite a number using scientific notation the decimal point is
moved so that only one nonzero digit is to the left of the decimal point. The
number of places the decimal point was moved becomes the exponent on
10. This power is negative if the decimal point is moved to the right and
positive if it is moved to the left.
Example 1. Write each number in scientific notation.
(a)
56, 000, 000
(b)
0.0000000345
(c)
The speed of sound is 4790 feet per second
Solution 1.
(a) We begin by moving the decimal point so that only one
nonzero digit is in front of it, in other words, seven places
to the left. Seven becomes the power on 10 and it is positive
since it was moved to the left. Thus,
56, 000, 000 = 5.6 × 107 .
128
CHAPTER 4. EXPONENTS AND POLYNOMIALS
(b) Since we move the decimal point eight places to the right in
order to get only one nonzero digit is front of the decimal
point, we get that
0.0000000345 = 3.45 × 10βˆ’8 .
(c) The decimal place is moved three places to the left. Thus,
4790 = 4.790 × 103 .
Practice 1. Write each number in scientific notation. (Answers on page 130)
(a)
0.00000405
(b) The population of Cuyahoga County is approximately 1, 360, 000 people.
Remember that the exponent on 10 tells you the direction and the number of decimal places the decimal point is moved. To change a number from
scientific notation to standard notation, we reverse the process discussed
before Example 1. In other words, if the exponent n is negative we move
the decimal point |n| places to the left. If the exponent n is positive, we
move the decimal point n places to the right.
Example 2. Convert the following numbers to standard notation (without
exponents).
(a) βˆ’9.75 × 108
(b)
1.4 × 10βˆ’5
(c)
A gram is 3.527 × 10βˆ’2 ounces.
(d)
The radius of the earth is 6.4 × 106 meters.
Solution 2.
(a) βˆ’9.75 × 107 = βˆ’975, 000, 000
(b) 1.4 × 10βˆ’5 = 0.000014
(c) 3.527 × 10βˆ’2 = 0.03527
(d) 6.4 × 106 = 6, 400, 000
4.4. SCIENTIFIC NOTATION
129
Practice 2. Convert the following numbers to standard notation (without
exponents). (Answers on page 130)
(a)
(b)
The number one all-time worldwide box office movie is Titanic
(1997) which earned approximately 1.84 × 109 dollars.
4.8 × 10βˆ’12
Scientific notation also provides an efficient way to multiply and divide
very small and very large numbers. Using the commutative and associative
properties of real numbers, we multiply the numbers between 1 and 10 and
use the rules of exponents to find the power of 10 in the final answer.
Example 3. Perform the indicated operations and write all answers in scientific notation.
(a)
(b)
(c)
(d)
¡
¢¡
¢
4 × 105 2 × 109
9 × 1012
3 × 107
¡
¢¡
¢
5 × 109 3 × 10βˆ’4
¡
¢¡
¢
4 × 107 6 × 103
5 × 104
Solution 3.
(a)
¡
¢¡
¢
4 × 105 2 × 109 = (4 × 2)(105 × 109 ) = 8 × 1014
9 × 1012
9 1012
= ×
= 3 × 105
7
3 × 10
3
107
¡
¢¡
¢
5 × 109 3 × 10βˆ’4 = (5 × 3)(109 × 10βˆ’4 ) = 15 × 105 =
(c)
(1.5 × 10) × 105 = 1.5 × 106
¡
¢
4 × 107 (6 × 103 )
4 × 6 107 × 103
24 1010
(d)
=
×
=
×
=
5 × 104
5
104
5
104
6
4.8 × 10
(b)
Practice 3. Perform the indicated operations and write all answers in scientific notation. (Answers on page 130)
130
CHAPTER 4. EXPONENTS AND POLYNOMIALS
(a)
(b)
¡
¢¡
¢
2.7 × 104 4 × 109
(2.25 × 108 )
¡
¢¡
¢
7 × 1012 8.1 × 10βˆ’16
(2 × 105 ) (2.1 × 108 )
ANSWERS TO PRACTICE PROBLEMS
1.
(a) 4.05 × 10βˆ’6
(b) 0.0000000000048
(b) 1.36 × 106
2.
3.
(a) 4.8 × 105
(b) 1.35 × 10βˆ’16
(a) 1, 840, 000, 000
SECTION 4.4 EXERCISES
(Answers are found on page 241.)
Convert the following numbers to standard notation (without exponents).
1.
9.46 × 1017
7.
6.93 × 1011
2.
1.81 × 106
8.
3.4 × 10βˆ’8
3.
10βˆ’10
9.
2.75 × 107
4.
4.3 × 10βˆ’3
10.
8.003 × 105
5.
3.2 × 10βˆ’5
11.
9.02 × 10βˆ’6
6.
5.789 × 108
12.
5.486 × 10βˆ’4
Convert the following numbers to scientific notation.
13.
8, 340
19.
294, 000, 000.
14.
0.00079
20.
0.000000345
15.
0.000042
21.
0.0000000703
16.
4, 360, 000
22.
6, 600, 000, 000
17.
0.003005
23.
2, 350, 000, 000, 000
18.
0.000007201
24.
0.000000000532
4.4. SCIENTIFIC NOTATION
131
Perform the indicated operations and write all answers in scientific notation.
¡
¢¡
¢
25. 2.4 × 10βˆ’5 3.0 × 10βˆ’7
¡
¢¡
¢
26. 5.2 × 107 4.3 × 10βˆ’3
¢¡
¢¡
¢
¡
27. 2.1 × 10βˆ’4 3.5 × 108 1.2 × 106
¡
¢¡
¢¡
¢
28. 3.5 × 104 6.12 × 10βˆ’11 4.1 × 108
¡
¢¡
¢¡
¢
29. 5.1 × 10βˆ’6 3.02 × 1011 2 × 10βˆ’8
30.
9.6 × 109
1.5 × 103
31.
6.88 × 104
4.3 × 10βˆ’8
4.8 × 1013
(5 × 109 ) (2.4 × 1015 )
¡
¢¡
¢
3.5 × 10βˆ’3 5.2 × 108
33.
9.1 × 10βˆ’7
32.
34.
6.7 × 104 7.1 × 103
×
5 × 10βˆ’6
4 × 1010
35.
3
8 × 106
×
4 × 105 2 × 108
132
CHAPTER 4. EXPONENTS AND POLYNOMIALS
Chapter 5
Factoring
5.1
GCF and Factor by Grouping
Now we will develop some techniques to factor polynomials with integer
coefficients. The first and most basic technique is to factor out the greatest
common factor. Since factoring is the reverse process of multiplication, we
will start this section with a multiplication problem.
¡
¢
Example 1. Find the product: 2x2 y 3x3 y 2 + 5xy 2 + 7x + 10
Solution 1. We use the Distributive Law as follows:
¡
¢
2x2 y 3x3 y 2 + 5xy 2 βˆ’ 10 = 2x2 y · 3x3 y 2 + 2x2 y · 5xy 2 βˆ’ 2x2 y · 10
= 6x5 y 3 + 10x3 y 3 βˆ’ 20x2 y.
Note that the result is a polynomial, each of whose terms has the
monomial 2x2 y as a factor. That is, 2x2 y is a common factor
of all of the terms of the polynomial.
Recall that the greatest common factor of two integers is the largest
integer that is a factor (or divisor) of both. We used this idea when writing a
fraction in lowest terms. Similarly, we define the greatest common factor
(GCF) of two or more monomials to be the monomial of highest degree and
largest coefficient that is a factor of all. We have already used this idea when
writing fractions containing variables in lowest terms. (Click here to review
GCF from from Core Math I.)
Example 2. Find the GCF of the list of integers: 18, 63, and 36.
133
134
CHAPTER 5. FACTORING
Solution 2. You may recognize right away that 9 is the greatest
common factor. If this isn’t immediate, simply factor each into
a product of prime factors.
18 = 2 · 32
63 = 32 · 7
36 = 22 · 32
So the GCF of the integers 18, 63, and 36 is 32 = 9.
Practice 1. Find the GCF of each triple.
(a) 126, 42, 105
(Answers on page 139.)
(b) 625, 81, 15
Example 3. Find the GCF of the list of powers of x: x5 , x3 , and x8 .
Solution 3. The smallest exponent of x appearing in the list is
3. This means that x3 is a factor of each expression in the list.
If it’s not clear why, consider the following factorizations using
the laws of exponents.
x5 = x3 · x2
x3 = x3 · 1
x8 = x3 · x5
So the GCF of x5 , x3 , and x8 is x3 .
Practice 2. Find the GCF of each triple.
(a) y 12 , y 5 , y 6
(Answers on page 139.)
(b) w2 , w, w4
Example 4. Find the GCF of the monomials 18x5 yz 2 , 63x3 y 3 z 3 , and 36x8 y 2 .
5.1. GCF AND FACTOR BY GROUPING
135
Solution 4. We can consider the coefficients and variables one
at a time.
β€’
β€’
β€’
β€’
The GCF of the coefficients 18, 63, and 36 is 9.
The highest power of x that divides x5 , x3 , and x8 is x3 .
The highest power of y that divides y, y 3 , and y 2 is y.
Finally, z is a factor of the first two monomials, but not
the third, so the GCF will not contain z.
So the GCF of the monomials 18x5 yz 2 , 63x3 y 3 z 3 , and 36x8 y 2 is
9x3 y.
Practice 3. Find the GCF of each triple
(a) 22xy 2 z, 121x3 yz 2 , 33y 2 z 3
(Answers on page 139.)
(b) 17a2 b, 3ab2 , 15b3
We now apply this to the factoring of polynomials with more than one
term. Find the GCF of all the terms of the polynomial and then factor it
out using the distributive law.
Example 5. Factor out the GCF of each of the following polynomials.
(a) P (x) = 8x + 4
(b) P (x) = 9x3 βˆ’ 12x2 + 15x
(c) P (x, y, z) = 18x5 yz 2 + 63x3 y 3 z 3 + 36x8 y 2
Solution 5. (a) The GCF of the terms 8x and 4 is 4. Thus,
we obtain
P (x) = 8x + 4
= 4 · 2x + 4 · 1
= 4 (2x + 1) .
(b) The GCF of the terms 9x3 , 12x2 , and 15x is 3x. Thus, we
obtain
P (x) = 9x3 βˆ’ 12x2 + 15x
= 3x · 3x2 βˆ’ 3x · 4x + 3x · 5
¡
¢
= 3x 3x2 βˆ’ 4x + 5 .
136
CHAPTER 5. FACTORING
(c) We saw in Example 4 that the GCF of the terms of this
polynomial is 9x3 y. Thus, we obtain
P (x, y, z) = 18x5 yz 2 + 63x3 y 3 z 3 + 36x8 y 2
= 9x3 y · 2x2 z 2 + 9x3 y · 7y 2 z 3 + 9x3 y · 4x5 y
¡
¢
= 9x3 y 2x2 z 2 + 7y 2 z 3 + 4x5 y .
Practice 4. Factor the GCF out of each polynomial.
(a) P (x) = x2 βˆ’ x
(b) P (y) = 5y 2 + 10y βˆ’ 15
(c) P (x, y) = 18x2 y 2 βˆ’ 36xy 3 + 60xy 5
(Answers on page 139.)
If the leading coefficient of the polynomial is negative, it is customary
to factor out the negative. It is important to factor βˆ’1 out of each term.
Example 6. Factor out the GCF of P (t) = βˆ’100t5 + 75t3 βˆ’ 125t2 βˆ’ 25t.
Solution 6. The GCF of the terms of this polynomial is 25t.
Since the leading coefficient, βˆ’100, is negative, we will factor
out βˆ’25t. Thus, we obtain
P (t) = βˆ’100t5 + 75t3 βˆ’ 125t2 βˆ’ 25t
¡ ¢
¡
¢
= (βˆ’25t) 4t4 + (βˆ’25t) βˆ’3t2 + (βˆ’25t) (5t) + (βˆ’25t) (1)
¡
¢
= βˆ’25t 4t4 βˆ’ 3t2 + 5t + 1 .
Be careful with the last term! Since the expression being factored
out is one of the terms of the polynomial, we are left with a factor
of 1 for that term when the GCF is factored out.
Practice 5. Factor the GCF out of each polynomial.
(a) P (x) = βˆ’9x5 + 21x4 βˆ’ 15x3 + 3x2
(b) P (s, t) = βˆ’s4 t2 + 5s3 t3 βˆ’ 11s2 t4
(Answers on page 139.)
In the previous examples, the GCF was a monomial. However, it is also
possible to have a binomial as a common factor.
5.1. GCF AND FACTOR BY GROUPING
137
Example 7. Factor out the GCF of P (x) = 3x(x + 2) βˆ’ 5(x + 2).
Solution 7. The GCF of the terms of this polynomial is the
binomial (x + 2). Thus, we obtain
P (x) = 3x(x + 2) βˆ’ 5(x + 2)
= (x + 2)(3x βˆ’ 5).
Practice 6. Factor the GCF out of the polynomial: P (y) = 2y 2 (3y βˆ’ 1) + 7(3y βˆ’ 1)
(Answer on page 139.)
We have seen from the previous examples that the GCF can be a number,
variable, or algebraic expression. In the previous example, the GCF was easy
to see because it was inside of parentheses. Now suppose that our polynomial
has four terms such as x3 + 2x2 + 2x + 4. Note that this polynomial does
not have a GCF. However, if we group the first two terms together then
the GCF of the first two terms is x2 , giving us x3 + 2x2 = x2 (x + 2). The
GCF of the last two terms is 2 which yields 2x + 4 = 2(x + 2). Putting this
together we have
3
2
+ 4} = x2 (x + 2) + 2(x + 2).
x
{z2x} + |2x{z
| +
Now the (x + 2) becomes the GCF as in our last example. Therefore, we
can factor the polynomial as
x3 + 2x2 + 2x + 4 = x2 (x + 2) + 2(x + 2)
= (x + 2)(x2 + 2)
This method is called factor by grouping. Note that in order for factor
by grouping to succeed, the algebraic expression in the parentheses must be
identical.
Example 8. Factor:
2x3 + x2 + 50x + 25
Solution 8. We will group the first two terms together and group
the last two terms together. For the first two terms the GCF is
x2 and for the last two terms the GCF is 25. Factor by grouping
yields
3
2x
+ x}2 + 50x
+ 25} = x2 (2x + 1) + 25(2x + 1)
| {z
| {z
= (2x + 1)(x2 + 25)
138
CHAPTER 5. FACTORING
Practice 7. Factor:
3x3 + 12x2 βˆ’ 5x βˆ’ 20 (Answer on the next page.)
Example 9. Factor:
18x3 βˆ’ 27x2 + 8x βˆ’ 12
Solution 9. Again, we group the first two terms and also the
last two terms. The GCF of the first two terms is 9x2 and the
GCF of the last two terms is 4. Therefore, factor by grouping
yields
3
βˆ’ 12} = 9x2 (2x βˆ’ 3) + 4(2x βˆ’ 3)
βˆ’ 27x}2 + 8x
| {z
|18x {z
= (2x βˆ’ 3)(9x2 + 4)
Practice 8. Factor:
6x3 βˆ’ 4x2 + 15x βˆ’ 10 (Answer on the facing page.)
In the previous two examples, we did not need to rearrange the terms
of our polynomial. However, sometimes it is necessary to reorder the terms
for factor by grouping to work.
Example 10. Factor:
10x2 βˆ’ 12y + 15x βˆ’ 8xy
Solution 10. The current order does not lead to a common factor since
10x2 βˆ’ 12y + 15x βˆ’ 8xy = 2(5x2 βˆ’ 6y) + x(15 βˆ’ 8y).
| {z } | {z }
Factoring cannot proceed because the polynomial inside each of
the parentheses is different. Therefore, we must first rearrange
the terms.
10x2 βˆ’ 12y + 15x βˆ’ 8xy = 10x2 βˆ’ 8xy βˆ’ 12y + 15x
{z
} | {z }
|
= 2x(5x βˆ’ 4y) + 3(βˆ’4y + 5x)
= 2x(5x βˆ’ 4y) + 3(5x βˆ’ 4y)
= (5x βˆ’ 4y)(2x + 3)
Practice 9. Factor 10x3 βˆ’ 2x2 y 2 βˆ’ 5xy + y 3 (Answers on the next page.)
5.1. GCF AND FACTOR BY GROUPING
139
ANSWERS TO §5.1 PRACTICE PROBLEMS
1.
(c) 6xy 2 (3x βˆ’ 6y + 10y 3 )
(a) 21
(b) 1
2.
(a) y
5.
5
(b) βˆ’s2 t2 (s2 βˆ’ 5st + 11t2 )
(b) w
3.
6. (3y βˆ’ 1)(2y 2 + 7)
(a) 11yz
7. (x + 4)(3x βˆ’ 5)
(b) b
4.
(a) βˆ’3x2 (3x3 βˆ’ 7x2 + 5x βˆ’ 1)
8. (2x2 + 5)(3x βˆ’ 2)
(a) x(x βˆ’ 1)
(b) 5(y 2 + 2y βˆ’ 3)
9. (2x2 βˆ’ y)(5x βˆ’ y 2 )
Section 5.1 EXERCISES:
(Answers are found on page 242.)
Find GCF of the following pairs and triples.
1. 52 and 78
5. 6, 9, and 27
2. 36 and 30
6. 10, 40, and 50
3. 100 and 140
7. 20, 18, and 45
4. 63 and 45
8. 64, 36, and 100
9. 24, 40, and 60
10. 15, 60, and 81
Find GCF of the following pairs and triples.
11. x4 , x8 , and x7
16. xy 2 , y 3 z 2 , and x4 z 4
12. y 2 , y 5 , and 1
17. 14x2 y and 36x5 y 3
13. x3 y 2 , xy 5 , and x6 y 3
18. 18x5 yz 2 ,
24x3 y 2 z 4
14. x5 y 3 , x3 y 2 , and x5 y 4
19. 6x3 y 3 z 2 , 10x4 y 2 z, and 8x3 y 4 z 2
15. x3 y 2 z, x5 y 2 z 3 , and x2 y 4 z 2
20. 15x2 y 2 , 25x2 z 3 , and 35x3 z 3
15x2 y 2 z 3 ,
Factor out the GCF of the following polynomials.
21. 15x2 βˆ’ 10
23. 10x2 + 15x βˆ’ 45
22. 26x3 βˆ’ 39x
24. x3 + 6x2 βˆ’ 17x
and
140
CHAPTER 5. FACTORING
25. βˆ’18x6 + 18x4 + 18x2 βˆ’ 18
28. 40x4 y 2 + 24x2 y 8 βˆ’ 32x3 y 5
26. βˆ’x3 y + 2x2 y + 3xy βˆ’ 5y
29. 16x2 y 2 + 4x2 y βˆ’ 6xy 2
27. 16a8 βˆ’ 18a6 βˆ’ 30a5
30. x3 y 2 z 2 + 3x2 y 3 z 2 βˆ’ 2x2 y 2 z 3
Factor out the GCF of the following polynomials.
31. 3(x + 2) βˆ’ x(x + 2)
36. 7x(x2 βˆ’ 5x βˆ’ 4) + 8(x2 βˆ’ 5x βˆ’ 4)
32. 3x2 (x2 βˆ’ x) + 2x(x2 βˆ’ x)
37. 2y(xy + 1) + 3x(xy + 1)
33. x(y βˆ’ 3) + 5(y βˆ’ 3)
38. x2 y(x2 βˆ’ y 2 ) βˆ’ xy 2 (x2 βˆ’ y 2 )
34. 5x(x + y) + 4y(x + y)
39. 5x2 (5x2 βˆ’ 6) βˆ’ 6(5x2 βˆ’ 6)
35. 3x2 (x + 6) βˆ’ 5x(x + 6) + 7(x + 6) 40. 3x(x2 + 9x βˆ’ 1) βˆ’ (x2 + 9x βˆ’ 1)
Factor out the GCF of the following polynomials.
41. x2 (x2 + 2x + 3) βˆ’ 10x(x2 + 2x + 3) βˆ’ 2(x2 + 2x + 3)
42. xy(2x2 + 3xy + y) βˆ’ 4x(2x2 + 3xy + y) + 5y(2x2 + 3xy + y)
43. (x2 βˆ’ 3x)(x2 + 4x + 5) + (6x + 2)(x2 + 4x + 5)
44. (2x + 3y)(6x βˆ’ 5y) + 6(6x βˆ’ 5y)
45. 13x2 (x2 + xy + y) βˆ’ 26(x2 + xy + y)
46. 15(x + 3)(3x2 βˆ’ 5) + 25(3x2 βˆ’ 5)
Factor the following polynomials by grouping.
47.
2x3 + 3x2 + 8x + 12
54.
7x3 + 3x2 + 63x + 27
48.
5x3 βˆ’ x2 + 45x βˆ’ 9
55.
16x3 βˆ’ 12x2 + 4x βˆ’ 3
49.
3x3 βˆ’ 2x2 + 3x βˆ’ 2
56.
24x3 βˆ’ 4x2 βˆ’ 18x + 3
50.
4x3 + 12x2 + 9x + 27
57.
16x3 βˆ’ 4x2 βˆ’ 20x + 5
51.
18x3 βˆ’ 9x2 + 2x βˆ’ 1
58.
18x3 βˆ’ 27x2 + 8x βˆ’ 12
52.
12x3 βˆ’ 16x2 + 3x βˆ’ 4
59.
16x3 βˆ’ 32x2 βˆ’ 3x + 6
53.
5x3 βˆ’ x2 + 20x βˆ’ 4
60.
25x3 βˆ’ 25x2 βˆ’ 7x + 7
5.1. GCF AND FACTOR BY GROUPING
141
61.
2x3 + x2 + 50x + 25
66.
7x2 βˆ’ xy + 14x βˆ’ 2y
62.
9x3 βˆ’ 4x2 + 9x βˆ’ 4
67.
6ab + 24b + a + 4
63.
2a3 βˆ’ b2 + a2 b βˆ’ 2ab
68.
10x2 βˆ’ 8xy βˆ’ 12y + 15x
64.
2xy + 12 βˆ’ 3y βˆ’ 8x
69.
5x2 + 15xy βˆ’ 2xz βˆ’ 6yz
65.
4x3 βˆ’ 5 + x βˆ’ 20x2
70.
6x + 4xy + 3 + 2y
142
5.2
CHAPTER 5. FACTORING
Factoring Special Products
In this section, we will learn to recognize and factor polynomials which follow
certain patterns.
First, we will factor trinomials which are the square of a binomial. Remembering that factoring is the reverse process of multiplication, we will
first see how these patterns arise by squaring binomials.
Example 1. Find each square and simplify the result.
(a) (x + 5)2
(b) (2x βˆ’ 3)2
Solution 1. (a) We may use the Distributive Law as follows.
(x + 5)2 = (x + 5) (x + 5)
= x (x + 5) + 5 (x + 5)
= x2 + 5x + 5x + 52
= x2 + 2(5x) + 52
= x2 + 10x + 25.
(b) This time we will use the rectangular array approach that
was introduced in Section 4.2. The set-up is:
2x
βˆ’3
2x
βˆ’3
Multiplying each row entry by each column entry, we obtain:
2x
βˆ’3
2x
(2x)2
βˆ’6x
βˆ’3
βˆ’6x
(βˆ’3)2
Thus,we obtain:
(2x βˆ’ 3)2 = (2x)2 + 2(βˆ’6x) + (βˆ’3)2
= 4x2 βˆ’ 12x + 9
5.2. FACTORING SPECIAL PRODUCTS
143
In each of these examples, we observe that the product of the binomial
has three terms. The first term of the product is the square of the first term
of the binomial; the third term of the product is the square of the last term
of the binomial; and the middle term of the product is twice the product of
the two terms of the binomial. This leads us to the following special product
forms.
Special products:
a2 + 2ab + b2 = (a + b)2
a2 βˆ’ 2ab + b2 = (a βˆ’ b)2
Example 2. Factor:
(a) P (x) = x2 + 14x + 49
(b) P (y) = 9y 2 βˆ’ 12y + 4
(c) P (x, y) = 121x2 + 110xy + 25y 2
Solution 2. (a) The first term of P (x) is x2 and the last term
is 72 . Now we must check if the middle term follows either
of our patterns above. Twice the product of x and 7 is
2 · x · 7 = 14x, which is what we have. Thus,
P (x) = x2 + 14x + 49 = (x + 7)2 .
(b) The first term of P (y) is (3y)2 and the last term is 22 .
Now we must check if the middle term follows either of our
patterns above. Twice the product of 3y and 2 is 2 · 3y · 2 =
12y. Since the middle term of P (y) is βˆ’12y, we have the
square of a difference. Thus,
P (y) = 9y 2 βˆ’ 12y + 4 = (3y βˆ’ 2)2 .
(c) The first term of P (x, y) is (11x)2 and the last term is (5y)2 .
Now we must check if the middle term follows either of
144
CHAPTER 5. FACTORING
our patterns above. Twice the product of 11x and 5y is
2 · 11x · 5y = 110xy, which is the middle term of P (x, y).
Thus,
P (x, y) = 121x2 + 110xy + 25y 2 = (11x + 5y)2 .
Practice 1. Factor.
(Answers on page 148.)
(a) P (x) = x2 βˆ’ 4x + 4
(b) P (t) = 49t2 + 70t + 25
(c) P (x, y) = x2 βˆ’ 6xy + 9y 2
Sometimes we will have to use more than one factoring technique for the
same polynomial. Always check first if the terms have a GCF which can be
factored out.
Example 3. Factor: P (t) = t3 + 12t2 + 36t.
Solution 3. We see that t is the GCF of the three terms of P (t)
and so we first factor out t.
¡
¢
P (t) = t3 + 12t2 + 36t = t t2 + 12t + 36
Now
the polynomial that remains. The first term
¡ 2we try to factor
¢
of t + 12t + 36 is t2 and the last term is 62 . Twice the product
of t and 6 is 2 · t · 6 = 12t, which is what we have. Thus,
P (t) = t(t + 6)2 .
Practice 2. Factor.
(Answers on page 148.)
(a) P (x) = x4 + 20x3 + 100x2
(b) P (u, v) = 2u2 v βˆ’ 4uv + 2v
In the previous section we studied perfect squares of binomials, that
is, polynomials of the form (a + b)2 and (a βˆ’ b)2 . But what happens if we
multiply (a + b) by (a βˆ’ b)?
5.2. FACTORING SPECIAL PRODUCTS
Example 4. Multiply and simplify the result.
(a) (x βˆ’ 2)(x + 2)
(b) (3x + 1)(3x βˆ’ 1)
Solution 4. (a) We may use the Distributive Law as follows.
(x βˆ’ 2) (x + 2) = x (x + 2) βˆ’ 2 (x + 2)
= x2 + 2x βˆ’ 2x βˆ’ 2 · 2
= x2 + 0x βˆ’ 22
= x2 βˆ’ 4.
There are two important observations to make. First, the
two x terms in the product are additive inverses of one another, so the result when simplified has just two terms. Second, the resulting binomial is the difference of two perfect
squares.
(b) This time we will use the rectangular array approach. The
set-up is:
3x
βˆ’1
3x
1
Performing the multiplication, we have:
3x
1
3x
(3x)2
3x
βˆ’1
βˆ’3x
(1)(βˆ’1)
Thus,we obtain,
(3x + 1) (3x βˆ’ 1) = (3x)2 βˆ’ 3x + 3x βˆ’ 1 · 1
= (3x)2 + 0x βˆ’ 12
= 9x2 βˆ’ 1
Once again, two of the four terms of the product are additive inverses of one another. Thus, the simplified form is a
binomial. Also, the result is the difference of two squares.
145
146
CHAPTER 5. FACTORING
In fact, if we multiply any factors of the form (a βˆ’ b)(a + b), we obtain
(a βˆ’ b) (a + b) = a (a βˆ’ b) βˆ’ b (a + b)
= a · a + ab βˆ’ ba βˆ’ b · b
= a2 + 0 βˆ’ b2
= a2 βˆ’ b2
We can interpret this as a factoring formula for the difference of two
squares.
Difference of Two Squares:
a2 βˆ’ b2 = (a βˆ’ b)(a + b)
Example 5. Factor.
(a) P (t) = 4t2 βˆ’ 121
(b) P (x) = x3 βˆ’ x
(c) P (y) = y 4 βˆ’ 81
Solution 5. (a) We observe that 4t2 = (2t)2 and 121 = 112 .
Thus, P (t) is the difference of two squares.
P (t) = 4t2 βˆ’ 121
= (2t)2 βˆ’ 112
= (2t βˆ’ 11)(2t + 11).
(b) First we factor out the greatest common factor.
P (x) = x3 βˆ’ x
= x(x2 βˆ’ 1)
= x(x2 βˆ’ 12 )
= x(x βˆ’ 1)(x + 1).
(c) Here we observe that y 4 = (y 2 )2 .
P (y) = y 4 βˆ’ 81
= (y 2 )2 βˆ’ 92
= (y 2 βˆ’ 9)(y 2 + 9)
= (y βˆ’ 3)(y + 3)(y 2 + 9).
5.2. FACTORING SPECIAL PRODUCTS
147
Note that y 2 + 9 is the sum of two squares and does not
factor further under the real numbers.
We say that a polynomial such as y 2 + 9 in Example 55 is prime (or
irreducible) over the real numbers since it cannot be factored as a
product of nonconstant factors with real coefficients. In fact, the sum of
two squares is always prime over the real numbers.
Practice 3. Factor.
(Answers on the next page.)
(a) P (x) = 25 βˆ’ 49x2
(b) P (y) = y 5 βˆ’ 36y 3
(c) P (t) = 625t4 βˆ’ 1
Finally, let us consider factor by grouping again. Remember to look for
the difference of squares.
Example 6. Factor:
8x3 + 4x2 βˆ’ 18x βˆ’ 9
Solution 6. Since this polynomial has four terms, we will use
factoring by grouping.
3
2
8x
+ 4x}2 βˆ’ 18x
| {z
| {zβˆ’ 9} = 4x (2x + 1) βˆ’ 9(2x + 1)
= (2x + 1)(4x2 βˆ’ 9)
= (2x + 1)(2x βˆ’ 3)(2x + 3)
Practice 4. Factor:
2x3 + 3x2 βˆ’ 8x βˆ’ 12 (Answers on the following page.)
148
CHAPTER 5. FACTORING
ANSWERS TO PRACTICE PROBLEMS
1.
(a) (x βˆ’ 2)2
3.
(b) (7t + 5)2
(c) (x βˆ’ 3y)2
2.
(a) (5 βˆ’ 7x)(5 + 7x)
(b) y 3 (y βˆ’ 6)(y + 6)
(c) (5t βˆ’ 1)(5t + 1)(25t2 + 1)
(a) x2 (x + 10)2
(b) 2v(u βˆ’ 1)2
4. (2x + 3)(x βˆ’ 2)(x + 2)
Section 5.2 EXERCISES:
(Answers are found on page 243.)
Multiply and simplify your answers.
1. (x + 4)2
9. (2x + 1)(2x + 1)
2. (x βˆ’ 4)2
10. (4x βˆ’ 3)(4x + 3)
3. (x + 4)(x βˆ’ 4)
4. (x βˆ’ 10)2
5. (x + 10)(x + 10)
6. (x + 10)(x βˆ’ 10)
11. (3x + 7)2
12. (4x βˆ’ 5)(4x βˆ’ 5)
13. (2x + 3y)2
7. (2x βˆ’ 1)(2x βˆ’ 1)
14. (3x + 5y)(3x βˆ’ 5y)
8. (2x + 1)(2x βˆ’ 1)
15. (4x βˆ’ 2z)2
Factor each of the following polynomials.
16. x2 βˆ’ 16
22. x2 + 6x + 9
17. x2 βˆ’ 81
23. x2 βˆ’ 8x + 16
18. 9x2 βˆ’ 4
24. 4x2 + 4x + 1
19. 5x2 βˆ’ 20
25. 2x2 βˆ’ 4x + 2
20. 36x2 βˆ’ 9
26. 12x2 + 36x + 27
21. 28x2 βˆ’ 63
27. 5x2 + 20x + 20
5.2. FACTORING SPECIAL PRODUCTS
149
Find the value(s) of b which make each of the following polynomials a square.
28. x2 + 12x + b
39. bx2 + 12x + 4
29. x2 βˆ’ 6x + b
40. x2 + bx + 1
30. x2 βˆ’ 14x + b
41. x2 + bx + 16
31. x2 + 8x + b
32. 4x2 βˆ’ 4x + b
2
33. 9x + 6x + b
2
34. 9x + 12x + b
35. 25x2 βˆ’ 30x + b
42. 9x2 + bx + 1
43. 25x2 + bx + 1
44. 9x2 + bx + 4
45. 4x2 + bx + 36
36. bx2 + 2x + 1
46. x2 + bxy + y 2
37. bx2 βˆ’ 6x + 1
47. x2 + bxy + 9y 2
38. bx2 + 10x + 25
48. 25x2 + bxy + y 2
Determine which of the following polynomials is a difference of squares.
Factor those polynomials that are differences of squares.
49. x2 βˆ’ 49
52. 2x2 βˆ’ 8
50. x2 βˆ’ 121
53. 16x2 βˆ’ 25
51. x2 + 9
54. 9x2 + 4
Determine which of the following polynomials is a square. Factor those
polynomials that are squares.
55. x2 βˆ’ 49
56. x2 βˆ’ 14x + 49
60. 9x2 + 6x + 1
61. 25x2 + 20x + 4
2
57. x + 7x + 49
58. x2 + 16x + 64
59. 4x2 βˆ’ 2x + 1
62. 4x2 βˆ’ 4x βˆ’ 1
63. 4x2 βˆ’ 12x + 9
150
CHAPTER 5. FACTORING
Factor the following differences of squares.
64. x2 βˆ’
69.
9 2
x βˆ’ 16
49
65.
70.
16 2
4
x βˆ’
25
49
71.
49
1 2
x βˆ’
121
100
66.
67.
68.
1
4
4
y2 βˆ’
9
9 2
x βˆ’1
4
25 2
y βˆ’1
16
1 2
x βˆ’ 25
64
72. x2 βˆ’ 169
73. 25x2 βˆ’ 4
Factor each of the following squares.
74. x2 + 20x + 100
75. y 2 + 14y + 49
76. x2 βˆ’ 22x + 121
77. 4w2 βˆ’ 36w + 81
78. x2 y 2 + 4xy + 4
79. (xy)2 + 10xy + 25
9
6
83. y 2 + y +
5
25
84. z 2 βˆ’
85.
1 2 2
x βˆ’ x+1
81
9
86.
1 2 3
r βˆ’ r+9
16
2
87.
4 2 1
1
x + x+
9
3
16
88.
25 2 5
4
x + x+
16
3
9
80. 28x2 + 84x + 63
81. x2 βˆ’ x +
1
4
2
1
82. x2 + x +
3
9
12
36
z+
7
49
Factor by grouping.
89. 5x3 βˆ’ x2 βˆ’ 45x + 9
92. 18x3 + 9x2 βˆ’ 2x βˆ’ 1
90. 3x3 βˆ’ 2x2 βˆ’ 3x + 2
93. 12x3 βˆ’ 16x2 βˆ’ 3x + 4
91. 4x3 + 12x2 βˆ’ 9x βˆ’ 27
94. 5x3 βˆ’ x2 βˆ’ 20x + 4
5.2. FACTORING SPECIAL PRODUCTS
95. 7x3 + 3x2 βˆ’ 63x βˆ’ 27
99. 18x3 βˆ’ 27x2 βˆ’ 8x + 12
96. 16x3 βˆ’ 12x2 βˆ’ 4x + 3
100. 16x3 + 32x2 βˆ’ x βˆ’ 2
97. 24x3 βˆ’ 4x2 βˆ’ 6x + 1
101. 25x3 βˆ’ 25x2 βˆ’ 4x + 4
98. 16x3 βˆ’ 4x2 βˆ’ 4x + 1
102. 2x3 + x2 βˆ’ 50x βˆ’ 25
151
152
5.3
CHAPTER 5. FACTORING
Factoring Trinomials
In this section, we will factor trinomials which are not perfect squares. As
usual, we will begin with some multiplication problems.
Example 1. Find each product and simplify the result.
(a) (x + 3) (x βˆ’ 5)
(b) (2x + 1) (x + 8)
Solution 1. (a) We may use the Distributive Law as follows.
(x + 3) (x βˆ’ 5) =
=
=
=
x (x βˆ’ 5) + 3 (x βˆ’ 5)
x2 βˆ’ 5x + 3x + 3(βˆ’5)
x2 + (βˆ’5 + 3)x βˆ’ 15
x2 βˆ’ 2x βˆ’ 15
(b) This time we will use the rectangular array approach. The
set-up is:
x
8
2x
1
Performing the multiplication, we have:
2x
1
x
2x2
x
8
16x
8
Thus,we obtain,
(2x + 1) (x + 8) = 2x2 + 16x + x + 8
= 2x2 + (16 + 1)x + 8
= 2x2 + 17x + 8
Practice 1. Find each product and simplify the result. (Answers on page 159.)
(a) (x βˆ’ 7) (x βˆ’ 12)
(b) (3x βˆ’ 2) (2x + 6)
5.3. FACTORING TRINOMIALS
153
In each of these examples, the product of the binomials has three terms
after like terms have been combined. A polynomial with three terms is
called a trinomial. Now we wish to factor trinomials into products of
two binomials. We will restrict our attention to factorizations over the
integers; that is, factorizations where the coefficients of the factors are all
integers.
Example 2. Factor P (x) = x2 + 11x + 30.
Solution 2. Since the leading coefficient is 1, we are looking for
a factorization of the form
P (x) = (x + a)(x + b)
where a and b are integers and the product ab = 30 and the
sum a + b = 11. If such a factorization does not come to mind
immediately, we can systematically consider all factorizations of
30 into a product of two positive integers.
ab
1 · 30
2 · 15
3 · 10
5·6
a+b
1 + 30 = 31
2 + 15 = 17
3 + 10 = 13
5 + 6 = 11 ?
Thus, we claim that
P (x) = x2 + 11x + 30 = (x + 5)(x + 6).
We will check our factorization by multiplying.
x
5
x
x2
5x
6
6x
30
Thus,
(x + 5)(x + 6) = x2 + 6x + 5x + 30
= x2 + 11x + 30,
which is what we claimed.
154
CHAPTER 5. FACTORING
Practice 2. Factor P (x) = x2 + 5x + 6. (Answer on page 159.)
Example 3. Factor P (x) = x2 βˆ’ x βˆ’ 12.
Solution 3. We will first list all pairs of integer factors of βˆ’12
and their sums.
ab
βˆ’1 · 12
1 · βˆ’12
βˆ’2 · 6
2 · βˆ’6
βˆ’3 · 4
3 · βˆ’4
a+b
βˆ’1 + 12 = 11
1 βˆ’ 12 = βˆ’11
βˆ’2 + 6 = 4
2 βˆ’ 6 = βˆ’4
βˆ’3 + 4 = 1
3 + 4 = βˆ’1 ?
We are hoping to find a pair of factors of βˆ’12 whose sum is βˆ’1,
since this is the coefficient of x. We see that the last pair in the
table satisfies this condition. Therefore,
P (x) = x2 βˆ’ x βˆ’ 12
= (x + 3)(x βˆ’ 4).
It’s a good idea to check this by multiplying.
x
3
x
x2
3x
βˆ’4
βˆ’4x
βˆ’12
Thus,
(x + 3)(x βˆ’ 4) = x2 βˆ’ 4x + 3x βˆ’ 12
= x2 βˆ’ x βˆ’ 12,
which is what we claimed.
We could have reduced our work by half in the previous example by
observing that, since the middle coefficient was negative, it must be the
negative factor of βˆ’12 which is largest in absolute value.
5.3. FACTORING TRINOMIALS
Practice 3. Factor P (x) = x2 βˆ’ 2x βˆ’ 35. (Answer on page 159.)
Example 4. Factor P (y) = y 2 βˆ’ 16y + 28.
Solution 4. This time the constant term is positive, but the
coefficient of y is negative. This means we should consider pairs
of negative factors of 28.
ab
βˆ’1 · βˆ’28
βˆ’2 · βˆ’14
βˆ’4 · βˆ’7
a+b
βˆ’1 βˆ’ 28 = βˆ’29
βˆ’2 βˆ’ 14 = βˆ’16 ?
βˆ’4 βˆ’ 7 = βˆ’11
Thus,
P (y) = y 2 βˆ’ 16y + 28
= (y βˆ’ 2)(y βˆ’ 14).
We leave it to the reader to check this by multiplying.
Practice 4. Factor P (w) = w2 βˆ’ 14w + 48. (Answer on page 159.)
Example 5. Factor P (t) = 2t5 + 12t4 βˆ’ 14t3 .
Solution 5. We begin by factoring out the GCF which is 2t3 .
P (t) = 2t5 + 12t4 βˆ’ 14t3
= 2t3 (t2 + 6t βˆ’ 7)
Next, we consider pairs of integer factors of βˆ’7 for which the
largest factor in absolute value is positive. No need to make a
chart; since 7 is prime, the only possibility is βˆ’1 · 7. Thus,
P (t) = 2t3 (t2 + 6t βˆ’ 7)
= 2t3 (t βˆ’ 1)(t + 7).
The reader should check this by multiplying.
155
156
CHAPTER 5. FACTORING
Practice 5. Factor P (x) = 3x4 + 3x3 βˆ’ 330x2 . (Answer on page 159.)
Example 6. Factor P (x) = x2 + 18x + 24, if possible.
Solution 6. Let us consider pairs of integer factors of 24. Since
both the constant term (24) and the coefficient of x (18) are
positive, it suffices to consider pairs of positive integers,
ab
1 · 24
2 · 12
3·8
4·6
a+b
1 + 24 = 25
2 + 12 = 14
3 + 8 = 11
4 + 6 = 10
This is a complete list of all pairs of positive integers whose product is 24. However, no pair in the list has a sum of 18. We
conclude that P (x) is prime over the integers.
Practice 6. Factor P (x) = x2 βˆ’ 10x + 30, or else show that it is prime over
the integers. (Answer on page 159.)
Example 7. Find all values of k so that the polynomial P (x) = x2 + kx βˆ’ 10
will factor into a product of linear factors.
Solution 7. We wish to write P (x) = x2 + kx βˆ’ 10 as a product of the form (x + a)(x + b) where a and b are integers. This
means that ab = βˆ’10 and a + b = k. We will make a complete
list of all pairs of integer factors of βˆ’10.
ab
1 · βˆ’10
βˆ’1 · 10
2 · βˆ’5
βˆ’2 · 5
a+b=k
1 βˆ’ 10 = βˆ’9
βˆ’1 + 10 = 9
2 βˆ’ 5 = βˆ’3
βˆ’2 + 5 = 3
Therefore, if k is βˆ’9, 9, βˆ’3, or 3, then P (x) = x2 + kx βˆ’ 10 will
factor.
5.3. FACTORING TRINOMIALS
157
Practice 7.
a. Write down the four polynomials that arise in Example 7
and factor each.
b. Find all values of k so that the polynomial P (x) = x2 + kx βˆ’ 55 will
factor into a product of linear factors. Then factor each of the resulting
polynomials.
(Answers on page 159.)
Example 8. Factor P (x) = 14x2 + 9x + 1.
Solution 8. In this example, the leading coefficient is not 1,
but the constant term is. Also, the coefficient of x is positive.
Therefore our factorization will be of the form (ax + 1)(bx + 1)
for some integers a and b for which ab = 14 and a + b = 9. Let
us consider pairs of (positive) integer factors of 14.
ab
1 · 14
2·7
a+b
1 + 14 = 15
2+7=9 ?
Hence,
P (x) = 14x2 + 9x + 1
P (x) = (2x + 1)(7x + 1).
We will check our factorization by multiplying.
2x
1
7x
14x2
7x
1
2x
1
Thus,
(2x + 1)(7x + 1) = 14x2 + 2x + 7x + 1
= 14x2 + 9x + 1,
which is what we claimed.
158
CHAPTER 5. FACTORING
Practice 8. Factor P (x) = 15x2 βˆ’ 8x + 1. (Answer on the next page.)
Example 9. Factor P (t) = 5t2 + 14t βˆ’ 3.
Solution 9. In this example, neither the leading coefficient nor
the constant term is 1. Our factorization will be of the form
(ax + c)(bx + d) for some integers a, b, c and d for which ab = 5,
cd = βˆ’3, and ad + bc = 14. That’s an awful lot to keep track
of ! Fortunately, 5 and 3 are both prime, so there are not that
many possibilities. We will use the factorization 5 = 5 · 1 for the
coefficients of x in the factors. Then we need to consider four
factorizations (counting order) of βˆ’3:
βˆ’3 = 3 · (βˆ’1),
βˆ’3 = βˆ’3 · 1,
βˆ’3 = 1 · (βˆ’3), and
βˆ’3 = βˆ’1 · 3.
We will try all of the combinations to see which one (if any)
works.
(5x + 3)(1x βˆ’ 1) = 5x2 βˆ’ 2x βˆ’ 3
(5x βˆ’ 3)(1x + 1) = 5x2 + 2x βˆ’ 3
(5x + 1)(1x βˆ’ 3) = 5x2 βˆ’ 14x βˆ’ 3
(5x βˆ’ 1)(1x + 3) = 5x2 + 14x βˆ’ 3?
Hence,
P (x) = 5x2 + 14x βˆ’ 3
P (x) = (5x βˆ’ 1)(x + 3).
Practice 9. Factor P (x) = 7x2 + 5x βˆ’ 2. (Answer on the facing page.)
Example 10. Factor P (x, y) = 4x2 + 24xy + 11y 2 .
5.3. FACTORING TRINOMIALS
159
Solution 10. In this example, neither the leading coefficient nor
the constant term is 1, and we have two variables. This time
our factorization will be of the form (ax + cy)(bx + dy) for some
integers a, b, c and d for which ab = 4, cd = 11, and ad + bc =
24. Note that a, b, c and d will all be positive. We have two
factorizations of 4 and just one of 11. However, order matters
when the lead coefficients of the binomials are different, so we
actually have 3 possibilities to try.
(4x + 11y)(1x + 1y) = 4x2 + 15xy + 11y 2
(4x + 1y)(1x + 11y) = 4x2 + 45xy + 11y 2
(2x + 11y)(2x + 1y) = 4x2 + 24xy + 11y 2 ?
Hence,
P (x, y) = 4x2 + 24xy + 11y 2
P (x) = (2x + 11y)(2x + y).
Practice 10. Factor P (x) = 6x2 + 13xy + 5y 2 . (Answer below.)
ANSWERS TO PRACTICE PROBLEMS
1.
2
(a) x βˆ’ 19x + 84
2
(b) 6x + 14x βˆ’ 12
(b) k = βˆ’54, 54, βˆ’6, 6
2
P (x) = x βˆ’ 54x βˆ’ 55 = (x + 1)(x βˆ’ 55)
2
P (x) = x + 54x βˆ’ 55 = (x βˆ’ 1)(x + 55)
2
P (x) = x βˆ’ 6x βˆ’ 55 = (x + 5)(x βˆ’ 11)
2
P (x) = x + 6x βˆ’ 55 = (x βˆ’ 5)(x + 11)
2. (x + 2)(x + 3)
3. (x + 5)(x βˆ’ 7)
4. (w βˆ’ 6)(w βˆ’ 8)
2
5. 3x (x βˆ’ 10)(x + 11)
6. prime over the integers
7.
2
P (x) = x βˆ’ 3x βˆ’ 10 = (x + 2)(x βˆ’ 5)
2
P (x) = x + 3x βˆ’ 10 = (x βˆ’ 2)(x + 5)
8. (3x βˆ’ 1)(5x βˆ’ 1)
9. (7x βˆ’ 2)(x + 1)
2
(a) P (x) = x βˆ’ 9x βˆ’ 10 = (x + 1)(x βˆ’ 10)
2
P (x) = x + 9x βˆ’ 10 = (x βˆ’ 1)(x + 10)
10. (2x + y)(3x + 5y)
160
CHAPTER 5. FACTORING
Section 5.3 EXERCISES:
(Answers are found on page 245.)
Factor the following trinomials.
1. x2 + 3x + 2
10. βˆ’x2 + 9x βˆ’ 14
2. x2 βˆ’ x βˆ’ 2
11. 2x2 βˆ’ 3x βˆ’ 2
3. x2 + 2x βˆ’ 3
12. 5x2 + 6x + 1
4. x2 + 6x βˆ’ 7
13. 2x2 βˆ’ 4x + 2
5. x2 βˆ’ 12x + 11
14. 5x2 + 35x + 30
6. x2 βˆ’ 10x βˆ’ 11
15. 6x2 + 8x + 2
7. x2 + 5x + 6
16. x2 + 4xy + 4y 2
8. x2 βˆ’ 2x βˆ’ 8
17. x2 βˆ’ 7xy + 12y 2
9. x2 βˆ’ 7x + 12
18. 5x2 + 11xy + 2y 2
Determine which of the following trinomials can be factored into a product
of linear factors and which are prime.
19. x2 + 4x + 4
27. 3x2 + 5x + 2
20. x2 + 9x + 14
28. 3x2 + 7x + 3
21. x2 + x βˆ’ 2
22. x2 + x βˆ’ 3
2
23. x + x βˆ’ 4
24. x2 + 3x + 3
29. 3x2 + 9x + 3
30. 5x2 + 20x + 10
31. x2 + 5xy + 7y 2
25. 3x2 + 4x + 1
32. x2 βˆ’ 9xy + 18y 2
26. 3x2 + 9x βˆ’ 12
33. 7x2 + 2xy βˆ’ 5y 2
5.3. FACTORING TRINOMIALS
161
For each of the following polynomials, find all the values for b so that the
polynomial will factor into a product of linear factors.
34. x2 + bx + 3
42. x2 + bx + 18
35. x2 + bx βˆ’ 3
43. 3x2 + bx + 3
36. x2 + bx βˆ’ 7
44. x2 + bx + 8
2
37. x + bx + 7
2
38. x + bx + 6
39. x2 + bx βˆ’ 6
45. 6x2 + bx + 5
46. x2 + bxy + 9y 2
40. 2x2 + bx + 1
47. x2 + bxy βˆ’ 8y 2
41. 2x2 + bx βˆ’ 1
48. 7x2 + bxy βˆ’ 2y 2
Factor the following polynomials.
49. 12x2 + 20x βˆ’ 8
63. 12x2 βˆ’ 14x βˆ’ 6
50. 5x3 βˆ’ 45x
64. 12x3 βˆ’ 27x2 βˆ’ 27x
51. 16x2 βˆ’ 40xy + 16y 2
65. 12x3 βˆ’ 62x2 + 10x
52. 32x4 βˆ’ 200
66. 4x2 βˆ’ 2x βˆ’ 20
53. βˆ’28x4 + 84x3 βˆ’ 63x2
67. 8x2 + 46x βˆ’ 12
54.
9 4
y βˆ’ 25
4
68. 8x2 + 34x βˆ’ 84
55. 81t4 βˆ’ 256
69. 6x2 βˆ’ 2x βˆ’ 60
56. 2y 4 βˆ’ 26y 2 + 72
70. 10x2 βˆ’ 15x βˆ’ 10
57. 8x10 βˆ’ 98x4
71. 6x3 βˆ’ 16x2 βˆ’ 6x
58. 450 βˆ’ 5a βˆ’ 5a2
72. 28x2 + 10x βˆ’ 2
59. βˆ’60n3 βˆ’ 300n2 βˆ’ 375n
73. 8x2 βˆ’ 20x + 12
60. 6x3 + 21x2 βˆ’ 12x
74. 30x4 + 2x3 βˆ’ 4x2
61. 21x2 βˆ’ 48x βˆ’ 45
75. 12x2 + 15x βˆ’ 18
62. 12x2 βˆ’ 14x βˆ’ 10
76. 18x2 + 12x βˆ’ 48
162
5.4
CHAPTER 5. FACTORING
Factoring Review
Now it is time to review all the factoring techniques we have just learned.
The following is a general factoring strategy that can be followed.
1. Order the terms in either descending or ascending order according to
the powers of the variable. If there is more than one variable, order
according to the variable of your choice.
2. Look for a Greatest Common Factor (GCF). If the problem has one,
then factor it out. Remember to include the GCF in your final answer.
3. Determine the number of terms in the expression.
* If there are only two terms, determine if it is a difference of two
squares. If so, then factor according to
a2 βˆ’ b2 = (a βˆ’ b)(a + b)
Remember that the sum of two squares cannot be factored further
over the real numbers.
* If there are three terms, determine if it is one of the special forms
shown below:
a2 + 2ab + b2 = (a + b)2
a2 βˆ’ 2ab + b2 = (a βˆ’ b)2
If it is not one of the special forms, factoring using any of the
techniques discussed.
* If there are four or more terms, try to factor by grouping.
4. Be sure to look to see if any factors can be factored further.
The best way to improve your factoring skills is to practice. The more
factoring problems you do, the easier they will become and the quicker you
will be able to see how an individual problem factors.
5.4. FACTORING REVIEW
163
Example 1. Factor Completely.
(a) 4x2 y + 8xy βˆ’ 12y
(e) 7x3 βˆ’ 3x2 βˆ’ 63x + 27
(b) 12x2 + 15x βˆ’ 18
(f ) 30x4 + 2x3 βˆ’ 4x2
(c) 12 + 5x βˆ’ 2x2
(g) 2x2 + 7x βˆ’ 72
(d) 64x2 βˆ’ 25y 2
(h) 48x3 βˆ’ 75x
Solution 1.
(a)
Since the GCF is 4y, we factor this out first. Continuing,
we get
4x2 y + 8xy βˆ’ 12y = 4y(x2 + 2x βˆ’ 3) = 4y(x + 3)(x βˆ’ 1).
(b)
Here the GCF is 3, so we begin by factoring this out.
12x2 + 15x βˆ’ 18 = 3(4x2 + 5x βˆ’ 6) = 3(4x βˆ’ 3)(x + 2).
(c)
There is no GCF and the terms are arranged in ascending
order. Therefore, we use the techniques discussed to factor
as
12 + 5x βˆ’ 2x2 = (4 βˆ’ x)(3 + 2x).
(d)
Since this problem only has two terms, we look to see if
it is the difference of two squares. Note that 64x2 = (8x)2
and 25y 2 = (5y)2 . Therefore, using the difference of squares
formula, we obtain
64x2 βˆ’ 25y 2 = (8x βˆ’ 5y)(8x + 5y).
(e)
Since there are four terms, we will attempt to factor this
by grouping. We group the first two terms together and the
last two terms together. Thus,
3
7x
βˆ’ 3x}2 βˆ’ 63x
+ 27} = x2 (7x βˆ’ 3) βˆ’ 9(7x βˆ’ 3)
| {z
| {z
= (7x βˆ’ 3)(x2 βˆ’ 9)
= (7x βˆ’ 3)(x βˆ’ 3)(x + 3)
Note that the difference of squares formula was also used in
this problem.
164
CHAPTER 5. FACTORING
(f )
The GCF of 2x2 is factored out first. Thus,
30x4 + 2x3 βˆ’ 4x2 = 2x2 (15x2 + x βˆ’ 2)
= 2x2 (5x + 2)(3x βˆ’ 1)
(g)
Since there is no GCF, we use the techniques discussed
to obtain
2x2 + 7x βˆ’ 72 = (2x βˆ’ 9)(x + 8).
(h)
The GCF of 3x is factored out first. Then, the difference
of squares formula is used to obtain
48x3 βˆ’ 75x = 3x(16x2 βˆ’ 25)
= 3x(4x βˆ’ 5)(4x + 5)
Practice 1. Factor Completely . (Answers below )
(a) 4x3 + 8x2 + 4x
(d) 12x3 βˆ’ 62x2 + 10x
(b) 16x3 βˆ’ 4x2 βˆ’ 4x + 1
(e) 10x2 + 17x + 3
(c) 40x8 y 7 βˆ’ 16x9 y 5
(f ) 3x2 y βˆ’ 6xy βˆ’ 45y
ANSWERS TO PRACTICE PROBLEMS
1.
(a) 4x(x + 1)2
(d) 2x(6x βˆ’ 1)(x βˆ’ 5)
(b) (4x βˆ’ 1)(2x βˆ’ 1)(2x + 1)
(e) (5x + 1)(2x + 3)
(c) 8x8 y 5 (5y 2 βˆ’ 2x)
(f) 3y(x βˆ’ 5)(x + 3)
SECTION 5.4 EXERCISES
(Answers are found on page 247.)
Factor Completely.
1. 4x3 y βˆ’ xy 3
4. 1 βˆ’ (a + b)2
2. 5x2 + 10xy βˆ’ 40y 2
5. 6u2 + 7uv βˆ’ 3v 2
3. x4 βˆ’ 2x3 + 3x βˆ’ 6
6. 9x2 βˆ’ x6 y 2
5.4. FACTORING REVIEW
165
7. 2y 4 βˆ’ 5y 2 βˆ’ 12
31. 18x3 βˆ’ 3x2 βˆ’ 6x + 1
8. 3xy + 6xz βˆ’ wy βˆ’ 2wz
32. 3a4 βˆ’ 48a2
9. (x2 βˆ’ 5)2 βˆ’ 6(x2 βˆ’ 5) + 8
33. 40x2 + 14x βˆ’ 45
34. (3x + 2y)2 + (3x + 2y) βˆ’ 12
10. 7a2 + 17a βˆ’ 12
11. 8t3 + 80t2 + 150t
36. 81 βˆ’ (a + 1)2
12. 5x2 βˆ’ 45
13.
a3
14.
27t3
βˆ’
13a2
+
35. x2 y 2 βˆ’ 225
37. 2xy 2 βˆ’ 6x2 βˆ’ 3y 3 + 9xy
βˆ’ 90a
108t2
38. 4(x βˆ’ 7)3 βˆ’ (x βˆ’ 7)
+ 36t + 144
15. (x βˆ’ 3)10 βˆ’ 1
39. 4x3 βˆ’ 20x2 + 25x
40. (a βˆ’ 3)2 βˆ’ (a2 + 3)2
16. 3x3 y βˆ’ 6x2 y 2 + 3xy 3
41. 36x2 βˆ’ 49y 2
17. 2z(x βˆ’ 2y) βˆ’ w(x βˆ’ 2y)
42. 5x3 βˆ’ x2 + 20x βˆ’ 4
18. b2 + 20b βˆ’ 800
43. 12x2 βˆ’ 14x βˆ’ 10
19. 12 βˆ’ 8x βˆ’ 7x2
44. 2x2 + 13x + 15
20. 6ax βˆ’ 2by + 4bx βˆ’ 3ay
45. x2 βˆ’ 7xy + 12y 2
21. 6x2 + 48x + 72
46. 6x2 βˆ’ 11x βˆ’ 10
47. 2x4 y βˆ’ 2x4 y 2 + 2x3 y 2
22. 81y 4 βˆ’ 16x4
23. a2 b2 + b2 c2 βˆ’ c2 d2 βˆ’ a2 d2
24. (3a + 5)a2 + (3a + 5)a
25.
4y 2
βˆ’ 29yz βˆ’
24z 2
26. 18x3 βˆ’ 8x
27.
2x3 y
βˆ’
6x2 y 2
48. 3x3 + 18x2 + 27x
49. 6x3 y 2 βˆ’ 2x2 y βˆ’ 60x
50. 9 + 13x βˆ’ 10x2
51. 25x3 βˆ’ 25x2 βˆ’ 4xy + 4y
52. 36x3 y βˆ’ 16xy 3
βˆ’
8xy 3
53. 30x4 + 2x3 βˆ’ 4x2
28. 15ac βˆ’ 20ad + 3bc βˆ’ 4bd
54. 2x2 y 2 βˆ’ 12xy + 18
29. (x + 2)2 βˆ’ 9y 2
55. 2x5 βˆ’ 14x4 + 24x3
30. 12y 3 + 44y 2 βˆ’ 16y
56. 8x2 βˆ’ 17x + 9
166
CHAPTER 5. FACTORING
57. 8x2 + 34x βˆ’ 84
64. 4x2 βˆ’ 4x βˆ’ 48
58. 81x2 βˆ’ 49
65. 8x2 + 46x βˆ’ 12
59. 14 + 3x βˆ’ 2x2
66. x3 βˆ’ 13x2 βˆ’ 90x
60. 2x3 + 3x2 βˆ’ 8x βˆ’ 12
67. 8x3 + 80x2 + 150x
61. 16x4 βˆ’ 81
68. 3x4 y + 6x3 y βˆ’ 72x2 y
62. 12x2 + 15x βˆ’ 18
69. 8x2 + 6xy βˆ’ 5y 2
63. 4x3 y βˆ’ 12x2 y 2 + 8x2 y
70. 1 βˆ’ 8x βˆ’ 20x2
5.5. SOLVING EQUATIONS BY FACTORING
5.5
167
Solving Equations by Factoring
The key idea in finding the solution to an equation by factoring is the Zero
Product Property . This is because zero has a special property. It is
impossible to multiply two nonzero numbers to obtain a product of zero. In
other words, if a product is zero, then at least one of the factors must be
zero.
Zero Product Property: Let a and b be real numbers. If ab = 0,
then
a = 0 or b = 0 (or both).
Example 1. Solve: (x βˆ’ 3) (x + 2) = 0
Solution 1. We have a product equal to zero. By the Zero Product Property, at least one of the factors must be zero. That is,
xβˆ’3=0
or
x + 2 = 0.
Solving each of the resulting linear equations for x, we obtain
x=3
or
x = βˆ’2.
We leave it to the reader to check each solution in the original
equation.
Practice 1. Solve for the variable: (2x βˆ’ 1)(x + 4) = 0 (Answers on page 174.)
Example 2. Solve βˆ’16t2 βˆ’ 16t + 96 = 0 by factoring.
Solution 2. Factoring the left-hand side, we obtain:
¡
¢
βˆ’16 t2 + t βˆ’ 6 = 0
βˆ’16(t + 3)(t βˆ’ 2) = 0
168
CHAPTER 5. FACTORING
So the solutions to this equation are precisely the values of t for
which the product of βˆ’16, t + 3, and t βˆ’ 2 is equal to zero.
Thus, we conclude that
βˆ’16 = 0
or
t+3=0
or
t βˆ’ 2 = 0.
Since βˆ’16 6= 0, we must have
t+3=0
or
t βˆ’ 2 = 0.
That is,
t = βˆ’3
or
t = 2.
Practice 2. Solve by factoring: 6x + 28x βˆ’ 10 = 0 (Answers on page 174.)
Example 3. Solve each of the following equations for the variable.
(a) x2 βˆ’ 13x = 0
(b) x(x βˆ’ 13) = βˆ’30
(c) 3t(t + 2) + 4 = t + 6
Solution 3. (a) Since the right-hand side is 0, we start by factoring the left-hand side.
x2 βˆ’ 13x = 0
x(x βˆ’ 13) = 0
Using the Zero Product Property, we obtain:
x=0
or
x βˆ’ 13 = 0,
x=0
or
x = 13.
and so
(b) Although the left-hand side is factored, we cannot conclude
that
x = βˆ’30
or
x βˆ’ 13 = βˆ’30.
This is because there is no β€œβˆ’30 Product Property” analogous to the Zero Product Property! It is quite possible to
5.5. SOLVING EQUATIONS BY FACTORING
169
have a product of two numbers equal to βˆ’30 even though
neither number is itself βˆ’30. (Consider βˆ’5 × 6, for example.) Thus, we must first do a little algebra to get 0 on one
side of the equation.
x(x βˆ’ 13) = βˆ’30
x2 βˆ’ 13x = βˆ’30
x2 βˆ’ 13x + 30 = βˆ’30 + 30
x2 βˆ’ 13x + 30 = 0
(x βˆ’ 3)(x βˆ’ 10) = 0
Now using the Zero Product Property, we obtain:
xβˆ’3=0
or
x βˆ’ 10 = 0,
x=3
or
x = 10.
and so
(c) Again, we have a bit of algebra to do before we can factor.
3t(t + 2) + 4 = t + 6
3t2 + 6t + 4 = t + 6
3t2 + 6t + 4 βˆ’ t βˆ’ 6 = t + 6 βˆ’ t βˆ’ 6
3t2 + 5t βˆ’ 2 = 0
(3t βˆ’ 1)(t + 2) = 0
Using the Zero Product Property, we obtain:
3t βˆ’ 1 = 0
or
t + 2 = 0,
and so
3t = 1
1
t=
3
or
t = βˆ’2,
and so
or
t = βˆ’2.
Practice 3. Solve by factoring: 2x(x + 3) = βˆ’4 (Answers on page 174.)
Example 4. Solve: 25y 2 + 20y + 4 = 0
170
CHAPTER 5. FACTORING
Solution 4. We note that the right-hand side is 0 and the lefthand side is a perfect square.
25y 2 + 20y + 4 = 0
(5y)2 + 20y + 22 = 0
(5y + 2)2 = 0
Using the Zero Product Property, we obtain:
5y + 2 = 0
5y = βˆ’2
2
y=βˆ’
5
In this example, we had just one solution since the quadratic
polynomial was the square of a binomial.
Practice 4. Solve by factoring: 9t2 βˆ’ 42t + 49 = 0 (Answers on page 174.)
Example 5. Solve for the variable by factoring: 4x2 = 9
Solution 5. We first need one side to be zero, and then factor
using the difference of squares formula.
4x2 = 9
4x2 βˆ’ 9 = 0
(2x)2 βˆ’ 32 = 0
(2x βˆ’ 3)(2x + 3) = 0
Using the Zero Product Property, we obtain:
2x βˆ’ 3 = 0
or
2x + 3 = 0,
and so
2x = 3
3
x=
2
or
2x = βˆ’3,
3
x=βˆ’ .
2
and so
or
5.5. SOLVING EQUATIONS BY FACTORING
171
Practice 5. Solve for the variable by factoring: 16w2 = 1
(Answers on
page 174.)
Example 6. Solve for the variable.
¡
¢
a. (x βˆ’ 7) 2x2 + 5x + 3 = 0
b. u3 βˆ’ 7u2 = βˆ’12u
Solution 6. We will solve each of these by factoring.
a. This polynomial is already partially factored.
(x βˆ’ 7)(2x2 + 5x + 3) = 0
(x βˆ’ 7)(2x + 3)(x + 1) = 0
And so by Zero Product Property,
xβˆ’7=0
or
2x + 3 = 0
or
x+1=0
and so
x=7
or
or
x = βˆ’1
and so
x=7
or
2x = βˆ’3
3
x=βˆ’
2
or
x = βˆ’1.
b. Here we will first factor out the GCF.
u3 βˆ’ 7u2 = βˆ’12u
u3 βˆ’ 7u2 + 12u = 0
u(u2 βˆ’ 7u + 12) = 0
u(u βˆ’ 3)(u βˆ’ 4) = 0
And so by Zero Product Property,
u=0
or
uβˆ’3=0
or
uβˆ’4=0
u=0
or
u=3
or
u = 4.
and so
172
CHAPTER 5. FACTORING
Practice 6. Solve for the variable by factoring. (Answers on page 174.)
a. (4t βˆ’ 1)(t2 + 6t βˆ’ 16) = 0
b. 5y 4 = 7y 3 βˆ’ 2y 2
Example 7. Solve for x:
4x3 + 6x2 βˆ’ 40x = 0
Solution 7. Since one side is already zero, we factor to obtain
4x3 + 6x2 βˆ’ 40x = 0
2x(2x2 + 3x βˆ’ 20) = 0
2x(2x βˆ’ 5)(x + 4) = 0
So, the Zero Product Property gives us
2x = 0
or
2x βˆ’ 5 = 0
or
x+4=0
x=0
or
2x = 5
or
x = βˆ’4
x=0
or
x=
5
2
or
x = βˆ’4
Practice 7. Solve by factoring: 12x3 +12x2 βˆ’9x = 0 (Answers on page 174.)
Example 8. Solve for x:
2x(x βˆ’ 3) + 3(x βˆ’ 5) = x2 + 3
Solution 8. We must first remove all parentheses and then set
the equation equal to zero.
5.5. SOLVING EQUATIONS BY FACTORING
173
2x(x βˆ’ 3) + 3(x βˆ’ 5) = x2 + 3
2x2 βˆ’ 6x + 3x βˆ’ 15 = x2 + 3
2x2 βˆ’ 3x βˆ’ 15 = x2 + 3
x2 βˆ’ 3x βˆ’ 18 = 0
(x βˆ’ 6)(x + 3) = 0
Using the Zero Product Property, we obtain
xβˆ’6=0
or
x+3=0
x=6
or
x = βˆ’3
Practice 8. Solve by factoring: 3x(x + 1) + 2(x + 2) = 2x(x + 5) (Answers
on the next page.)
Example 9. Solve for x:
5x3 βˆ’ 2x2 βˆ’ 20x + 8 = 0
Solution 9. Since the left hand side of the equation has four
terms, we will factor by grouping.
3
5x
βˆ’ 2x}2 βˆ’ 20x
| {z
| {z+ 8} = 0
x2 (5x βˆ’ 2) βˆ’ 4(5x βˆ’ 2) = 0
(5x βˆ’ 2)(x2 βˆ’ 4) = 0
(5x βˆ’ 2)(x βˆ’ 2)(x + 2) = 0
174
CHAPTER 5. FACTORING
Using the Zero Product Property, we get
5x βˆ’ 2 = 0
or
xβˆ’2=0
or
x+2=0
5x = 2
or
x=2
or
x = βˆ’2
2
5
or
x=2
or
x = βˆ’2
x=
Practice 9. Solve by factoring: 36x3 βˆ’ 9x2 βˆ’ 4x + 1 = 0 (Answers below.)
ANSWERS TO PRACTICE PROBLEMS
1. x = βˆ’4,
2. x = βˆ’5,
1
5. w = ±
2
7. x = βˆ’
4
3
2
, 0,
1
2
1
3
6.
3. x = βˆ’2, βˆ’1
4. t =
1
7
(a) t = βˆ’8,
(b) y = 0,
3
2
5
1
4
,2
,1
8. x = 1, 4
9. x = ±
1 1
,
3 4
Section 5.5 EXERCISES:
(Answers are found on page 249.)
Solve the following equations.
1. (x βˆ’ 3)(x + 2) = 0
9. 3(10x + 1)(7x βˆ’ 2) = 0
2. (x βˆ’ 7)(x βˆ’ 2) = 0
10. x2 + 3x βˆ’ 4 = 0
3. (x + 3)(x + 4) = 0
11. x2 βˆ’ 7x + 12 = 0
4. x(x βˆ’ 6) = 0
12. x2 + 10x + 25 = 0
5. (x + 4)(x + 4) = 0
13. 3x2 + 18x + 27 = 0
6. 5(x + 1)(x βˆ’ 4) = 0
14. 5x2 + 6x + 1 = 0
7. (2x + 1)(x + 6) = 0
15. 4x2 βˆ’ 8x + 3 = 0
8. (3x βˆ’ 2)(4x βˆ’ 1) = 0
16. x(x βˆ’ 3) = 4
5.5. SOLVING EQUATIONS BY FACTORING
175
17. (x + 1)(x + 2) = 12
37.
12x2 βˆ’ 14x βˆ’ 10 = 0
18. x2 + 10x = βˆ’25
38.
5x2 βˆ’ 7x = 0
19. x2 + 5x = x + 8
39.
x2 βˆ’ 8x βˆ’ 9 = 0
20. 3x2 + 5x = 2x2 βˆ’ 3x
40.
(x βˆ’ 4)(x + 2) = 4x(x βˆ’ 3)
21. 5x2 + 3x + 9 = 4x2 + 3x βˆ’ 7
41.
16x2 βˆ’ 25 = 0
42.
2x(3xβˆ’1)+2 = 2x(x+1)+5
43.
2x3 βˆ’ 9x2 βˆ’ 5x = 0
44.
18x3 + 9x2 βˆ’ 2x βˆ’ 1 = 0
45.
9x4 βˆ’ 13x2 + 4 = 0
46.
9x3 βˆ’ 4x2 βˆ’ 9x + 4 = 0
47.
10x3 + 17x2 + 3x = 0
48.
4x4 βˆ’ 13x2 + 9 = 0
49.
24x3 βˆ’ 4x2 βˆ’ 6x + 1 = 0
22. 2x3 βˆ’ 2x2 βˆ’ 12x = 0
3
2
23. 2x βˆ’ 18x + 36x = 0
4
24. x βˆ’ 81 = 0
25. x4 βˆ’ 4x2 + 3 = 0
26.
x2
27.
10x2
28.
4x(x βˆ’ 2) = 3(x βˆ’ 2)
29.
2x2 βˆ’ 5x βˆ’ 3 = 0
30.
x2 + 6x + 9 = 0
31.
3x2 = 2x
50.
16x4 βˆ’ 17x2 + 1 = 0
32.
4x2 βˆ’ 2x βˆ’ 12 = 0
51.
2x3 βˆ’ 14x2 + 12x = 0
33.
x(x + 7) = 5(x + 3)
52.
3x3 + 9x2 βˆ’ 30x = 0
34.
4x2 βˆ’ 2x βˆ’ 20 = 0
53.
4x3 + 12x2 βˆ’ 9x βˆ’ 27 = 0
35.
(2x + 1)(x βˆ’ 3) = βˆ’5
54.
4x4 βˆ’ 17x2 + 4 = 0
36.
3x(3x + 1) βˆ’ 2 = x(3x + 4)
55.
8x3 βˆ’ 20x2 + 12x = 0
βˆ’ 12x + 11 = 0
βˆ’ 15x βˆ’ 10 = 0
Find the value(s) for b so that the following equations have integer solutions.
56. x2 + bx + 5 = 0
59. x2 + bx βˆ’ 9 = 0
57. x2 + bx βˆ’ 5 = 0
60. x2 + bx + 12 = 0
58. x2 + bx + 9 = 0
61. x2 + bx βˆ’ 8 = 0
176
62. x(x + b) = βˆ’10
CHAPTER 5. FACTORING
64. y 2 + by = 18
63. y 2 + by = 17
Find the value(s) for b so that the given value a is a solution of the the
equation.
65. x2 + bx + 16 = 0; a = 2
67. x2 + bx βˆ’ 16 = 0; a = βˆ’1
66. x2 + bx + 16 = 0; a = 4
68. x2 + bx βˆ’ 16 = 0; a = 4
EXPLORATION: FACTORS AND INTERCEPTS
177
Exploration: Factors and x-Intercepts
Note to the Instructor: Students will need a simple graphing device
with zoom and trace functionality. This could be a graphing calculator or a
graphing application on the Web such as gcalc.net. You will need to spend
a few minutes demonstrating how to graph functions and how to zoom and
trace on the graph. Ideally the material in this exploration will be covered
in two lab days: one day for linear functions and one day for quadratic
functions. You might wish to have students work together in pairs or small
groups. Because this is a discovery activity, no answers are provided in the
text. Therefore, it is crucial to provide ample assistance during the activity
and feedback afterward.
In this exploration you will be lead to discover one of the fundamental
ideas of algebra: the connection between linear factors, x-intercepts, and
solutions of polynomial equations. This is a central idea to which you will
return many times throughout this course and as you continue your algebra
studies. Therefore, it is crucial for you to actually do all of the activities
in this section, participate in class discussions, and get feedback from your
instructor.
x-intercepts of linear polynomials
Recall from Core Math I that the equation y = mx + b represents a line
with slope m and y-intercept (0, b). (Click here to review this material from
Core Math I) Here, we want to switch our focus to x-intercepts, that is, the
points where a graph crosses the x-axis. Remember that the y-coordinate
of an x-intercept is 0.
Each row in the following table represents a linear function. Use your
graphing device and algebra skills from Core Math I to fill in the blanks in
each row. As you do so, observe the patterns that emerge. These patterns
will help you to fill in the remaining rows. Your instructor may require you
to turn in your work, written neatly on a separate piece of paper.
178
CHAPTER 5. FACTORING
y = mx + b
form
y = m(x βˆ’ c)
form
Slope
x-intercept
1
y =xβˆ’2
y = 1(x βˆ’ 2)
2
y = x βˆ’ 13
y = 1(x βˆ’ 13)
3
1
(35, 0)
4
1
5
y =x+8
6
y =x+4
1
8
1
y = 3x βˆ’ 18
10
y = 2x βˆ’ 1
5
12
βˆ’1
y = 4x + 12
14
y = βˆ’7x βˆ’ 14
(βˆ’12, 0)
x = βˆ’10
y = 3(x βˆ’ 6)
11
13
x=7
y = 1(x βˆ’ (βˆ’8))
7
9
Solution
of
equation y = 0
(2, 0)
x=1
y = 4(x βˆ’ (βˆ’3))
15
βˆ’3
(βˆ’10, 0)
16
9
x = βˆ’3
17
m
x=c
EXPLORATION: FACTORS AND INTERCEPTS
179
x-intercepts of quadratic polynomials
From the above exploration, we know that the graph of y = x βˆ’ 2 is a line
with slope 1 and x-intercept 2 and the graph of y = x βˆ’ 5 is a line with
slope 1 and x-intercept 5. (Graph these on your graphing device to confirm.)
What happens if we create a new polynomial function by taking the product
of x βˆ’ 2 and x βˆ’ 5, that is y = (x βˆ’ 2)(x βˆ’ 5)? Do you have a guess as to
what the shape of this graph is? What do you think the x-intercepts are?
Graph y = (x βˆ’ 2)(x βˆ’ 5) to check your guesses. When you have found the
x-intercepts correctly, write them in the first row of the table on the next
page.
Next, we wish to write y = (x βˆ’ 2)(x βˆ’ 5) in expanded form. To do this,
perform the multiplication and combine like terms. Write the result in the
table. Also write the leading coefficient of the polynomial in the table. Is
it obvious from the expanded form of the equation what the x-intercepts
are? If you wish to determine the x-intercepts, would you rather have the
factored form y = (x βˆ’ 2)(x βˆ’ 5) or the expanded form y = x2 βˆ’ 7x + 10?
To finish this example, we wish to solve the equation (x βˆ’ 2)(x βˆ’ 5) = 0
for x. If you have not yet learned an algebraic method to do this, you can use
your grapher instead as follows. Remember that the expression (xβˆ’2)(xβˆ’5)
is equal to y. So we want to know what x is when y = 0. Trace along your
graph until the y-coordinate is 0. What is the x-coordinate? (Be careful:
there might be more than one answer!) Write this in the table and you have
filled in the first row.
Each row in the table represents a quadratic function. As you fill in the
blanks in each row, observe the patterns that emerge. These patterns will
help you to fill in the remaining rows. Your instructor may require you to
turn in your work, written neatly on a separate piece of paper.
180
CHAPTER 5. FACTORING
Expanded Form
1
2
Factored Form
Lead
Coeff
x-int(s)
Solutions
of
equation y = 0
y = (x βˆ’ 2)(x βˆ’ 5)
y = x2 βˆ’ 3x + 2
y = 1(x βˆ’ 2)(x βˆ’ 1)
3
1
4
1
5
y = x2 + 5x βˆ’ 24
6
y = x2 + 12x + 35
1
8
1
y = 2x2 βˆ’ 12x βˆ’ 14
10
y = 5x2 + 25x + 30
βˆ’3
12
10
y = 2x2 + 5x βˆ’ 3
14
y = 5x2 + 12x + 4
(βˆ’20, 0), (4, 0)
x = βˆ’5, βˆ’4
y = 2(x + 1)(x βˆ’ 7)
11
13
x = 7, 2
y = 1(x + 8)(x βˆ’ 3)
7
9
(5, 0), (3, 0)
(βˆ’1, 0), (1, 0)
x = 2, 12
y = 2(x βˆ’ 12 )(x + 3)
(βˆ’2, 0), ( 32 , 0)
15
3
16
2
x = βˆ’ 27 , 1
17
a
x = d1 , d2
Chapter 6
Radicals
6.1
Introduction to Radicals
Recall the following definition for square roots which was discussed in Core
Math I.
Square roots of a: The square root of a, denoted
whose square is a. In other words,
√
a = b means b2 = a.
√
a, is the number
Remember that the radical sign implies the principal, or nonnegative,
square root.
Example 1. Find each square root.
√
121
r
49
(b)
64
(a)
√
βˆ’16
(e)
√
x4
√
(d) βˆ’ 16
(f )
√
4x8
(c)
√
Solution 1. (a)
121 = 11 since (11)2 = 121.
r
µ ¶2
49
7
7
49
(b)
= since
=
64
8
8
64
√
(c)
βˆ’16 is not a real number. This is because there is no real
number a such that a2 = βˆ’4.
181
182
CHAPTER 6. RADICALS
√
√
√
√
(d) βˆ’ 16 = βˆ’4 since βˆ’ 16 = βˆ’1 · 16 and 16 = 4 since
42 = 16.
√
(e)
x4 = x2 since (x2 )2 = x4 .
√
(f )
4x8 = 2x4 since (2x4 )2 = 4x8 .
Practice 1. Find each square root. (Answers on page 184 )
r
(a)
25
81
(b)
√
9x12
(c)
√
βˆ’36
We now generalize our square root definition to general n-th roots.
√
n-th roots of a: The n-th root of a, denoted n a, is a number whose
n-th power equals a. In other words,
√
n
a = b means bn = a.
The number n is called the index and a is called the radicand.
Remember that the index two is generally omitted for square roots. As with
√
square roots, if n is even, the radical n represents the nonnegative n-th
root.
Example 2. Find each root.
√
3
27
r
8
(b) 3
125
√
4
βˆ’81
r
16
(d) 4
81
(a)
(c)
Solution 2. (a)
r
(b)
3
√
3
27 = 3 since 33 = 27.
8
2
= since
125
5
µ ¶3
2
8
=
.
5
125
(e)
√
5
βˆ’32
(f )
√
6
x12
6.1. INTRODUCTION TO RADICALS
(c)
√
4
βˆ’81 is not a real number since there does not exist a real
number a such that a4 = βˆ’81.
r
(d)
183
4
16
2
= since
81
3
µ ¶4
2
16
= .
3
81
(e)
√
5
βˆ’32 = βˆ’2 since (βˆ’2)5 = βˆ’32.
(f )
√
6
x12 = x2 since (x2 )6 = x12 .
Practice 2. Find each root. (Answers on page 184 )
r
√
√
4
64
(b)
16x8
(c) 6 64
3
(a)
βˆ’
27
When working with variables, it is not clear whether the variable represents a positive or negative real number. Because when n is even the radical
√
n
represents the nonnegative n-th root, when variables are present it may
be necessary to include absolute values in the answer. We state this rule
next.
√
Evaluating n an :
√
β€’ If n is even, then n an = |a|.
√
β€’ If n is odd, then n an = a.
Note that the absolute value sign is not required unless the index is even.
Example 3. Find each root.
p
(a) 4 (βˆ’2)4
p
(b) 3 (βˆ’6)3
√
x10
√
5
(d) x15
(c)
p
Solution 3. (a) 4 (βˆ’2)4 = | βˆ’ 2| = 2.
p
(b) 3 (βˆ’6)3 = βˆ’6. No absolute value sign is necessary since
the index is odd.
184
CHAPTER 6. RADICALS
√
x10 = |x5 |. The absolute value sign is needed because it is
not clear if x5 is positive.
√
5
(d) x15 = x3 . No absolute value sign is necessary since the
index is odd.
(c)
Practice 3. Find each root. (Answer below.)
√
p
3
(a)
x9
(b) 4 (βˆ’8)4
(c)
ANSWERS TO PRACTICE PROBLEMS
1.
5
(a)
2.
9
(c)
2
(a)
x3
6
(b)
3x
(c)
not a real number
(a)
βˆ’4
3
(b)
8
(b)
2
(c)
|x3 |
3.
2x
SECTION 6.1 EXERCISES
(Answers are found on page 250.)
Evaluate.
1.
√
81
7.
√
2. βˆ’ 121
r
3.
9
16
√
4. βˆ’ 400
r
5. βˆ’
6.
100
49
√
βˆ’4
√
3
βˆ’8
√
8. βˆ’ 4 16
9.
√
3
1000
r
10.
3
βˆ’
125
27
√
11. βˆ’ 3 64
12.
√
4
βˆ’256
√
6
x18
6.1. INTRODUCTION TO RADICALS
13.
28.
p
4
1
βˆ’
8
29.
√
4 4
7
r
sµ ¶
3 2
βˆ’
30.
5
√
5
βˆ’32
r
14.
3
15. βˆ’
16.
1
4
√
3
125
√
17. βˆ’ 3 βˆ’64
r
18.
185
27
8
3
(βˆ’10)4
31.
p
7
32.
√
x14
33.
√
x20
(βˆ’3)7
19.
√
0.64
34.
√
4
x4
20.
√
0.36
35.
√
4
x16
21.
√
0.25
36.
√
100x4
22.
√
3
0.008
37.
p
49y 6
23.
√
3
βˆ’0.027
38.
√
3
216x9
24.
p
(βˆ’5)2
39.
√
64x12
25.
√
72
40.
√
3
64x12
26.
p
3
41.
√
4
81x12
27.
√
3 3
9
42.
√
81x12
(βˆ’8)3
186
CHAPTER 6. RADICALS
43.
√
16x16
47.
44.
√
4
16x16
48.
45.
√
3
64x9
49.
46.
p
4
16x8 y 12
50.
p
121x6 y 2
p
3
p
5
p
125x15 y 21
βˆ’243x10 y 25
36x4 y 10
6.2. SIMPLIFYING RADICAL EXPRESSIONS
6.2
187
Simplifying Radical Expressions
Before we can perform any operations with radical expressions, we need to
be able to simplify radicals. We know that
√ √
√
√
4 9 = 2 · 3 = 6 and
4 · 9 = 36 = 6.
√ √
√
Since both cases result in 6, we can conclude that 4 9 = 4 · 9. Furthermore, we know that
r
√
3
3
27
3
3 27
=
and √
= .
3
8
2
2
8
r
√
3
3
27
3 27
Since both cases result if , we can again conclude that
= √
. It
3
2
8
8
is left to the reader to examine examples with other roots. The two results
are stated below in general.
√
√
Product rule for radicals: If n a and n b are real numbers and n
is a positive integer,
√
√ √
n
n
ab = n a · b
In other words, the radical of a product is the product of the radicals.
√
√
Quotient rule for radicals: If n a and n b are real numbers and
n is a positive integer,
r
√
n
a
a
n
= √
.
n
b
b
In other words, the radical of a quotient is the quotient of the radicals.
We will use the above rules to simplify a radical. Unless otherwise stated,
assume that all variables represent positive real numbers. This means that
there is no need to include the absolute value sign in the answer.
Example 1. Simplify. Assume each variable represents a positive real number.
r
p
(a)
4x6 y 8
x4
(b)
16
188
CHAPTER 6. RADICALS
(c)
p
3
βˆ’27x3 y 9 z 6
s
(d)
3
64x3
y6
Solution 1. (a) Using the product rule for radicals, we have
p
√ √ p
4x6 y 8 = 4 x6 y 8
= 2x3 y 4
(b) Using the quotient rule for radicals, we obtain
r
√
x4
x4
x2
=√ =
16
4
16
(c) Using the product rule for radicals, we have
p
p √
√
√
3
3
3
βˆ’27x3 y 9 z 6 = 3 βˆ’27 x3 3 y 9 z 6
= βˆ’3xy 3 z 2
(d) Using the quotient rule and then the product rule for radicals, we get
s
√
3
3
64x3
3 64x
p
=
3
y6
y6
√
√
3
3
64 x3
= p
3
y6
4x
= 2
y
Practice 1. Simplify. Assume all variables represent positive real numbers.
s
p
√
4
12
(a)
49x4 x10
(c)
16x4 y 12 z 16
8x
(b) 3 15
y
(Answers on page 191 )
Not all the radical problems we encounter will work out to be perfect
n-th roots. As a result, we need to be able to simplify any radical. The
definition of a radical in simplified form follows.
6.2. SIMPLIFYING RADICAL EXPRESSIONS
189
Simplifying radicals: A radical is in simplest form when the following conditions are satisfied:
β€’ There is no fraction under the radical.
β€’ There is no radical in the denominator.
β€’ There is no common factor, other than 1, between the exponents on factors under the radical and the index.
β€’ The quantity under the radical has no factor raised to a power
greater than or equal to the index.
Example 2. Simplify
real number
√
12x9 . Assume the variable represents a positive
Solution 2. Since both 12 and x9 are not perfect squares, we
need to rewrite each as a product of two terms: one a perfect
square and one that is not only not a perfect square, but contains
no factors that are perfect squares. Hence, 12 = 4 · 3 and x9 =
x8 · x. Using the product rule for radicals, we get
√
√
√
√
√
12x9 = 4 · 3 · x8 · x = 4x8 3x = 2x4 3x.
Notice that we placed each term that is a perfect square in the
first radical and everything that would be left over in the second
radical.
√
Practice 2. Simplify
32x7 . Assume the variable represents a positive
real number. (Answers on page 191 )
Example 3. Simplify. Assume all variables represent positive real numbers.
p
32x5 y 11
p
(b) 75x13 y 7 z 4
(a)
p
147x6 y 3 z 4
p
(d) 128x7 y 2 z 19
(c)
p
3
54x3 y 5 z 4
p
(f ) 4 32x5 y 7 z 9
(e)
190
CHAPTER 6. RADICALS
Solution 3. (a)
p
p
16 · 2 · x4 · x · y 10 · y
p
p
= 16x4 y 10 2xy
p
= 4x2 y 5 2xy
32x5 y 11 =
(b)
p
p
25 · 3 · x12 · x · y 6 · y · z 4
p
p
= 25x12 y 6 z 4 3xy
p
= 5x6 y 3 z 2 3xy
75x13 y 7 z 4 =
(c)
p
147x6 y 3 z 4 =
p
49 · 3 · x6 · y 2 · y · z 4
p
= 49x6 y 2 z 4 3y
p
= 7x3 yz 2 3y
p
(d)
p
p
128x7 y 2 z 19 = 64 · 2 · x6 · x · y 2 · z 18 · z
p
√
= 64x6 y 2 z 18 2xz
√
= 8x3 yz 9 2xz
(e)
p
p
3
54x3 y 5 z 4 = 3 27 · 2 · x3 · y 3 · y 2 · z 3 · z
p
p
= 3 27x3 y 3 z 3 3 2y 2 z
p
= 3xyz 3 2y 2 z
(f )
p
4
p
4
16 · 2 · x4 · x · y 4 · y 3 · z 8 · z
p
p
= 4 16x4 y 4 z 8 4 2xy 3 z
p
= 2xyz 2 4 2xy 3 z
32x5 y 7 z 9 =
6.2. SIMPLIFYING RADICAL EXPRESSIONS
191
Practice 3. Simplify. Assume the variable represents a positive real number. (Answers on page 191 )
p
p
p
(a) 72x9 y 3
(b) 3 108x5 y 9 z 4
(c) 4 162x4 y 9 z 5
For a radical to be in simplest form, there can be no common factors,
other than p
one, between the exponents of factors under the radical and the
index. So, 8 x6 y 2 would not be in simplest form since the exponents on the
factors of the radicand and the index have a common factor of 2. In order
to simplify this, we need to use the Index Rule for radicals.
Index Rule for Radicals: If k, m and n are positive integers, then
√
√
kn
akm = n am .
Example 4. Simplify
p
8
x6 y 2 .
Solution 4. Note that the exponents on the factors under the
radical and the index have a common factor of 2. Therefore,
using the Index Rule for radicals, we get
p
p
8
x6 y 2 = 4·2 x3·2 y 1·2
p
= 4 x3 y
Practice 4. Simplify
p
12
x3 y 6 (Answer below )
ANSWERS TO PRACTICE PROBLEMS
1.
(a) 7x2 y 5
(a) 6x4 y
3.
4
(b)
2x
(b) 3xy 3 z
y5
√
2x
√
3
4x2 z
√
(c) 3xy 2 z 4 2yz
(c) 2xy 3 z 4
2. 4x3
√
2xy
4.
p
4
xy 2
192
CHAPTER 6. RADICALS
SECTION 6.2 EXERCISES
(Answers are found on page 251.)
Simplify. Assume all variables represent positive real numbers.
p
√
√
1. 8x3
15. 32a5 b15
29. 3 40x4 y 8
p
2.
√
162x2
16.
40x11 y 7
30.
3.
√
3
βˆ’24x6
√
17. 2 16a2 b3
31.
4.
√
72x3
√
18. 4 20a4 b7
32.
5.
√
4
x9
p
19. 3x 12x2 y 7
33.
6.
√
4
x2
20. 4y
7.
√
8
x4
21. 2x2
8.
√
x4
12
22. 3y
p
18x5 y 4
p
p
34.
8x2 y 3
35.
28x3 y 4
36.
9.
p
6
x4 y 8
p
23. 5 18xy 3
37.
10.
p
4
x2 y 8
p
24. 2x 48x5 y 2
38.
11.
p
6
x18 y 4
25.
p
3
16x4 y 9
39.
12.
p
4
16x2 y 10
26.
p
3
54x5 y 7
40.
13.
√
24x2
27.
p
3
8x6 y 4
41.
14.
p
18x5 y 7
28.
p
3
16x5 y 7
42.
p
243x9 y 25 z 18
p
3
135x9 y 14 z 25
p
4
324x13 y 11 z 22
p
252x10 y 13 z 21
p
3
108x7 y 9 z 14
p
147x10 y 6 z 3
p
320x5 y 9 z 12
p
4
32x8 y 11 z 12
p
192x8 y 15 z 6
p
3
βˆ’108x15 y 7 z 11
p
4
48x8 y 14 z 11
p
5
128x19 y 23 z 10
p
3
βˆ’81x4 y 10
6.2. SIMPLIFYING RADICAL EXPRESSIONS
193
194
6.3
CHAPTER 6. RADICALS
Addition and Subtraction of Radicals
Now that we know how to simplify radicals we turn our attention to adding
and subtracting radicals. Before we do this, recall that we performed addition and subtraction of algebraic expressions in Section 4.2 by combining
like terms. For example, 3x + 5x = (3 + 5)x = 8x. The same procedure
can be used for the addition and subtraction of radicals. When adding and
subtracting radicals, you may only combine radicals with the same index
and exactly
For ex√ the√same quantity
√ under
√ the radical (same
√ radicand).
√
ample, 3 7 + 5 7 = (3 + 5) 7 = 8 7. However, 3 7√and 5 3 7√cannot be
combined since their indices are different. Likewise, 3 7 and 5 3 cannot
be combined because the quantity under the radical is different.
√
√
√
Example 1. Simplify: 7 11 + 5 11 βˆ’ 4 11
Solution 1. Since all the radical parts are identical, we have
√
√
√
√
7 11 + 5 11 βˆ’ 4 11 = (7 + 5 βˆ’ 4) 11
√
= 8 11
√
√
√
Practice 1. Simplify: 8 3 βˆ’ 3 3 + 2 3 (Answer on page 198)
Remember you cannot add two radicals together if they have different
radical parts. Although we saw that there is a product and quotient rule for
radicals, as stated in Section 6.2, there is no addition property of radicals.
In other words,
√
√
√
n
n
n
a + b 6= a + b.
√
√
√
Example 2. Simplify: 3 5 βˆ’ 7 2 + 6 2
Solution 2. Since only the last two terms have identical radical
parts, these are the only two terms that we can combine. Hence,
√
√
√
√
√
3 5 βˆ’ 7 2 + 6 2 = 3 5 + (βˆ’7 + 6) 2
√
√
=3 5βˆ’ 2
6.3. ADDITION AND SUBTRACTION OF RADICALS
195
√
√
√
Practice 2. Simplify: 7 6 + 9 2 βˆ’ 11 6 (Answer on page 198)
Of course, not every problem will have the same radical parts. What
happens if the radicands do not match? The next example illustrates that
some simplification might be needed before we can combine the radicals.
√
√
Example 3. Simplify: 3 48 + 5 27
Solution 3.
√
√
√
√
3 48 + 5 27 = 3 16 · 3 + 5 9 · 3
√ √
√ √
= 3 16 3 + 5 9 3
√
√
=3·4 3+5·3 3
√
√
= 12 3 + 15 3
√
= 27 3
√
√
Practice 3. Simplify: 4 32 βˆ’ 5 50 (Answer on page 198)
√
√
√
Example 4. Simplify: 3 12 βˆ’ 7 72 + 5 50
Solution 4.
√
√
√
√
√
√
3 12 βˆ’ 7 72 + 5 50 = 3 4 · 3 βˆ’ 7 36 · 2 + 5 25 · 2
√ √
√ √
√ √
= 3 4 3 βˆ’ 7 36 2 + 5 25 2
√
√
√
=3·2 3βˆ’7·6 2+5·5 2
√
√
√
= 6 3 βˆ’ 42 2 + 25 2
√
√
= 6 3 βˆ’ 17 2
196
CHAPTER 6. RADICALS
√
√
√
Practice 4. Simplify: βˆ’2 63 + 2 28 + 3 7 (Answer on page 198)
r
Example 5. Simplify:
3
+
16
r
75
4
Solution 5. Applying the Quotient rule for radicals, we have
r
3
+
16
r
Practice 5. Simplify:
r
75
+
64
√
√
75
3
75
=√ + √
4
16
4
√
√
3
25 · 3
=
+
4
2
√ √
√
3
25 3
+
=
4
2
√
√
3 5 3
+
=
4
2
√
√
3 10 3
=
+
4
4
√
11 3
=
4
r
3
(Answer on page 198)
16
When adding or subtracting radicals that contain variables, remember
that in order to have like terms, the radical parts and the variable parts
must be identical.
√
√
3
3
Example 6. Simplify: 2x 54x4 βˆ’ 5 16x7
6.3. ADDITION AND SUBTRACTION OF RADICALS
197
Solution 6.
√
√
√
√
3
3
3
3
2x 54x4 βˆ’ 5 16x7 = 2x 27 · 2x3 x βˆ’ 5 8 · 2x6 x
√
√
√
√
3
3
3
3
= 2x 27x3 2x βˆ’ 5 8x6 2x
√
√
3
3
= 2x · 3x 2x βˆ’ 5 · 2x2 2x
√
√
3
3
= 6x2 2x βˆ’ 10x2 2x
√
3
= βˆ’4x2 2x
Practice 6. Simplify:
p
p
√
5x 3 54x3 y βˆ’ 2 3 250x6 y + 9x2 3 128y (Answer on
page 198)
√
√
√
50 3 8
3
Example 7. Simplify:
+
+√
3
2
4
Solution 7.
√
√
√
√
√
√
50 3 8
3
25 · 2 3 4 · 2
3
+
+√ =
+
+
3
2
3
2
2
4
√ √
√ √
√
25 2 3 4 2
3
=
+
+
3
2
2
√
√
√
3
5 2 6 2
+
+
=
3
2
2
√
√
√
5 2
3
+3 2+
=
3
2
√
√
√
5 2 9 2
3
=
+
+
3
3
2
√
√
14 2
3
=
+
3
2
√
√
√
2 3 3 27
3
Practice 7. Simplify:
βˆ’
+
(Answer on page 198)
4
2
3
198
CHAPTER 6. RADICALS
ANSWERS TO PRACTICE PROBLEMS
√
1. 7 3
5.
√
√
2. βˆ’4 6 + 9 2
8
√
6. 41x2 3 2y
√
3. βˆ’9 2
4.
√
7 3
√
7
7. βˆ’
√
11 3
3
SECTION 6.3 EXERCISES
(Answers are found on page 252.)
Perform the indicated operations and simplify. Assume all variables represent positive real numbers.
√
√
√
√
√
1. 12 + 75
10. 2x2 + 6 32x2 βˆ’ 2 x2
2.
√
√
3
16 + 3 54
√
√
3. 27 βˆ’ 192
√
√
4. 3 20 + 5 45
√
√
√
√
5. 4 3 16 βˆ’ 2 3 2 + 2 2 βˆ’ 2 16
r
6.
8
βˆ’
49
r
8.
12.
p
12y 3 +
p
√
27y 3 + 3y 3y
√
√
√
3
28x2 βˆ’ 63x2 + 56x3
√
√
13. 5 4x βˆ’ 3 9x
√
√
14. 3 3x2 βˆ’ 5 27x2
√
√
15. βˆ’2 8x2 + 5 32x2
50
9
√
√
7. 5 27x βˆ’ 75x
r
11.
r
3
3
+4
25
100
√
√
9. 3 8x2 βˆ’ 50x2
√
√
16. 5 18 βˆ’ 2 75
√
√
17. 3x 12x βˆ’ 5 27x3
√
√
√
18. 8 8 βˆ’ 4 32 βˆ’ 9 50
√
√
√
19. βˆ’2 3 + 5 27 βˆ’ 4 45
6.3. ADDITION AND SUBTRACTION OF RADICALS
20.
√
√
√
25x βˆ’ 9x + 16x
199
√
√
√
36. βˆ’2 4 48 + 3 4 243 + 5 4 3
√
√
√
21. 3 3x + 27x βˆ’ 8 75x
√
√
√
37. 3x 75x3 + 7 12x5 βˆ’ x 27x
p
√
√
22. x 3y 2 βˆ’ 2y 12x2 + xy 3
38.
23.
√
√
√
18 + 8 βˆ’ 32
24.
√
√
√
√
48 + 20 βˆ’ 27 + 2 20
25.
√
√
√
18x3 + x 32x + 50x2
√
√
26. 25 + 12
√
√
27. 49 βˆ’ 90
√
√
√
28. 8 18 βˆ’ 2 50 βˆ’ 3 98
√
√
√
29. 3 375 + 2 3 81 βˆ’ 7 3 24
r
30.
5
βˆ’
4
r
45
16
√
√
31. 5 3 16 + 9 3 128
√
√
√
32. 7 8 + 9 18 βˆ’ 4 162
√
√
√
3
18x
√ βˆ’ 2 12x + 32x +
3x 27x
√
√
√
39. 4 3 16 βˆ’ 5 3 54 + 7 3 128
√
√
40. 3 48 βˆ’ 2 300
√
√
√
41. βˆ’2 63 + 3 7 + 4 28
r
42.
3
+
16
r
75
4
√
√
√
43. 3 48 + 5 27 βˆ’ 6 108
√
√
44. 5 3 16 βˆ’ 7 3 54
√
√
√
45. 12 8 βˆ’ 5 18 + 3 162
√
√
√
46. 2 3 16 βˆ’ 5 3 54 + 3 3 128
√
√
√
47. 4 18x βˆ’ 72x + 50x
r
8
βˆ’
81
r
32
9
√
√
√
33. βˆ’5 63x3 + 9x 28x + 6 7x3
48.
√
√
√
34. 3 8 βˆ’ 6 50 + 2 200
√
√
√
49. βˆ’8 27 + 7 48 βˆ’ 3 108
√
√
√
35. 5 3 16 + 9 3 54 βˆ’ 7 3 2
√
√
√
50. 2 18 βˆ’ 5 50 + 3 32
200
6.4
CHAPTER 6. RADICALS
Multiplication of Radicals
We can multiply radicals using many of the same properties and techniques
that we used in Section 4.2 to multiply polynomials.
¡ √ ¢¡ √ ¢
Example 1. Simplify: 3 3 2 6
Solution 1.
³ √ ´³ √ ´
³βˆš √ ´
3 3 2 6 = (3 · 2)
3 6
√
= 6 18
√ √
=6 9 2
√
=6·3 2
√
= 18 2
Practice 1. Simplify:
¡ √ ¢¡ √ ¢
5 2 2 6 (Answer on page 204)
Be careful not to combine factors on the outside of √
the radical
√ with the
numbers that appear in the radicand. For example, 3x 2x 6= 6x2 .
√ ¡ √
¢
Example 2. Simplify: 3 2 5 3 βˆ’ 4
Solution 2. Using the distributive property, we obtain
´ ³ √ ´³ √ ´ ³ √ ´
√ ³ √
3 2 5 3 βˆ’ 4 = 3 2 5 3 βˆ’ 3 2 (4)
√
√
= 15 6 βˆ’ 12 2
Since the radicands are different, they cannot be combined.
√ ¡
√ ¢
Practice 2. Simplify: 4 3 2 + 6 5 (Answer on page 204)
6.4. MULTIPLICATION OF RADICALS
201
√ ¡ √
√ ¢
Example 3. Simplify: 5x 3 9 2 3 3 βˆ’ 6 3 9
Solution 3. Once again, using the distributive property, we obtain
³ √
´
√
√
√
√
3
3
3
3
3
5x 9 2 3 βˆ’ 6 9 = 10x 27 βˆ’ 30x 81
√
3
= 10x · 3 βˆ’ 30x 27 · 3
√
√
3
3
= 30x βˆ’ 30x 27 3
√
3
= 30x βˆ’ 90x 3
√ ¡ √
√ ¢
Practice 3. Simplify: 3x 3 2 5 3 4 + 7 3 2 (Answer on page 204)
Next, we illustrate multiplying a radical expression containing two terms
with another radical expression containing two terms. As in Section 4.2, we
use the distributive property twice.
¡ √
¢¡ √
¢
Example 4. Simplify: 3 5 + 2 2 5 βˆ’ 7
Solution 4.
³ √
´³ √
´
√
√
√
3 5 + 2 2 5 βˆ’ 7 = 6 25 βˆ’ 21 5 + 4 5 βˆ’ 14
√
√
= 6 · 5 βˆ’ 21 5 + 4 5 βˆ’ 14
√
√
= 30 βˆ’ 21 5 + 4 5 βˆ’ 14
√
= 16 βˆ’ 17 5
Be careful in the preceding example.
√ A common mistake is to combine
the 16 βˆ’ 17. Remember that the βˆ’17 5 is considered one term. Therefore,
since the 16 does not have a radical, these two terms cannot be combined.
202
CHAPTER 6. RADICALS
Practice 4. Simplify:
¡ √
¢¡ √
¢
4 2 βˆ’ 3 2 2 + 5 (Answer on page 204)
Recall from Section 4.2 that (a + b)2 = (a + b)(a + b).
√ ¢2
¡ √
Example 5. Simplify: 2 3 + 6
Solution 5.
³ √
√ ´2 ³ √
√ ´³ √
√ ´
2 3+ 6 = 2 3+ 6 2 3+ 6
√
√
√
√
= 4 9 + 2 18 + 2 18 + 36
√
= 4 · 3 + 4 18 + 6
√ √
= 12 + 4 9 2 + 6
√
= 18 + 4 · 3 2
√
= 18 + 12 2
Practice 5. Simplify:
¡βˆš
¢2
8 βˆ’ 3 (Answer on page 204)
Example 6. Simplify:
¡ √
¢¡ √
¢
3 5βˆ’4 3 5+4
Solution 6.
³ √
´³ √
´
√
√
√
3 5 βˆ’ 4 3 5 + 4 = 9 25 + 12 5 βˆ’ 12 5 βˆ’ 16
√
= 9 25 βˆ’ 16
= 9 · 5 βˆ’ 16
= 45 βˆ’ 16
= 29
6.4. MULTIPLICATION OF RADICALS
203
The preceding example could also be simplified using the difference of
squares formula, (a βˆ’ b)(a + b) = a2 βˆ’ b2 , stated in Section 5.2. Using this
formula, the previous example is simplified as
³ √
´³ √
´ ³ √ ´2
3 5 βˆ’ 4 3 5 + 4 = 3 5 βˆ’ 42
√
= 9 25 βˆ’ 16
= 45 βˆ’ 16
= 29
¡ √
¢¡ √
¢
Practice 6. Simplify: 2 7 + 3 2 7 βˆ’ 3 (Answer on page 204)
Example 7. Simplify:
√ ¢¡ √
√ ¢
¡ √
2 8 βˆ’ 3 2 48 βˆ’ 2
√
√
Solution 7. Since 8 and 48 can be simplified, we will do this
first.
³ √
√ ´³ √
√ ´ ³ √
√ ´³ √
√ ´
2 8 βˆ’ 3 2 48 βˆ’ 2 = 2 4 · 2 βˆ’ 3 2 16 · 3 βˆ’ 2
³
√
√ ´³
√
√ ´
= 2·2 2βˆ’ 3 2·4 3βˆ’ 2
³ √
√ ´³ √
√ ´
= 4 2βˆ’ 3 8 3βˆ’ 2
Finally, using the distributive property, we obtain
³ √
√ ´³ √
√ ´ ³ √
√ ´³ √
√ ´
2 8 βˆ’ 3 2 48 βˆ’ 2 = 4 2 βˆ’ 3 8 3 βˆ’ 2
√
√
√
√
= 32 6 βˆ’ 4 4 βˆ’ 8 9 + 6
√
√
= 32 6 βˆ’ 4 · 2 βˆ’ 8 · 3 + 6
√
√
= 32 6 βˆ’ 8 βˆ’ 24 + 6
√
= βˆ’32 + 33 6
In the previous example, we could also distribute first and then simplify
your answer.
204
CHAPTER 6. RADICALS
Practice 7. Simplify:
√ ¢¡ √
√ ¢
¡ √
2 27 βˆ’ 2 2 3 + 8 (Answer on page 204)
ANSWERS TO PRACTICE PROBLEMS
√
1. 20 3
√
5. 17 βˆ’ 12 2
√
√
2. 8 3 + 24 5
√
3. 30x + 21x 3 4
6. 19
√
4. 1 + 14 2
√
7. 32 + 10 6
SECTION 6.4 EXERCISES
(Answers are found on page 253.)
Perform the indicated operations and simplify. Assume all variables represent positive real numbers.
1.
√ √
6 15
10.
√ √
√
8( 2 βˆ’ 5)
2.
¡ √ ¢¡ √ ¢
3 2 5 32
11.
√
√ √
3a( 3a βˆ’ 3b)
3.
√
√
3
9 3 βˆ’9
12.
√ √
√
5x( 10x βˆ’ x)
4.
¡ √ ¢¡ √ ¢
4 5 βˆ’3 15
13.
√
√
(3 x βˆ’ 2y)(5 x βˆ’ 4y)
5.
¡ √ ¢¡ √ ¢
βˆ’3 7 6 21
14.
√
√
√
√
(5 x + 2 y)(3 x βˆ’ y)
15.
√
√
( 3x + y)( 3x βˆ’ y)
16.
√
( x + 5)2
6.
7.
√
√
3a2 b5 6ab7
p
5x3 y
p
10x2 y
8.
√ √
√
2( 2 βˆ’ 3)
17.
√
(2 x βˆ’ 3)2
9.
√
√
3( 12 βˆ’ 3)
18.
(3 +
√
√
2)(2 βˆ’ 3)
6.4. MULTIPLICATION OF RADICALS
205
19.
√
√
( x βˆ’ 4)( x + 4)
35.
√
√
(2x 3x3 )(5x 12x)
20.
√
( x + 3)2
36.
√
(3 2 βˆ’ 4)2
21.
√
√
( x + 4)( x βˆ’ 1)
37.
¡ √
¢2
2 8βˆ’5
22.
√
√
(4 x + 3)2
38.
¢¡ √
¢
¡ √
3 5βˆ’2 2 5βˆ’7
23.
√ ¢¡ √
√ ¢
¡ √
3 2βˆ’4 3 5 2+ 3
39.
¡ √
¢¡ √
¢
3 5βˆ’4 3 5+4
24.
√ ¡ √
¢
4 3 2 6βˆ’3
40.
√ ¡ √
√ ¢
4 2 5 6 βˆ’ 3 10
25.
¡ √
¢2
3 5+4
41.
¡ √
¢¡ √ ¢
βˆ’8 4 8 2 4 4
26.
√ ¡ √
√ ¢
3 7 2 7βˆ’3 2
42.
√ ¡ √
√ ¢
3 7 2 7+6 2
27.
√ ¢¡ √
√ ¢
¡ √
2 3βˆ’5 2 2 3+5 2
43.
√ ¢¡ √
√ ¢
¡ √
2 3+5 6 3 3+4 6
28.
¡ √
¢¡ √
¢
4 12 βˆ’ 2 5 3 βˆ’ 3
44.
√ ¢2
¡ √
2 3βˆ’5 2
29.
¡ √
¢¡ √
¢
3 5+4 2 5βˆ’9
45.
¡ √
¢¡ √
¢
4 12 βˆ’ 2 5 3 βˆ’ 5
30.
√
√
( 8 βˆ’ 3)2
46.
√ ¡ √
√ ¢
4 5 2 5 βˆ’ 10
31.
√ ¡ √
√ ¢
4 3 2 27 βˆ’ 5 3
47.
¢¡ √
¢
¡ √
2 5+3 3 5βˆ’6
32.
√
√
√
√
(2 3 βˆ’ 5 6)(3 3 + 4 6)
48.
√ ¢2
¡ √
4 x+x 7
33.
√
√
√
3 2x(4 6x βˆ’ 8x)
49.
¢¡ √ ¢
¡ √
βˆ’4 3 9 3 3 9
34.
√
√
(3 2 βˆ’ 5)(4 2 + 7)
50.
¢¡ √
¢
¡ √
2 12 βˆ’ 4 5 3 + 5
206
6.5
CHAPTER 6. RADICALS
Rationalizing the Denominator
In Section 6.2, we noted that a simplified radical expression can have no
fraction under the radical and no radical in the denominator. Therefore, if
either of these occur we need to rewrite the expression so that there is only a
rational number in the denominator. Rationalizing the denominator is
the process of removing radicals from the denominator so that the expression
will be in simplified form.
2
Example 1. Simplify: √
3
Solution 1. Since we do not have an equation, the only number
that we can multiply by is one. We√need to multiply both the numerator and denominator
each by 3. Hence, we are multiplying
√
3
the expression by √ or one in disguise.
3
√
2
2
3
√ =√ ·βˆš
3
3
3
√
2 3
= √
9
√
2 3
=
3
Since the denominator is a rational number, we have successfully
simplified the radical expression.
Be careful not to divide (or cancel) terms that
√ are√inside a radical with
6
2
those that are outside a radical. For example,
6=
.
15
5
7
Practice 1. Simplify: √ (Answer on page 212)
5
βˆ’12
Example 2. Simplify: √
18
6.5. RATIONALIZING THE DENOMINATOR
207
√
√ √
√
Solution 2. First, note that 18 = 9 2 = 3 2. Therefore,
we really
√ only need to multiply the numerator and denominator by 2 in order
to rationalize the denominator. Therefore,
√
2
multiplying by √2 , we get
βˆ’12
βˆ’6
√ = √
18
3 2
√
2
βˆ’12
= √ ·βˆš
3 2
2
√
βˆ’12 2
√
=
3 4
√
βˆ’12 2
=
3·2
√
βˆ’12 2
=
6
√
= βˆ’2 2
The previous example can√also be solved by multiplying both the numerator and denominator by 18 first and then simplifying later.
9
Practice 2. Simplify: √ (Answer on page 212)
27
r
Example 3. Simplify:
5
6
Solution 3. First, we will use the quotient rule for√radicals and
then multiply both numerator and denominator by 6.
208
CHAPTER 6. RADICALS
r
r
Practice 3. Simplify:
√
5
5
=√
6
6
√ √
5
6
=√ ·βˆš
6
6
√
30
=√
36
√
30
=
6
3
(Answer on page 212)
5
√
Now, suppose it is√not a square root in the denominator, but 3 2. Notice
that multiplying
by 3√2 will NOT eliminate the radical from the denomi√
√
3
3
nator √
since 2 2 = 3 4. In Section 6.1 we noted
√ that if the index is odd,
then n an = a and if the index is even then n an = |a|. We will use this
property to eliminate
higher order radicals from
the denominator.
There√
√
√
3 2
3
3
fore, to eliminate 2 we need to multiply by 2 , or 4 in order for it to
work, since
√
√
√
3
3
3
2 · 22 = 23 = 2.
5
Example 4. Simplify: √
3
2
Solution 4. In order to eliminate
the radical in the denomina√
tor, we need to multiply by 3 4 on both top and bottom.
√
3
5
5
4
√
√
√
=
·
3
3
3
2
2
4
√
534
= √
3
8
√
534
=
2
6.5. RATIONALIZING THE DENOMINATOR
r
Practice 4. Simplify:
3
209
4
(Answer on page 212)
9
Whenever a radical expression contains a sum or difference involving
radicals in the denominator, we rationalize the denominator by multiplying
both numerator and denominator by the conjugate of the denominator. The
conjugate contains exactly the same numbers in exactly the same order
√
with the
the conjugate of 2 + 3
√ operation sign changed.√ For example,
√
is 2 βˆ’ 3 and the conjugate of 3 + 7 is 3 βˆ’ 7. This clears the radicals from the denominator as a result of the difference of square formula
(a + b)(a βˆ’ b) = a2 βˆ’ b2 .
2
Example 5. Simplify: √
5βˆ’3
Solution√5. We need to multiply both numerator and denominator by 5 + 3 which is the conjugate of the denominator in
order to simplify the radical expression.
√
2
( 5 + 3)
2
√
= √
· √
5βˆ’3
( 5 βˆ’ 3) ( 5 + 3)
√
2( 5 + 3)
√
√
=√
25 + 3 5 βˆ’ 3 5 βˆ’ 9
√
2( 5 + 3)
=
5βˆ’9
√
2( 5 + 3)
=
βˆ’4
√
βˆ’( 5 + 3)
=
2
Practice 5. Simplify:
12
√ (Answer on page 212)
3βˆ’ 6
210
CHAPTER 6. RADICALS
Example 6. Simplify:
4
√
5+2 3
Solution 6. Here√we need to multiply both numerator and denominator by 5βˆ’2 3 which is the conjugate of the denominator.
√
4
4
5βˆ’2 3
√ =
√ ·
√
5+2 3
5+2 3 5βˆ’2 3
√
4(5 βˆ’ 2 3)
√
√
√
=
25 βˆ’ 10 3 + 10 3 βˆ’ 4 9
√
4(5 βˆ’ 2 3)
=
25 βˆ’ 12
√
4(5 βˆ’ 2 3)
=
13
Practice 6. Simplify:
3
√
(Answer on page 212)
2 5+3
Multiplying by the conjugate works even when both of the terms in the
denominator contain a radical.
15
√
Example 7. Simplify: √
7+ 2
Solution 7. We will√rationalize
by multiplying the numerator
√
and denominator by 7 βˆ’ 2.
6.5. RATIONALIZING THE DENOMINATOR
√
√
15
15
( 7 βˆ’ 2)
√
√ = √
√ · √
√
7+ 2
( 7 + 2) ( 7 βˆ’ 2)
√
√
15( 7 βˆ’ 2)
√
√
√
=√
49 + 14 βˆ’ 14 βˆ’ 4
√
√
15( 7 βˆ’ 2)
=
7βˆ’2
√
√
15( 7 βˆ’ 2)
=
5
√
√
= 3( 7 βˆ’ 2)
βˆ’6
√ (Answer on page 212)
Practice 7. Simplify: √
3+2 5
√
1βˆ’ 2
√
Example 8. Simplify: √
8+ 6
Solution√8. We
√ need to multiply both numerator and denominator by 8 βˆ’ 6 which is the conjugate of the denominator.
√
√
√
√
1βˆ’ 2
(1 βˆ’ 2) ( 8 βˆ’ 6)
√
√ = √
√ · √
√
8+ 6
( 8 + 6) ( 8 βˆ’ 6)
√ √
√
(1 βˆ’ 2)( 8 βˆ’ 6)
√
√
√
=√
64 + 48 βˆ’ 48 βˆ’ 36
√ √
√
(1 βˆ’ 2)( 8 βˆ’ 6)
=
8βˆ’6
√ √ √
√
(1 βˆ’ 2)( 4 2 βˆ’ 6)
=
2
√
√
√
(1 βˆ’ 2)(2 2 βˆ’ 6)
=
2
211
212
CHAPTER 6. RADICALS
√
4+ 3
√ (Answer below)
Practice 8. Simplify: √
12 βˆ’ 2
ANSWERS TO PRACTICE PROBLEMS
1.
2.
√
7 5
5. 4(3 +
5
√
6.
3
√
3.
4.
15
7.
5
√
3
12
8.
3
√
6)
√
3(2 5 βˆ’ 3)
11
√
√
6( 3 βˆ’ 2 5)
17
(4 +
√
√
√
3)(2 3 + 2)
10
SECTION 6.5 EXERCISES
(Answers are found on page 254.)
Rationalize the denominator in each expression. Assume that all variables
represent positive real numbers.
r
r
8
11
3
√
1.
8.
15.
6
5
8
r
2.
3
√
3
9.
6
√
18
16.
3.
1
√
8
10.
4
√
6
16
17. √
28
11.
18
√
12
9
18. √
108
√
4 5
19. √
50
4.
5.
6
√
12x
r
11
4
r
6.
r
7.
8
9
2
3
12.
13.
3
√
12
r
5
9
r
14.
8
16
6
20. √
45
√
4 2
21. √
24
2
7
6.5. RATIONALIZING THE DENOMINATOR
7
22. √
72
27.
21
23. √
7
28.
5
24. √
4
x
29.
r
25.
5x
8
15
26. √
48
30.
√
6 2
√
5 3
√
4 3
√
6
√
20 15
√
5
√
8 2
√
24
7
31. √
3
2
213
r
32.
3
2
3
3
33. √
5
x2
4
34. √
4
8
8
35. √
7
x3
5
36. √
3
9
Rationalize the denominator in each expression. Assume that all variables
represent positive real numbers.
37.
6
√
7+3
45.
6
√
√
5+ 3
53.
38.
2
√
3βˆ’ 2
46.
5
√
√
7βˆ’ 3
54.
8
√
2+ 6
39.
4
√
1+ 3
15
√
47. √
6+ 3
55.
21
√
√
2 2βˆ’ 5
40.
10
√
√
7+ 5
12
48. √
7+4
56.
12
√
3βˆ’ 6
41.
6
√
5+1
49.
12
√
2βˆ’ 6
57.
18
√
√
10 + 6
42.
15
√
3βˆ’6
βˆ’14
√
50. √
3+ 7
43.
11
√
5βˆ’ 3
51.
44.
12
√
7βˆ’4
2
√
52. √
7βˆ’ 3
8
√
1βˆ’ 5
2
√
5βˆ’3
6
58. √
7βˆ’4
15
59. √
11 βˆ’ 4
60.
2
√
3 2+4
214
CHAPTER 6. RADICALS
βˆ’4
√
61. √
3βˆ’2 5
63.
24
√
5+2 3
14
√
62. √
11 βˆ’ 3 2
√
2+ 2
√
64.
5βˆ’ 3
√
1βˆ’ 7
√
65.
4+3 2
6.6. RATIONAL EXPONENTS
6.6
215
Rational Exponents
We have already defined the exponent rules for integer exponents. However,
what about exponential expressions that contain rational exponents such as
21/3 , 272/3 , and 32βˆ’1/5 . We need a definition that will still obey all the rules
discussed in sections 4.1 and 4.3.
Consider x = 21/3 . Then using the rules of exponents we see that
³
´3
x3 = 21/3 = 21/3 · 21/3 · 21/3 = 21 = 2.
Since x3 = 2 we know that x =
Hence,
√
3
2. However, we also know that x = 21/3 .
21/3 =
√
3
2.
We leave it to the reader to examine other examples. This leads to the
following definitions.
Rational exponents: If m and n are positive integers
with m/n in lowest terms, then
√
n
x
√
= n xm
β€’
x1/n =
β€’
xm/n
OR
√
xm/n = ( n x)m
If n is even then we require x β‰₯ 0.
In other words, in a rational exponent, the numerator indicates the power
and the denominator indicates the root.
Example 1. Write using radicals (do not rationalize):
5x1/2 + 3x2/3 + 7xβˆ’2/5 + 8xβˆ’1/3
216
CHAPTER 6. RADICALS
Solution 1.
1/2
5x
2/3
+ 3x
+ 7x
βˆ’2/5
βˆ’1/3
+ 8x
√
√
3
= 5 x + 3 x2 + 7
µ
1
¶
x2/5
µ
+8
1
x1/3
√
√
8
7
3
= 5 x + 3 x2 + √
+√
5
3
x
x2
Practice 1. Write using radicals (do not rationalize). (Answer on page 223)
2 1/3 3 βˆ’1/2 5 βˆ’2/3
x + x
+ x
5
7
8
Example 2. Write using rational exponents:
√
√
√
8
5
3 xβˆ’2 3 x+ √
+7 x2
4
x3
Solution 2.
√
√
√
8
8
5
3 xβˆ’23x+ √
+ 7 x2 = 3x1/2 βˆ’ 2x1/3 + 3/4 + 7x2/5
4
x
x3
1/2
1/3
= 3x βˆ’ 2x + 8xβˆ’3/4 + 7x2/5
Practice 2. Write using rational exponents. (Answer on page 223)
√
√
4
9
4
√
βˆ’
3
x5 + 7 x βˆ’ √
5
3
2
x
x2
Example 3. Simplify: 163/4
¶
6.6. RATIONAL EXPONENTS
217
Solution 3. Since 34 is the exponent, we know that we need to
use the fourth root and the third power. So,
163/4 =
³ √ ´3
4
16
= (2)3
=8
Practice 3. Simplify: 324/5 (Answer on page 223)
As in Section 4.3, we need to be very careful with negatives. The placement of the negative sign does matter.
Example 4. Simplify: (a) βˆ’163/4 and (b) (βˆ’16)3/4 .
Solution 4. (a) Note that for βˆ’163/4 the negative is not raised
to the three fourths power. So, we view βˆ’163/4 as βˆ’1·163/4 .
From the previous example we see that βˆ’163/4 = βˆ’8.
(b) Here, the negative is raised to the three fourths power. So,
¡βˆš
¢3
switching to radical form, we have (βˆ’16)3/4 = 4 βˆ’16 .
However, we know that we cannot take the even root of
a negative number in the Real Number System. Hence,
(βˆ’16)3/4 is not a real number.
Practice 4. Simplify: (a) βˆ’82/3 and (b) (βˆ’8)2/3 (Answer on page 223)
µ
Example 5. Simplify:
16
81
¶3/4
218
CHAPTER 6. RADICALS
Solution 5.
µ
16
81
Ãr
¶3/4
4
=
16
81
!3
µ ¶3
2
=
3
=
µ
Practice 5. Simplify:
25
36
8
27
¶3/2
(Answer on page 223)
Recall from section 4.3 that xβˆ’n = x1n . Since the exponent rules must
be valid for rational exponents, we have
xβˆ’m/n =
1
.
xm/n
Example 6. Simplify: 32βˆ’2/5
Solution 6.
32βˆ’2/5 =
1
322/5
1
= ¡ √ ¢2
5
32
=
1
(2)2
=
1
4
Practice 6. Simplify: 64βˆ’2/3 . (Answer on page 223)
6.6. RATIONAL EXPONENTS
219
We now turn our attention to operations involving rational exponential
expressions. Remember that according to the order of operations, exponents
are evaluated before multiplication.
Example 7. Evaluate:
3(27)1/3 βˆ’ 3(8)1/3
Solution 7.
√
√
3
3
3(27)1/3 βˆ’ 3(8)1/3 = 3 27 βˆ’ 3 8
= 3(3) βˆ’ 3(2)
=9βˆ’6
=3
Practice 7. Simplify: 5(36)1/2 βˆ’ 5(4)1/2 (Answer on page 223)
Example 8. Evaluate:
4 3/2 4 3/2
(9) βˆ’ (1)
3
3
Solution 8.
4 3/2 4 3/2 4 √ 3 4 √ 3
(9) βˆ’ (1) = ( 9) βˆ’ ( 1)
3
3
3
3
4
4
= (3)3 βˆ’ (1)3
3
3
4
4
= (27) βˆ’ (1)
3
3
=
108 4
βˆ’
3
3
=
104
3
220
CHAPTER 6. RADICALS
Practice 8. Evaluate:
Example 9. Evaluate:
3 2/3 3 2/3
(8) βˆ’ (1)
(Answer on page 223)
5
5
3(βˆ’1)βˆ’2/3 βˆ’ 3(βˆ’8)βˆ’2/3
Solution 9.
µ
3(βˆ’1)
βˆ’2/3
βˆ’2/3
βˆ’ 3(βˆ’8)
=3
µ
=3
1
√
3
( βˆ’1)2
1
(βˆ’1)2
¶
µ
βˆ’3
¶
µ
βˆ’3
1
√
3
( βˆ’8)2
1
(βˆ’2)2
¶
¶
µ ¶
µ ¶
1
1
βˆ’3
=3
1
4
=3βˆ’
Practice 9. Evaluate:
3
4
=
12 3
βˆ’
4
4
=
9
4
5(βˆ’1)βˆ’2/5 βˆ’ 5(βˆ’32)βˆ’2/5 (Answer on page 223)
In Sections 4.1 and 4.3 we discussed the exponent rules as they applied
to integer exponent values. These same rules apply to rational exponents.
The exponent rules are stated here, again, for convenience.
6.6. RATIONAL EXPONENTS
221
Exponent Rules: Let m and n be real numbers.
β€’ am · an = am+n
am
β€’ n = amβˆ’n
a
β€’ aβˆ’n =
β€’ (am )n = amn
β€’
β€’ (ab)n = an bn
β€’
β€’ a0 = 1
1
an
bn
aβˆ’m
=
bβˆ’n
am
³ a ´βˆ’n
b
=
µ ¶n
b
a
Example 10. Use the rules of exponents to simplify the following. Assume
that all variable represent positive real numbers. Write answer using positive
exponents only.
(x2/3 )2
(x2 )7/3
Solution 10.
(x2/3 )2
x4/3
=
(x2 )7/3
x14/3
= x4/3βˆ’14/3
= xβˆ’10/3
=
1
x10/3
Practice 10. Use the rules of exponents to simplify the following. Assume
that all variable represent positive real numbers. Write using positive exponents only. (Answer on page 223)
¡ 4 ¢2/3
x
¡
¢2
x3/4
222
CHAPTER 6. RADICALS
Example 11. Use the rules of exponents to simplify the following. Assume that all variable represent positive real numbers. Write using positive
exponents only.
³
´3 ¡ ¢
2/3
2x4 y βˆ’4/5
8y 2
Solution 11.
³
2x4 y βˆ’4/5
´3 ¡ ¢
³
´³
´
2/3
8y 2
= 8x12 y βˆ’12/5 82/3 y 4/3
³
´³
´
= 8x12 y βˆ’12/5 4y 4/3
= 32x12 y βˆ’12/5+4/3
= 32x12 y βˆ’36/15+20/15
= 32x12 y βˆ’16/15
=
32x12
y 16/15
Practice 11. Use the rules of exponents to simplify the following. Assume
that all variable represent positive real numbers. Write using positive exponents only. (Answer on page 223)
³
βˆ’3xβˆ’2/3 y 2
´3
(4x)βˆ’1/2
Example 12. Use the rules of exponents to simplify the following. Assume that all variable represent positive real numbers. Write using positive
exponents only.
¡ 15 βˆ’5 ¢1/5
x y
(xβˆ’2 y 3 )1/3
6.6. RATIONAL EXPONENTS
223
Solution 12.
¡ 15 βˆ’5 ¢1/5
x y
(xβˆ’2 y 3 )1/3
=
x3 y βˆ’1
xβˆ’2/3 y
=
x3 x2/3
yy
=
x3+2/3
y2
=
x11/3
y2
Practice 12. Use the rules of exponents to simplify the following. Assume
that all variable represent positive real numbers. Write using positive exponents only. (Answer below)
¡ βˆ’3 4 ¢1/2
x y
(x8 y βˆ’12 )1/4
ANSWERS TO PRACTICE PROBLEMS
1.
2 √
3
x+
5
5
3
√ + √
3
7 x
8 x2
2. 4xβˆ’2/5 βˆ’ 3x5/4 + 7x1/2 βˆ’ 9xβˆ’2/3
3. 16
4.
6.
8.
9.
9
5
15
4
(a) βˆ’4
(b) 4
5.
7. 20
125
10. x7/6
11. βˆ’
216
1
16
12.
27y 6
2x5/2
y5
x7/2
224
CHAPTER 6. RADICALS
SECTION 6.6 EXERCISES
(Answers are found on page 256.)
Write using radicals (do not rationalize).
11 1/4
x
12
1.
4x2/3 + 3x1/3 βˆ’ 2x1/2
5.
7x4/7 βˆ’ 3xβˆ’3/5 +
2.
5x1/2 + 2x2/3 + 3x3/4
6.
5 2/3
x βˆ’ 3x1/5 + 6x3/4
6
3.
4xβˆ’2/3 + 2xβˆ’1/3 + 6x1/2
7.
2x3/2 βˆ’ 6x2/3 βˆ’ 4xβˆ’1/2
4.
3 βˆ’2/3 3 βˆ’1/2 7 1/3
x
+ x
+ x
4
8
5
8.
9xβˆ’1/4 + 6x3/5 βˆ’ 2xβˆ’1/3
Write using rational exponents.
√
√
√
5
9.
x2 + 3 x βˆ’ 2 x
13.
√
6
2
√ βˆ’βˆš
+84x
3
5 x
x
10.
√
√
√
√
7
3
5 x4 + 8 x + 2 x2 βˆ’ 5 4 x
14.
√
1
4
√
+
3
x3 βˆ’ √
3
3
2 x
x
11.
√
√
√
5
3
4
7 x2 + 3 x2 βˆ’ 4 x3
15.
√
√
1
5
4
4 x4 βˆ’ √ + 5 x3
3 x
12.
√
√
5
93xβˆ’ √
βˆ’ 2 x5
4
x3
16.
√
3
√
√
x
5
4
+ 3 x2 βˆ’ 6 x5
4
Evaluate.
17.
361/2
22.
8βˆ’1/3
27.
(βˆ’1)2/5
18.
(βˆ’64)2/3
23.
(βˆ’27)βˆ’2/3
28.
4(βˆ’8)βˆ’1/3
19.
93/2
24.
45/2
29.
5(4)3/2
20.
25βˆ’1/2
25.
2563/4
30.
32βˆ’3/5
21.
4βˆ’3/2
26.
9βˆ’3/2
31.
βˆ’161/2
6.6. RATIONAL EXPONENTS
32.
25βˆ’3/2
33.
81βˆ’3/4
34.
(βˆ’125)βˆ’2/3
35.
βˆ’491/2
36.
2433/5
37.
225
µ
38.
βˆ’3
µ
39.
µ
811/4
8
27
40.
8
27
¶2/3
µ
41.
µ
¶4/3
42.
81
16
27
8
¶βˆ’3/4
¶2/3
169
25
¶βˆ’1/2
Evaluate.
43.
3(8)2/3 βˆ’ 3(βˆ’1)2/3
49.
(8)βˆ’1/3 βˆ’ (1)βˆ’1/3
44.
2(9)3/2 βˆ’ 2(4)3/2
50.
1
1
(16)βˆ’1/2 βˆ’ (4)βˆ’1/2
2
2
45.
2(16)1/2 βˆ’ 3(9)1/2
51.
2 βˆ’1/2 2 βˆ’1/2
(4)
βˆ’ (1)
3
3
46.
3
3
(36)1/2 βˆ’ (16)1/2
2
2
52.
5(βˆ’1)1/3 βˆ’ 5(βˆ’8)1/3
47.
2
2
(27)1/3 βˆ’ (8)1/3
5
5
53.
2 5/2 2 5/2
(4) βˆ’ (1)
5
5
48.
6(9)βˆ’1/2 βˆ’ 2(4)βˆ’1/2
54.
3
3
βˆ’ (8)βˆ’2/3 + (1)βˆ’2/3
2
2
Use the rules of exponents to simplify the following. Assume that all variable
represent positive real numbers. Write using positive exponents only.
55.
56.
57.
¡ 1/3 1/2 ¢6
2x y
x3/4
x1/2 x
59.
x2/3 xβˆ’3/4 x7/6
x3/2
60.
¡ 3/4 βˆ’3/8 ¢βˆ’2 5/2
x
x x
(8x2 )1/3 (16x)βˆ’1/4
61.
58.
(25x3 )βˆ’1/2 (4x3 )1/2
x3/5 xβˆ’2
x3/10
226
CHAPTER 6. RADICALS
62.
¡
¢2/3
βˆ’64a3/4 b6
63.
¡
¢2/3
27x9 y 12
64.
¡
¢3/2
16x4 y 6
65.
x1/3
x2/3 · xβˆ’1
Ã
!1/2
66.
4x1/3 y
9xy 2
67.
x3/4 y βˆ’1/2
xβˆ’1/3 y 1/6
68.
¡ 3 ¢1/2
4x y
8x1/4 y βˆ’1/2
Chapter 7
Answers to Exercises
Chapter 1
Section 1.1 (Exercises on page 10.)
1. function;
Domain = {3, 4, 5, 7, 10};
Range = {βˆ’8, βˆ’2, 6}
8. not a function
9. function;
Domain = {βˆ’3, 5, 6, 8};
Range = {βˆ’5, 3, 12}
2. not a function
3. function;
Domain = {βˆ’2, βˆ’1, 1, 2};
Range = {7}
10. function;
Domain = {βˆ’3, βˆ’2, 2, 3};
Range = {4, 9}
4. function;
Domain = {βˆ’5, βˆ’3, βˆ’1, 1, 4};
Range = {βˆ’125, βˆ’27, 1, 64}
11. not a function
12. function;
Domain = {2, 4, 6};
Range = {12}
5. function;
Domain = {βˆ’1, 1, 2, 3, 7};
Range = {βˆ’10, 2, 3, 4, 8}
13. function
14. not a function
6. function;
Domain = {1, 2, 3, 4, 5};
Range = {3}
15. function
16. not a function
7. function;
Domain = {βˆ’2, βˆ’1, 0, 1, 2};
Range = {0, 1, 4}
17. function
18. not a function
227
228
CHAPTER 7. ANSWERS TO EXERCISES
19. not a function
(c) βˆ’5
20. function
(d) βˆ’6
21. not a function
22. function
23. function
24. not a function
25. (a) βˆ’7
30. (a) βˆ’13
(b) βˆ’13
(c) βˆ’13
(d) βˆ’13
31. (a) 22
(b) 16
(b) βˆ’22
(c) 10
(c) 2
(d) 31
(d) βˆ’31
26. (a) βˆ’4
(b) βˆ’6
(c) βˆ’4
21
(d) βˆ’
4
32. (a) 2
(b) βˆ’2
5
(c)
2
17
(d) βˆ’
4
1
8
3
(b) βˆ’
8
(c) 0
33. (a) βˆ’7
(d) βˆ’12
34. (a) 5
27. (a) βˆ’
(b) βˆ’3
(c) βˆ’19
(d) βˆ’39
28. (a) βˆ’2
(b) 11
(b) 6
14
(c)
5
2
(d)
3
(c) 39
29. (a) 22
(b) 9
(d) 3
35. (a) 4
(b) 10
(c) 13
(d) 16
229
Chapter 2
Section 2.1 (Exercises on page 22.)
1. x = βˆ’
2. x =
1
32
7
4
4. x = βˆ’3
24
17
6. x = βˆ’
34
25
13. no solution
3. no solution
5. x =
12. x = βˆ’
1
6
7. all real numbers
14. x = 7
15. no solution
16. x = 5
17. x = 0
18. all real numbers
19. x = 0
3
25
8. x =
3
14
20. x =
9. x =
11
17
21. x = βˆ’
10. x =
33
16
22. x = βˆ’6
19
11. x = βˆ’
11
7
5
23. x = 4
24. all real numbers
Section 2.2 (Exercises on page 31.)
1. 6
8. 12
2. 8
9. 9
3. 5
10. all real numbers
4. 47
11. 60
5. 10
12. βˆ’16
6. 2
13. 7 and 8
7. 5
14. βˆ’2
230
CHAPTER 7. ANSWERS TO EXERCISES
15. 12, 24, 45
16. 180
17. 89, 91
18. 12, 14
19. 5,015, 5,017
20. 3,342, 3,344, 3,346
21. 3,343, 3,344, 3,345
22. 22, 24
23. 11, 12, 13
24. 5 feet and 12 feet
25. 152 adults
26. Savanna pitched 3 games,
Kayla pitched 15 games,
Cheyenne pitched 10 games
27. Roger sold 15, Will sold 24, Jacob sold 27
(b) 5 gallons for two coats.
34. first piece: 8.5 in, second piece:
11 in; third piece: 8.5 in;
fourth piece: 11 in
35. 8 ft, 28 ft
36. 70 yd, 130 yd
37. 25 inches
38. 15 minutes
39. first song: 30 min; second
song: 60 min; third song: 90
min
40. 700 adults
41. 150 children, 1858 adults
42. 18 yards
43. 20 feet
44. each angle is 139β—¦
28. cashier for 3.5 hours, stocked
shelves for 2.75 hours, trained
new employees for 1.75 hours
45. 48β—¦ and 132β—¦
29. w = 5, l = 13
47. 48β—¦ and 42β—¦
30. width is 14 ft, length is 49 ft
48. each angle is 73β—¦
31. x = 25 ft, y = 50 ft
49. 35β—¦ and 55β—¦
32. 25 ft by 25 ft
50. 85β—¦
33. (a) 3 gallons for one coat.
51. 65β—¦
46. 54β—¦ and 126β—¦
231
Section 2.3 (Exercises on page 43.)
1. They are the same.
24. 62.5%
2. They are the same.
25. 75%
3. $5.00
26. 87.5%
4. $45.00
27. $6000
5. $8.50
28. $12000
6. $140
7. $800
8. 36 liters
9. 19.2 liters
10. 18 cds
11. 9 cups
12. 96%
13. 40%
14. 24 students
15. 180 women
16. 7 students
17. 20%
18. 25%
29. 2.5%
30. Job 2
31. Discount of 14 is the same
as paying 34 of the original
amount.
32. Candidate B won by one vote
33. $1500
34. 100β—¦ , 50β—¦ , 30β—¦
35. $6467.89
36. 125 square feet
37. $230.50
38. $21.18
39. $23.08
40. $233.16
41. $180
19. 120 oz
42. 15%
20. 20%
43. $13, 200
21. $540
44. $34, 170.74
22. $17
45. $9, 910
23. 25%
46. 130 games
232
CHAPTER 7. ANSWERS TO EXERCISES
47. 15.56%
48. 32%
(c) $318.75
51. (a) interest is $480; account
balance is $12, 480
49. $149.45
(b) $12, 979.20
50. (a) $637.50
(c) $19.20
(b) $1, 912.50
52. 6.5%
Section 2.4 (Exercises on page 52.)
1. (a) 16 ounces
(b) 24 ounces
(c) 11 ounces
(b)
5
8
9
(c) 1 16
15. 7 inches
2. 450 flashes
16. 2800 markers
3. 50 feet
17. 2 14 cups
4. 3000 sheets
18. 110 miles
5. 4.5 miles
19. 85.5 inches
6. 10 times
20. 75 teachers
7. 75 gallons
21. 960 female students
8. 3 miles
22. 500 fish
9. 40 pens
23. 7 lbs cement, 21 lbs gravel
10. 4 83 cups
24. 21 lbs
11. $62.25
25. 20.32 cm
12. 4 61 cups of flour
26. 35 gallons
13. Missed 16 goals
27. 14 lbs
14. (a) 2 12
28. $1, 174.90
233
Chapter 3
Section 3.1 (Exercises on page 64.)
1. (4, βˆ’2)
15. (2, βˆ’3)
2. (2, βˆ’4)
16. (1, βˆ’2)
3. no solution
17. no solution
4. (3, 2)
18. (7, 3)
5. (0, βˆ’3)
19. (2, 2)
6. (2, 1)
20. (3, 1)
7. (βˆ’4, βˆ’1)
21. (2, βˆ’2)
8. (1, 0)
22. no solution
9. (0, 9)
23. (2, 4)
10. infinite number of solutions
24. (0, 3)
11. (2, 2)
25. (6, βˆ’1)
12. (10, 1)
26. infinite number of solutions
13. (1, 6)
27. (0, βˆ’2)
14. (6, 2)
28. (βˆ’3, 4)
Section 3.2 (Exercises on page 74.)
¡ 38
7
11 , βˆ’ 11
1. (15, βˆ’6)
8.
2. (2, βˆ’1)
¡
¢
3. βˆ’ 85 , βˆ’ 11
5
9. (3, βˆ’2)
4. no solution
¢
10. (2, βˆ’1)
5. infinite number of solutions
11. no solution
¡ 1 22 ¢
12. 13
, 13
6. no solution
13. (5, βˆ’3)
7. infinite number of solutions
14. (1, 4)
234
CHAPTER 7. ANSWERS TO EXERCISES
15. (7, 0)
16. no solution
17. (3, βˆ’2)
18. (βˆ’4, 1)
19. (βˆ’2, βˆ’6)
20. (2, βˆ’1)
21. infinite number of solutions
¡
¢
3
22. βˆ’ 24
7 ,7
¡
¢
23
23. βˆ’ 169
29 , βˆ’ 29
¡
¢
6
24. 12
5 , βˆ’5
¡
¢
25. 27 , βˆ’ 24
7
¡
¢
26. 95 , 23
5
¡
¢
11
27. 17
,
10 10
¡ 8
¢
41
28. βˆ’ 13 , βˆ’ 13
Section 3.3 (Exercises on page 83.)
1. (βˆ’7, βˆ’5)
2. (2, βˆ’1)
¡
¢
4
3. 12
11 , βˆ’ 11
¡
¢
37
4. 19
12 , 12
¢
¡
5. βˆ’ 12 , 32
6. no solution
¢
¡
7. βˆ’4, βˆ’ 52
¡
¢
8. 13 , βˆ’ 32
¡
¢
17
9. 13
12 , βˆ’ 12
10. (βˆ’5, βˆ’7)
¡
¢
16
11. 46
7 ,βˆ’ 7
12. (2, βˆ’1)
13. (102, βˆ’15)
¡
¢
14. 14 , 1
17. (βˆ’4, 3)
¡
¢
18. 65 , βˆ’ 35
19. (3, βˆ’4)
20. (βˆ’2, βˆ’1)
µ
¶
1
21. 4, βˆ’
5
22. (βˆ’3, 2)
¶
µ
29 1
,
23.
11 11
µ
¶
1 3
24.
,
2 2
µ
¶
7 5
25.
,
22 11
26. (20, 9)
µ
¶
14 23
27.
,βˆ’
25 25
28. (2, βˆ’4)
15. infinite number of solutions
29. (βˆ’1, βˆ’5)
16. (3, 12)
30. (βˆ’2, 3)
235
31. No solution
32. Infinite number of solutions
33. (0, βˆ’2)
34. (βˆ’3, 4)
35. (5, 1)
¡
¢
36. 17 , βˆ’ 22
7
¡
¢
37. 3, 39
2
¡1
¢
38. 10 , βˆ’ 103
10
39. (βˆ’7, 1)
¡
¢
40. βˆ’ 23 , 12
¡
¢
41. βˆ’2, 12
¡ 3 9¢
42. βˆ’ 14
, 14
¡
¢
15
43. 68
31 , 31
¡5 3¢
44. 31
, 31
¡
¢
40
45. 11
39 , βˆ’ 39
¡ 1 29 ¢
46. βˆ’ 10
, 15
¡ 1
¢
4
47. βˆ’ 17
, βˆ’ 17
Section 3.4 (Exercises on page 93.)
1. 1.3 lbs of Almonds and 2.7 lbs
of Pecans
2. 80
13. 410 mph
14. 6 miles
15. walking 4 mph; biking 10 mph
3. 44 letters and 12 postcards
16. plane 620 mph; wind 20 mph
4. 8 Nickels and 14 quarters
17. rowing 3.75 mph;
rent 1.25 mph
5. 4 and 1
6. Tuition $13, 500;
Board $7, 300
Room and
water cur-
18. $12, 000 at 12%; $8, 000 at 7%
19. 246 dimes; 54 nickels
7. $2, 800 at 8%; $3, 200 at 7 12 %
20. 12.5 hours on day one
8. $6, 180 at 6%; $2, 820 at 10%
21. 1 hour
9. $750 at 5%; $1, 500 at 6%
22. 2.5 hours
10. $800 at 4 12 %; $700 at 3% loss
23. Jen’s rate is 60 mph; Sarah’s
rate is 40 mph
11. $4, 444.44 at 5%; $5, 555.56 at
4%
24. 24 gal of 90%; 96 gal of 75%
12. $909.09 at 3%;
5%
$1, 090.91 at
25. 60 L of 40%; 30 L of 70%
26. 2 L of 40%; 6 L of 60%
236
CHAPTER 7. ANSWERS TO EXERCISES
27. 8 g of 25%; 12 g of 50%
29. 4 12 oz of 4%; 7 12 oz of 12%
28. 13 31 gal of 40%; 6 23 gal of 70%
Chapter 4
Section 4.1 (Exercises on page 104.)
1. 37
20. βˆ’1
2. 103
21. βˆ’25
3. 63
µ ¶6
1
4.
2
22. 0.000001
5. βˆ’1
25. 311
6. 1
26. 8
7. βˆ’32
27. x14
8. 216
28. 219
9. 81
29. 222
10. βˆ’64
30. 513
11.
1
8
12. 1
13. 2.25
14. βˆ’49
23. 81
24. 107 = 10, 000, 000
31. 67
32. 253
33. 1514
34. 338
35. 8x4
15. 0
36. 49x10
16. 16
37. 64x6
17. βˆ’256
38. 27x12 y 15
18. 1
39. 36x6 y 18
19. βˆ’1
40. 16x8 y 20 z 36
237
41. βˆ’27x3 y 9 z 6
58.
x14 y 21
2
59.
x9 y 14
4
42. 8x9
43.
βˆ’24x15
44. 6b9
45. x7 y 4
46.
x9 y 8
6
47. x6 y 9
48. 8x12 y 23
49. 12x11 y 10
9x8
50.
4y 10
51.
125y 12 z 3
8x9
52.
3x10
9y 7
53.
x10
2y 5
54. 32x20 y 5
55. 63x9 y 11
56.
36x4
y5
57. βˆ’72x14 y 17
60. x8 y 13 z 3
61. 428 = (22 )28 = 256 and 818 =
(23 )18 = 254 . Since 56 > 54,
428 > 818 .
62. 39 + 39 + 39 = 3 · 39 = 310
and 96 = (32 )6 = 312 . Since
12 > 10, 96 > 39 + 39 + 39 .
63. 279 = (33 )9 = 327 and 914 =
(32 )14 = 328 . Since 28 > 27,
914 > 279 .
64. 618 = (2 · 3)18 = 218 · 318 and
336 = 318 · 318 . Since 218 < 318 ,
618 < 336 .
¡ ¢2
65. Not equal. 34 = 34·2 = 38 ;
2
whereas, 34 = 316 .
66. (3 + 4)3 6= 33 + 43 since (3 +
4)3 = 73 = 343 and 33 + 43 =
27 + 64 = 91. In order for
(a+b)3 = a3 +b3 we need either
a or b to be zero. Furthermore,
the only value of n for which
(a+b)n = an +bn is when n = 1.
Section 4.2 (Exercises on page 114.)
1. Yes.
One
example: ¢ 3. No. A monomial times a bi¡ 3
¢ ¡
2
3x + 7x βˆ’ 5 + βˆ’3x3 + 8x βˆ’ 2 = nomial will always give a binomial.
7x2 + 8x βˆ’ 7.
2. Yes.
4. 5th degree; m + n degree
238
CHAPTER 7. ANSWERS TO EXERCISES
5. 8
29. βˆ’2x2 βˆ’ 8x + 6
6. 7
30. x3 βˆ’ 10x2 + 22x βˆ’ 5
7. 6
31. 7x3 βˆ’ 11x2 + 6x βˆ’ 9
8. 3
32. 6a3 b2 + a2 b2 βˆ’ 2ab2
9. 5
10. 3
11. βˆ’25
12.
βˆ’ 23
2
13. 6
14. 35
33. 19mn2 βˆ’ 16m2 n βˆ’ 5
34. 12x2 βˆ’ 8x
35. 30x3 + 15x2
36. βˆ’8x3 + 6x2 βˆ’ 10x
37. 15x4 βˆ’ 6x3 βˆ’ 21x2
38. βˆ’15x3 y 3 + 21x3 y 2 βˆ’ 9x4 y
15. βˆ’15
39. βˆ’10x4 y 6 + 6x3 y 7 βˆ’ 4x5 y 8 +
14x2 y 5
16. βˆ’6
40. 4x2 + 4x βˆ’ 15
17. 0
41. 15x2 βˆ’ 22x + 8
18. 27
19. 12
20. βˆ’97
21. βˆ’33
22. 1200
23. 108
24. βˆ’x + 16
42. 28x2 + 13x βˆ’ 6
43. 10x2 βˆ’ x βˆ’ 3
44. 24x2 + 8x βˆ’ 2
45. 4x2 βˆ’ 9
46. 16x2 βˆ’ 1
47. 25x2 βˆ’ 4
48. 9x2 βˆ’ 16
βˆ’
78
5 x
+2
25. 5x βˆ’ 2
50. 6x2 βˆ’
7
12 x
βˆ’
26. 4x2
51. 49x2 βˆ’ 28x + 4
27. βˆ’a2 βˆ’ a βˆ’ 1
52. 16x2 + 24x + 9
28. 11x2 βˆ’ 3x βˆ’ 11
53. 4x2 βˆ’ 12x + 9
49.
9 2
2x
1
6
239
54. x3 βˆ’ 6x2 + 12x βˆ’ 8
62. 6x4 βˆ’ 22x3 + 41x2 βˆ’ 41x + 10
55. 8x3 + 12x2 + 6x + 1
63. 6x4 βˆ’ 28x3 + 21x2 + 21x βˆ’ 10
56. 12x3 βˆ’ 17x2 βˆ’ 13x βˆ’ 2
64. 2x4 βˆ’ x3 βˆ’ 24x2 + 16x + 7
57. 2x3 βˆ’ 11x2 + 10x + 8
58.
21x3
βˆ’
19x2
βˆ’ 12x
59. 16x4 + 28x3 βˆ’ 30x2
60. 12x3 + 2x2 βˆ’ 24x
61. 10x3 βˆ’ 19x2 + 26x βˆ’ 8
65. 18x4 βˆ’ 15x3 βˆ’ 13x2 βˆ’ 2x βˆ’ 12
66. Let x = 1. Then x + 2(x βˆ’ 3) =
1 + 2(1 βˆ’ 3) = 1 + 2(βˆ’2) =
1 + (βˆ’4) = βˆ’3. However,
(x+2)(xβˆ’3) = (1+2)(1βˆ’3) =
(3)(βˆ’2) = βˆ’6.
Section 4.3 (Exercises on page 123.)
1.
1
81
14.
2.
1
64
15.
7
12
16.
βˆ’
17.
9
4
18.
106
9
1
7.
4
19.
7
12
8. 25
20.
βˆ’
21.
7
2
22.
112
9
3. βˆ’
1
625
4. 8
1
5.
16
1
6. βˆ’
4
9. 49
10.
81
256
11. 216
βˆ’5
36
12.
81
4
23.
7
3
13.
5
16
24.
βˆ’
7
16
7
8
9
4
240
CHAPTER 7. ANSWERS TO EXERCISES
25. x6
45.
9y 6
5x3
46.
7x5
12y 5
47.
x8
12y 2
48.
3x6
y3
49.
6x11
y8z7
50.
1
x4 y 6
51.
y 12
x9
52.
x2
4y 12
53.
b8
a4 c2
26. 23
27. 24
28. 57
29. 241
30. 322
31. xy 3
32.
y
x5
33. x13 y 8
34.
1
x2 y 4
35. x3 y
36. x10 y 8
6
37.
x
38.
2
3x2
54.
27x6
y9
39.
12
x2 y
55.
x20 z 16
y 28
40.
10a
b4
56.
8y 6
x9
41.
8y 9 z 15
x6
57.
16
5xy 9
42.
16x6
y 10 z 2
58.
2
y5
43.
z6
9x8 y 4
59.
x4
9y 8
44.
x12 z 18
125y 9
60.
4x2 y
3
241
61.
2x8
5y 10
67.
16x20
81y 8
62.
1
32x2 y 2
68.
2x12
9y 23
63.
3
8x2 y 11
69.
βˆ’8y 24
x27 z 3
70.
3y 12
4x6
71.
27y 6 z 8
4x15
8y 14
9x13
4
65. 2
x y
64.
66.
4y 6 z 10
x6
72. Answers vary; 213 6= 26
Section 4.4 (Exercises on page 130.)
1. 946, 000, 000, 000, 000, 000
15. 4.2 × 10βˆ’5
2. 1,810,000
16. 4.36 × 106
3. 0.0000000001
17. 3.005 × 10βˆ’3
4. 0.0043
18. 7.201 × 10βˆ’6
5. 0.000032
19. 2.94 × 108
6. 578, 900, 000
20. 3.45 × 10βˆ’7
7. 693, 000, 000, 000
21. 7.03 × 10βˆ’8
8. 0.000000034
22. 6.6 × 109
9. 27, 500, 000
23. 2.35 × 1012
10. 800, 300
24. 5.32 × 10βˆ’10
11. 0.00000902
25. 7.2 × 10βˆ’12
12. 0.0005486
26. 2.236 × 105
13. 8.34 × 103
27. 8.82 × 1010
14. 7.9 × 10βˆ’4
28. 8.7822 × 102
242
CHAPTER 7. ANSWERS TO EXERCISES
29. 3.0804 × 10βˆ’2
30. 6.4 × 106
31. 1.6 ×
1012
32. 4 × 10βˆ’12
33. 2 × 1012
34. 2.3785 × 103
35. 3 × 10βˆ’7
Chapter 5
Section 5.1 (Exercises on page 139.)
1. 26
19. 2x3 y 2 z
2. 6
20. 5x2
3. 20
21. 5(3x2 βˆ’ 2)
4. 9
22. 13x(2x2 βˆ’ 3)
5. 3
23. 5(2x2 + 3x βˆ’ 9)
6. 10
7. 1
8. 4
9. 4
10. 3
11. x4
12. 1
13.
xy 2
24. x(x2 + 6x βˆ’ 17)
25. βˆ’18(x6 βˆ’ x4 βˆ’ x2 + 1)
26. βˆ’y(x3 βˆ’ 2x2 βˆ’ 3x + 5)
27. 2a5 (8a3 βˆ’ 9a βˆ’ 15)
28. 8x2 y 2 (5x2 + 3y 6 βˆ’ 4xy 3 )
29. 2xy(8xy + 2x βˆ’ 3y)
30. x2 y 2 z 2 (x + 3y βˆ’ 2z)
31. (x + 2)(3 βˆ’ x)
32. x2 (x βˆ’ 1)(3x + 2)
14. x3 y 2
33. (y βˆ’ 3)(x + 5)
15. x2 y 2 z
34. (x + y)(5x + 4y)
16. 1
35. (x + 6)(3x2 βˆ’ 5x + 7)
17. 2x2 y
36. (x2 βˆ’ 5x βˆ’ 4)(7x + 8)
18. 3x2 yz 2
37. (xy + 1)(2y + 3x)
243
38. xy(x2 βˆ’ y 2 )(x βˆ’ y)
55. (4x βˆ’ 3)(4x2 + 1)
39. (5x2 βˆ’ 6)2
56. (6x βˆ’ 1)(4x2 βˆ’ 3)
40. (x2 + 9x βˆ’ 1)(3x βˆ’ 1)
57. (4x βˆ’ 1)(4x2 βˆ’ 5)
41. (x2 + 2x + 3)(x2 βˆ’ 10x βˆ’ 2)
58. (2x βˆ’ 3)(9x2 + 4)
42. (2x2 + 3xy + y)(xy βˆ’ 4x + 5y)
43. (x2 + 4x + 5)(x2 + 3x + 2)
44. (6x βˆ’ 5y)(2x + 3y + 6)
45.
13(x2
2)(x2
βˆ’
46.
5(3x2
βˆ’ 5)(3x + 14)
+ xy + y)
47. (2x + 3)(x2 + 4)
48. (5x βˆ’ 1)(x2 + 9)
49. (3x βˆ’ 2)(x2 + 1)
59. (x βˆ’ 2)(16x2 βˆ’ 3)
60. (x βˆ’ 1)(25x2 βˆ’ 7)
61. (2x + 1)(x2 + 25)
62. (9x βˆ’ 4)(x2 + 1)
63. (2a + b)(a2 βˆ’ b)
64. (2x βˆ’ 3)(y βˆ’ 4)
65. (4x2 + 1)(x βˆ’ 5)
50. (x + 3)(4x2 + 9)
66. (7x βˆ’ y)(x + 2)
51. (2x βˆ’ 1)(9x2 + 1)
67. (a + 4)(6b + 1)
52. (3x βˆ’ 4)(4x2 + 1)
68. (5x βˆ’ 4y)(2x + 3)
53. (5x βˆ’ 1)(x2 + 4)
69. (5x βˆ’ 2z)(x + 3y)
54. (7x + 3)(x2 + 9)
70. (2x + 1)(3 + 2y)
Section 5.2 (Exercises on page 148.)
1. x2 + 8x + 16
7. 4x2 βˆ’ 4x + 1
2. x2 βˆ’ 8x + 16
8. 4x2 βˆ’ 1
3. x2 βˆ’ 16
9. 4x2 + 4x + 1
4. x2 βˆ’ 20x + 100
10. 16x2 βˆ’ 9
5. x2 + 20x + 100
11. 9x2 + 42x + 49
6. x2 βˆ’ 100
12. 16x2 βˆ’ 40x + 25
244
CHAPTER 7. ANSWERS TO EXERCISES
13. 4x2 + 12xy + 9y 2
37. b = 9
14. 9x2 βˆ’ 25y 2
38. b = 1
15. 16x2 βˆ’ 16xz + 4z 2
39. b = 9
16. (x βˆ’ 4)(x + 4)
17. (x βˆ’ 9)(x + 9)
18. (3x βˆ’ 2)(3x + 2)
19. 5(x βˆ’ 2)(x + 2)
20. 9(2x βˆ’ 1)(2x + 1)
21. 7(2x βˆ’ 3)(2x + 3)
22. (x +
3)2
40. b = ±2
41. b = ±8
42. b = ±6
43. b = ±10
44. b = ±12
45. b = ±24
46. b = ±1
47. b = ±6
23. (x βˆ’ 4)2
48. b = ±10
24. (2x + 1)2
49. Yes: (x βˆ’ 7)(x + 7)
25. 2(x βˆ’ 1)2
50. Yes: (x βˆ’ 11)(x + 11)
26. 3(2x + 3)2
51. No
27. 5(x + 2)2
52. Yes: 2(x βˆ’ 2)(x + 2)
28. b = 36
29. b = 9
30. b = 49
31. b = 16
32. b = 1
33. b = 1
53. Yes: (4x βˆ’ 5)(4x + 5)
54. No
55. No
56. Yes: (x βˆ’ 7)2
57. No
58. Yes: (x + 8)2
59. No
34. b = 4
60. Yes: (3x + 1)2
35. b = 9
61. Yes: (5x + 2)2
36. b = 1
62. No
245
63. Yes: (2x βˆ’ 3)2
¡
¢¡
¢
64. x βˆ’ 12 x + 12
83. (y + 53 )2
¡
¢
84. z βˆ’ 76
65. (y βˆ’ 23 )(y + 23 )
¡
¢¡
¢
66. 32 x βˆ’ 1 32 x + 1
85. ( 19 x βˆ’ 1)2
¡
¢
86. 14 r βˆ’ 3
¡
¢2
87. 23 x + 41
¡
¢2
88. 54 x + 32
67. ( 54 y βˆ’ 1)( 54 y + 1)
¡
¢¡
¢
68. 18 x βˆ’ 5 18 x + 5
69. ( 37 x βˆ’ 4)( 37 x + 4)
¡
¢¡
¢
70. 45 x βˆ’ 27 54 x + 27
1
71. ( 11
xβˆ’
7
1
10 )( 11 x
+
7
10 )
89. (5x βˆ’ 1)(x βˆ’ 3)(x + 3)
90. (3x βˆ’ 2)(x βˆ’ 1)(x + 1)
91. (x + 3)(2x βˆ’ 3)(2x + 3)
72. (x βˆ’ 13)(x + 13)
92. (2x + 1)(3x βˆ’ 1)(3x + 1)
73. (5x βˆ’ 2)(5x + 2)
93. (3x βˆ’ 4)(2x βˆ’ 1)(2x + 1)
74. (x + 10)2
94. (5x βˆ’ 1)(x βˆ’ 2)(x + 2)
75. (y + 7)2
95. (7x + 3)(x βˆ’ 3)(x + 3)
76. (x βˆ’ 11)2
96. (4x βˆ’ 3)(2x βˆ’ 1)(2x + 1)
77. (2w βˆ’ 9)2
97. (6x βˆ’ 1)(2x βˆ’ 1)(2x + 1)
78. (xy + 2)2
98. (4x βˆ’ 1)(2x βˆ’ 1)(2x + 1)
79. (xy + 5)2
99. (2x βˆ’ 3)(3x βˆ’ 2)(3x + 2)
80. 7(2x + 3)2
100. (x + 2)(4x βˆ’ 1)(4x + 1)
81. (x βˆ’ 12 )2
101. (x βˆ’ 1)(5x βˆ’ 2)(5x + 2)
¢2
¡
82. x + 13
102. (2x + 1)(x βˆ’ 5)(x + 5)
Section 5.3 (Exercises on page 160.)
1. (x + 2)(x + 1)
4. (x + 7)(x βˆ’ 1)
2. (x βˆ’ 2)(x + 1)
5. (x βˆ’ 11)(x βˆ’ 1)
3. (x + 3)(x βˆ’ 1)
6. (x βˆ’ 11)(x + 1)
246
CHAPTER 7. ANSWERS TO EXERCISES
7. (x + 2)(x + 3)
31. prime over the integers
8. (x βˆ’ 4)(x + 2)
32. (x βˆ’ 6y)(x βˆ’ 3y)
9. (x βˆ’ 3)(x βˆ’ 4)
33. (7x βˆ’ 5y)(x + y)
10. βˆ’1(x βˆ’ 7)(x βˆ’ 2)
11. (2x + 1)(x βˆ’ 2)
12. (5x + 1)(x + 1)
13. 2(x βˆ’ 1)2
14. 5(x + 6)(x + 1)
15. 2(3x + 1)(x + 1)
16. (x +
2y)2
34. b = ±4
35. b = ±2
36. b = ±6
37. b = ±8
38. b = ±5, ±7
39. b = ±1, ±5
40. b = ±3
41. b = ±1
17. (x βˆ’ 3y)(x βˆ’ 4y)
42. b = ±9, ±11, ±19
18. (5x + y)(x + 2y)
43. b = ±6, ±10
19. (x + 2)2
44. b = ±6, ±9
20. (x + 7)(x + 2)
45. b = ±11, ±13, ±17, ±31
21. (x + 2)(x βˆ’ 1)
46. b = ±6, ±10
22. prime over the integers
23. prime over the integers
24. prime over the integers
25. (3x + 1)(x + 1)
26. 3(x + 4)(x βˆ’ 1)
27. (3x + 2)(x + 1)
28. prime over the integers
47. b = ±2, ±7
48. b = ±5, ±13
49. 4(3x βˆ’ 1)(x + 2)
50. 5x(x βˆ’ 3)(x + 3)
51. 8(2x βˆ’ y)(x βˆ’ 2y)
52. 8(2x2 βˆ’ 5)(2x2 + 5)
53. βˆ’7x2 (2x βˆ’ 3)2
¡
¢¡
¢
54. 32 y 2 βˆ’ 5 32 y 2 + 5
29. 3(x2 + 3x + 1)
55. (9t2 + 16)(3t + 4)(3t βˆ’ 4)
30. 5(x2 + 4x + 2)
56. 2(y βˆ’ 3)(y + 3)(y βˆ’ 2)(y + 2)
247
57. 2x4 (2x3 βˆ’ 7)(2x3 + 7)
67. 2(4x βˆ’ 1)(x + 6)
58. 5(9 βˆ’ a)(10 + a)
68. 2(4x βˆ’ 7)(x + 6)
59. βˆ’15n(2n + 5)2
69. 2(3x βˆ’ 10)(x + 3)
60. 3x(2x βˆ’ 1)(x + 4)
70. 5(2x + 1)(x βˆ’ 2)
61. 3(7x + 5)(x βˆ’ 3)
71. 2x(3x + 1)(x βˆ’ 3)
62. 2(3x βˆ’ 5)(2x + 1)
72. 2(7x βˆ’ 1)(2x + 1)
63. 2(2x βˆ’ 3)(3x + 1)
73. 4(2x βˆ’ 3)(x βˆ’ 1)
64. 3x(4x + 3)(x βˆ’ 3)
74. 2x2 (5x + 2)(3x βˆ’ 1)
65. 2x(6x βˆ’ 1)(x βˆ’ 5)
75. 3(4x βˆ’ 3)(x + 2)
66. 2(2x βˆ’ 5)(x + 2)
76. 6(3x βˆ’ 4)(x + 2)
Section 5.4 (Exercises on page 164.)
1. xy(2x βˆ’ y)(2x + y)
14. 9(3t2 + 4)(t + 4)
2. 5(x + 4y)(x βˆ’ 2y)
15. [(x βˆ’ 3)5 βˆ’ 1][(x βˆ’ 3)5 + 1]
3. (x3 + 3)(x βˆ’ 2)
16. 3xy(x βˆ’ y)2
4. (1 βˆ’ a βˆ’ b)(1 + a + b)
17. (2z βˆ’ w)(x βˆ’ 2y)
5. (3u βˆ’ v)(2u + 3v)
6. x2 (3 βˆ’ x2 y)(3 + x2 y)
7. (2y 2 + 3)(y βˆ’ 2)(y + 2)
8. (3x βˆ’ w)(y + 2z)
9. (x2 βˆ’ 7)(x βˆ’ 3)(x + 3)
18. (b βˆ’ 20)(b + 40)
19. (2 + x)(6 βˆ’ 7x)
20. (2x βˆ’ y)(3a + 2b)
21. 6(x + 6)(x + 2)
22. (3y βˆ’ 2x)(3y + 2x)(9y 2 + 4x2 )
23. (b βˆ’ d)(b + d)(a2 + c2 )
10. (7a βˆ’ 4)(a + 3)
24. a(3a + 5)(a + 1)
11. 2t(2t + 15)(2t + 5)
25. (4y + 3z)(y βˆ’ 8z)
12. 5(x βˆ’ 3)(x + 3)
26. 2x(3x βˆ’ 2)(3x + 2)
13. a(a βˆ’ 18)(a + 5)
27. 2xy(x βˆ’ 4y)(x + y)
248
CHAPTER 7. ANSWERS TO EXERCISES
28. (5a + b)(3c βˆ’ 4d)
50. (9 βˆ’ 5x)(1 + 2x)
29. (x + 2 βˆ’ 3y)(x + 2 + 3y)
51. (x βˆ’ 1)(25x2 βˆ’ 4y)
30. 4y(3y βˆ’ 1)(y + 4)
52. 4xy(3x βˆ’ 2y)(3x + 2y)
31. (3x2 βˆ’ 1)(6x βˆ’ 1)
53. 2x2 (5x + 2)(3x βˆ’ 1)
32. 3a2 (a βˆ’ 4)(a + 4)
54. 2(xy βˆ’ 3)2
33. (4x + 5)(10x βˆ’ 9)
55. 2x3 (x βˆ’ 4)(x βˆ’ 3)
34. (3x + 2y + 4)(3x + 2y βˆ’ 3)
35. (xy βˆ’ 15)(xy + 15)
36. (8 βˆ’ a)(a + 10)
37. (2x βˆ’ 3y)(y 2 βˆ’ 3x)
38. (x βˆ’ 7)(2x βˆ’ 15)(2x βˆ’ 13)
39. x(2x βˆ’ 5)2
40. a(a + 1)(a βˆ’ a2 βˆ’ 6)
41. (6x βˆ’ 7y)(6x + 7y)
42. (x2 + 4)(5x βˆ’ 1)
43. 2(3x βˆ’ 5)(2x + 1)
44. (2x + 3)(x + 5)
56. (8x βˆ’ 9)(x βˆ’ 1)
57. 2(4x βˆ’ 7)(x + 6)
58. (9x βˆ’ 7)(9x + 7)
59. (7 βˆ’ 2x)(2 + x)
60. (2x + 3)(x βˆ’ 2)(x + 2)
61. (4x2 + 9)(2x βˆ’ 3)(2x + 3)
62. 3(4x βˆ’ 3)(x + 2)
63. 4x2 y(x βˆ’ 3y + 2)
64. 4(x βˆ’ 4)(x + 3)
65. 2(4x βˆ’ 1)(x + 6)
45. (x βˆ’ 4y)(x βˆ’ 3y)
66. x(x βˆ’ 18)(x + 5)
46. (3x + 2)(2x βˆ’ 5)
67. 2x(2x + 5)(2x + 15)
47. 2x3 y(x βˆ’ xy + y)
68. 3x2 y(x + 6)(x βˆ’ 4)
48. 3x(x + 3)2
69. (4x + 5y)(2x βˆ’ y)
49. 2x(3xy βˆ’ 10)(xy + 3)
70. (1 βˆ’ 10x)(1 + 2x)
249
Section 5.5 (Exercises on page 174.)
1. x = βˆ’2, 3
24. x = ±3
2. x = 2, 7
√
25. x = ±1, ± 3
3. x = βˆ’4, βˆ’3
26. x = 1, 11
4. x = 0, 6
27. x = βˆ’ 12 , 2
5. x = βˆ’4
28. x = 34 , 2
6. x = βˆ’1, 4
7. x = βˆ’6, βˆ’ 12
8. x =
1 2
4, 3
1 2
9. x = βˆ’ 10
,7
29. x = βˆ’ 12 , 3
30. x = βˆ’3
31. x = 23 , 0
32. x = βˆ’ 32 , 2
33. x = βˆ’5, 3
10. x = βˆ’4, 1
34. x = βˆ’2, 52
11. x = 3, 4
35. x = 12 , 2
12. x = βˆ’5
36. x = βˆ’ 12 , 32
13. x = βˆ’3
37. x = βˆ’ 12 , 35
14. x = βˆ’1, βˆ’ 15
38. x = 0, 75
15. x = 12 , 32
39. x = βˆ’1, 9
16. x = βˆ’1, 4
17. x = βˆ’5, 2
18. x = βˆ’5
19. no rational solutions
40. x = 43 , 2
41. x = ± 54
42. x = βˆ’ 12 , 23
43. x = βˆ’ 12 , 0, 5
44. x = βˆ’ 12 , ± 13
20. x = βˆ’8, 0
45. x = ± 23 , ±1
21. no rational solutions
46. x = 49 , ±1
22. x = βˆ’2, 0, 3
47. x = βˆ’ 32 , βˆ’ 15 , 0
23. x = 0, 3, 6
48. x = ± 32 , ±1
250
CHAPTER 7. ANSWERS TO EXERCISES
49. x = 16 , ± 12
59. b = 0, ±8
50. x = ± 14 , ±1
60. b = ±7, ±8, ±13
51. x = 0, 1, 6
61. b = ±2, ±7
52. x = βˆ’5, 0, 2
62. b = ±7, ±11
53. x = βˆ’3, ± 32
63. b = ±16
54. x = ± 12 , ±2
64. b = ±3, ±7, ±17
55. x = 0, 1, 32
65. b = βˆ’10
56. b = ±6
66. b = βˆ’8
57. b = ±4
67. b = βˆ’15
58. b = ±6, ±10
68. b = 0
Chapter 6
Section 6.1 (Exercises on page 184.)
1. 9
12. not a real number
2. βˆ’11
13. βˆ’2
3
4
14. βˆ’
1
2
15. βˆ’
1
2
3.
4. βˆ’20
5. βˆ’
10
7
16. 5
6. not a real number
17. 4
7. βˆ’2
18.
8. βˆ’2
19. 0.8
9. 10
20. 0.6
3
2
5
3
21. 0.5
11. βˆ’4
22. 0.2
10. βˆ’
251
23. βˆ’0.3
37. 7|y 3 |
24. 5
38. 6x3
25. 7
39. 8x6
26. βˆ’8
40. 4x4
27. 9
41. 3|x3 |
28. 10
42. 9x6
29. 7
43. 4x8
3
5
44. 2x4
30.
31. βˆ’3
45. 4x3
32. |x7 |
46. 2x2 |y 3 |
33. x10
47. 11|x3 y|
34. |x|
48. 5x5 y 7
35. x4
49. βˆ’3x2 y 5
36. 10x2
50. 6x2 |y 5 |
Section 6.2 (Exercises on page 192.)
√
1. 2x 2x
√
2. 9x 2
√
3. βˆ’2x2 3 3
√
4. 6x 2x
√
5. x2 4 x
√
6. x
√
7. x
√
8. 3 x
p
9. y 3 x2 y
√
10. y 2 x
p
11. x3 3 y 2
√
12. 2y 2 xy
√
13. 2x 6
√
14. 3x2 y 3 2xy
√
15. 4a2 b7 2ab
√
16. 2x5 y 3 10xy
√
17. 8ab b
√
18. 8a2 b3 5b
252
CHAPTER 7. ANSWERS TO EXERCISES
√
19. 6x2 y 3 3y
√
20. 12x2 y 3 2x
√
21. 4x3 y 2y
√
22. 6xy 3 7x
√
23. 15y 2xy
√
24. 8x3 y 3x
√
25. 2xy 3 3 2x
p
26. 3xy 2 3 2x2 y
√
27. 2x2 y 3 y
p
28. 2xy 2 3 2x2 y
p
29. 2xy 2 3 5xy 2
√
30. 9x4 y 6 z 9 3xy
31. 3x3 y 4 z 8
32. 3x3 y 2 z 5
33.
34.
35.
36.
37.
38.
39.
40.
41.
42.
p
3
5y 2 z
p
4
4xy 3 z 2
√
6x5 y 6 z 10 7yz
√
3
3x2 y 3 z 4 4xz 2
√
7x5 y 3 z 3z
√
8x2 y 4 z 6 5xy
p
2x2 y 2 z 3 4 2y 3
√
8x4 y 7 z 3 3y
p
βˆ’3x5 y 2 z 3 3 4yz 2
p
2x2 y 3 z 2 4 3y 2 z 3
p
2x3 y 4 z 2 5 4x4 y 3
√
βˆ’3xy 3 3 3xy
Section 6.3 (Exercises on page 198.)
√
1. 7 3
√
2. 5 3 2
√
3. βˆ’5 3
√
4. 21 5
√
√
5. 6 3 2 + 2 2 βˆ’ 8
√
29 2
6. βˆ’
21
√
7. 10 3x
√
3 3
8.
5
√
9. x 2
√
10. 25x 2 βˆ’ 2x
√
11. 8y 3y
√
√
12. 2x 3 7 βˆ’ x 7
√
13. x
√
14. βˆ’12x 3
√
15. 16x 2
√
√
16. 15 2 βˆ’ 10 3
√
17. βˆ’9x 3x
√
18. βˆ’45 2
√
√
19. 13 3 βˆ’ 12 5
√
20. 6 x
√
21. βˆ’34 3x
√
22. βˆ’2xy 3
253
23.
24.
25.
26.
27.
28.
29.
30.
31.
32.
33.
34.
35.
36.
√
2
√
√
3+6 5
√
√
7x 2x + 5x 2
√
5+2 3
√
7 βˆ’ 3 10
√
βˆ’7 2
√
βˆ’3 3 3
√
5
βˆ’
4
√
46 3 2
√
5 2
√
9x 7x
√
βˆ’4 2
√
30 3 2
√
10 4 3
√
√
37. 29x2 3x βˆ’ 3x 3x
√
√
38. 7 2x + 5x 3x
√
39. 21 3 2
√
40. βˆ’8 3
√
41. 5 7
√
11 3
42.
4
√
43. βˆ’9 3
√
44. βˆ’11 3 2
√
45. 36 2
√
46. 3 2
√
47. 11 2x
√
10 2
48. βˆ’
9
√
49. βˆ’14 3
√
50. βˆ’7 2
Section 6.4 (Exercises on page 204.)
√
1. 3 10
2. 120
√
3. βˆ’3 3 3
√
4. βˆ’60 3
√
5. βˆ’126 3
√
6. 3ab6 2a
√
7. 5x2 y 2x
√
8. 2 βˆ’ 6
√
9. 3 3
√
10. 4 βˆ’ 2 10
√
11. 3a βˆ’ 3 ab
√
√
12. 5x 2 βˆ’ x 5
√
13. 15x βˆ’ 22y x + 8y 2
√
14. 15x + xy βˆ’ 2y
15. 3x βˆ’ y 2
√
16. x + 10 x + 25
254
CHAPTER 7. ANSWERS TO EXERCISES
√
17. 4x βˆ’ 12 x + 9
√
√
√
18. 6 βˆ’ 3 3 + 2 2 βˆ’ 6
19. x βˆ’ 16
√
20. x + 6 x + 9
√
21. x + 3 x βˆ’ 4
√
22. 16x + 8 3x + 3
√
23. 18 βˆ’ 17 6
√
√
24. 24 2 βˆ’ 12 3
√
25. 61 + 24 5
√
26. 42 βˆ’ 9 14
27. βˆ’38
√
28. 126 βˆ’ 34 3
√
29. βˆ’6 βˆ’ 19 5
√
30. 11 βˆ’ 4 6
31. 12
√
32. βˆ’102 βˆ’ 21 2
√
33. 24x 3 βˆ’ 12x
Section 6.5 (Exercises on page 212.)
√
4 6
1.
3
34. βˆ’11 +
35. 60x4
√
36. 34 βˆ’ 24 2
√
37. 57 βˆ’ 40 2
√
38. 44 βˆ’ 25 5
39. 29
√
√
40. 40 3 βˆ’ 24 5
√
41. βˆ’32 4 2
√
42. 42 + 18 14
√
43. 138 + 69 2
√
44. 62 βˆ’ 20 6
√
45. 130 βˆ’ 50 3
√
46. 40 βˆ’ 20 2
√
47. 12 βˆ’ 3 5
√
48. 16x + 8x 7x + 7x2
√
49. βˆ’36 3 3
50. 40
5.
√
3
6.
√
2
3.
4
7.
2.
√
3x
4.
x
√
2
8.
√
11
2
√
2 2
3
√
6
3
√
55
5
255
9.
10.
11.
12.
13.
14.
15.
16.
17.
18.
19.
20.
21.
22.
23.
24.
25.
√
2
√
2 6
3
√
3 3
√
3
2
√
5
3
√
2
2
√
6
4
√
14
7
√
8 7
7
√
3
2
√
2 10
5
√
2 5
5
√
2 3
3
√
7 2
12
√
3 7
√
4
5 x3
x
√
10x
4
26.
27.
28.
29.
30.
31.
32.
33.
34.
35.
36.
37.
38.
39.
40.
41.
42.
43.
√
5 3
4
√
2 6
5
√
2 2
√
20 3
√
4 3
3
√
734
2
√
3
18
3
√
5
3 x3
x
√
242
√
7
8 x4
x
√
533
3
√
βˆ’3( 7 βˆ’ 3)
√
2(3 + 2)
7
√
βˆ’2(1 βˆ’ 3)
√
√
5( 7 βˆ’ 5)
√
3( 5 βˆ’ 1)
2
√
5( 3 + 6)
βˆ’
11
√
5+ 3
2
256
44.
45.
46.
47.
48.
49.
50.
51.
52.
53.
54.
CHAPTER 7. ANSWERS TO EXERCISES
√
4( 7 + 4)
βˆ’
3
√
√
3( 5 βˆ’ 3)
√
√
5( 7 + 3)
4
√
√
5( 6 βˆ’ 3)
√
βˆ’4( 7 βˆ’ 4)
3
√
βˆ’6(2 + 6)
√
√
7( 3 βˆ’ 7)
2
√
βˆ’2(1 + 5)
√
√
7+ 3
2
√
βˆ’( 5 + 3)
2
√
βˆ’4(2 βˆ’ 6)
Section 6.6 (Exercises on page 224.)
√
√
√
3
1. 4 x2 + 3 3 x βˆ’ 2 x
√
√
√
3
4
2. 5 x + 2 x2 + 3 x3
√
4
2
3. √
+√
+6 x
3
3
2
x
x
√
3
3
73x
4. √
+ √ +
3
5
4 x2 8 x
√
√
3
11 4 x
7
4
5. 7 x βˆ’ √
+
5
12
x3
√
3
√
√
5 x2
4
βˆ’ 3 5 x + 6 x3
6.
6
√
55. 7(2 2 + 5)
√
56. 4(3 + 6)
√
√
9( 10 βˆ’ 6)
57.
2
√
βˆ’2( 7 + 4)
58.
3
√
59. βˆ’3( 11 + 4)
√
60. 3 2 βˆ’ 4
√
√
4( 3 + 2 5)
61.
17
√
√
62. βˆ’2( 11 + 3 2)
√
24(5 βˆ’ 2 3)
63.
13
√
√
(2 + 2)(5 + 3)
64.
22
√
√
(1 βˆ’ 7)(4 βˆ’ 3 2)
65. βˆ’
2
√
√
4
3
7. 2 x3 βˆ’ 6 x2 βˆ’ √
x
√
9
2
5
8. √
+
6
x3 βˆ’ √
4
3
x
x
9. x2/5 + x1/3 βˆ’ 2x1/2
10. 5x4/7 + 8x1/2 + 2x2/3 βˆ’ 5x1/4
11. 7x2/5 + 3x2/3 βˆ’ 4x3/4
12. 9x1/3 βˆ’ 5xβˆ’3/4 βˆ’ 2x5/2
13.
6 βˆ’1/2
x
βˆ’ 2xβˆ’1/3 + 8x1/4
5
257
14.
1 βˆ’1/3
x
+ 3x3/2 βˆ’ 4xβˆ’1/3
2
1
15. 4x4/5 βˆ’ xβˆ’1/2 + 5x3/4
3
1
16. x1/3 + 3x2/5 βˆ’ 6x5/4
4
17. 6
35. βˆ’7
36. 27
37. 3
38. βˆ’
39.
16
81
40.
8
27
18. 16
19. 27
4
3
20.
1
5
41.
21.
1
8
9
4
42.
22.
1
2
5
13
43. 9
23.
1
9
24. 32
44. 38
45. βˆ’1
46. 3
25. 64
1
26.
27
47.
2
5
48. 1
27. 1
28. βˆ’2
29. 40
30.
1
8
31. βˆ’4
1
32.
125
33.
1
27
34.
1
25
49. βˆ’
1
2
50. βˆ’
1
8
51. βˆ’
1
3
52. 5
53.
62
5
54.
9
8
55. 64x2 y 3
258
56.
CHAPTER 7. ANSWERS TO EXERCISES
1
x3/4
57. x5/12
2
58.
5
1
59. 5/12
x
60. x7/4
1
61. 17/10
x
62. 16a1/2 b4
63. 9x6 y 8
64. 64x6 y 9
65. x2/3
66.
2
3x1/3 y 1/2
67.
x13/12
y 2/3
68.
x5/4 y
4
Appendix A
Formulas
Area Formulas:
β€’ Area of square:
Asquare = s2 = side2
β€’ Area of a rectangle:
Arectangle = l × w = length × width
β€’ Area of a triangle:
Atriangle =
1
1
× b × h = × base × height
2
2
β€’ Area of a parallelogram:
Aparallelogram = b × h = base × height
β€’ Area of a trapezoid:
Atrapezoid =
1
1
× h (b1 + b2 ) = × height · (sum of the bases)
2
2
β€’ Area of a circle:
Acircle = Ο€r2 = Ο€(radius)2
Perimeter and Circumference Formulas:
β€’ Perimeter:
Pperimeter = distance around
In other words, add each side together.
259
260
APPENDIX A. FORMULAS
β€’ Circumference of a circle:
Ccircle = 2Ο€r = 2Ο€(radius)
Angle Properties:
β€’ Vertical Angles are opposite angles formed by intersecting lines. Vertical
angles always have the same measurement.
2
3
1
4
In the above figure, 1 and 3 are vertical angles; 2 and 4 are vertical angles.
β€’ Complementary Angles are two angles whose sum is 90β—¦ .
1
2
β€’ Supplementary Angles are two angles whose sum is 180β—¦ .
1
2
β€’ Corresponding Angles have the same location relative to lines `, m and
transversal t.
(IMPORTANT: ` k m if and only if corresponding angles formed by `, m,
and t are
congruent.) In Figure A-1, ∠1 and ∠5 are corresponding angles. The following pairs are also corresponding angles: ∠2 and ∠6; ∠3 and ∠7; ∠4 and ∠8.
261
t
1
3
2
4
l
5
m
7
6
8
Figure A-1
β€’ Alternate Interior Angles are nonadjacent angles formed by lines `, m,
and transversal t, the union of whose interiors contain the region between `
and m.
(IMPORTANT: ` k m if and only if alternate interior angles formed by
`, m and t are congruent.) In Figure A-1, ∠3 and ∠6 are alternate interior
angles. Likewise, ∠4 and ∠5 are also alternate interior angles.
β€’ Alternate Exterior Angles are angles on the outer sides of two lines cut
by a transversal, but on opposite sides of the transversal (IMPORTANT:
` k m if and only if alternate exterior angles formed by `, m and t are congruent.) In Figure A-1, ∠2 and ∠7 are alternate exterior angles. Similarly,
∠1 and ∠8 are alternate exterior angles.
β€’ Interior Angles on the same side of the transversal are interior angles
whose interiors are the same. (IMPORTANT: ` k m if and only if the
interior angles on the same side of the transversal are supplementary.) In
Figure A-1, ∠3 and ∠5, as well as ∠4 and ∠6, are interior angles on the same
side of the transversal.